Примеры на умножение деление сложение вычитание: примеры на сложение, вычитание, умножение и деление — РОСТОВСКИЙ ЦЕНТР ПОМОЩИ ДЕТЯМ № 7

Содержание

примеры на сложение, вычитание, умножение и деление — РОСТОВСКИЙ ЦЕНТР ПОМОЩИ ДЕТЯМ № 7

Содержание

примеры на умножение и деление, сложение и вычитание

Ваш ребенок еще только учится в начальной школе, а вы уже задумываетесь о его дальнейшей учебе, развитии и будущем? Это очень похвально. А думали ли вы над тем, что успеваемость ребенка можно улучшить, если заниматься с ним ежедневно по математике всего лишь 15 минут в день дополнительно? И это не выдумки. В материалах этой статьи мы приведем примеры и задачи для школьников начальной школы по математике, а именно, для третьеклассников. (Для удобства решения приведенные ниже задания вы можете распечатать).

Как учить ребенка учиться

Умеет ли ваш ребенок учиться? Уверена, что многих родителей этот вопрос поставил в тупик. А действительно, что значит «уметь учиться»? Когда ваш юный школьник только пошел в школу, после занятий, возможно, он бежал домой и очень хотел сразу же делать уроки. Так бывает, когда дети очень ждут поступления в 1 класс. Но со временем интересы к своевременному выполнению домашнего задания ослабевают и «домашка» становится скучным времяпровождением.

А ведь именно нежелание выполнять домашние задания, готовиться к школьным рефератам, семинарам и викторинам, становится основной причиной того, что ребенок вначале не хочет, а после и не умеет учиться. Пробелы в знаниях могут накапливаться словно снежный ком, снижая успеваемость школьника и убивая в нем желание учиться.

Чтобы школьник учился этой сложной и ответственной науке – учиться – родители должны всячески помогать ему: составить распорядок дня, учить ребенка выполнять домашнее задание наперед, прорешивать или прописывать дополнительные упражнения, чтобы тренировать и руку для письма, и мозг для устного счета. Математике дается детям начального звена сложнее всего, именно поэтому мы и подготовили для школьников 3 класса этот материал.

Примеры по математике на умножение и деление

Еще во втором классе дети выучили таблицу умножения. Если вы сейчас находитесь в полном заблуждении, как выучить с ребенком таблицу умножения, то рекомендуем к ознакомлению следующий материал по ссылке. На протяжении второго класса школьники постепенно осваивали простые примеры и задачи, используя таблицу умножения, а в третьем классе они оттачивают навыки умножения и сложения.

Задание 1

Заменить сложение вычитанием в тех примерах, в которых от замены знака ответ не изменится:

5 + 5 + 5 =
1 + 1 + 1 + 1 =
0 + 0 + 0 + 0 + 0 =
8 + 8 + 8 + 8 =
7 + 7 — 7 + 7 =
7 + 7 + 7 — 7 =
14 + 14 =
61 + 61 =

Подсказка:

5 + 5 + 5 = 15, если заменить знак «+» на знак «•», то получится
5 • 5 • 5 = 125. 15 не равно 125. Значит, в первом равенстве заменить знак «+» на знак «•» нельзя.

По аналогии решаем стальные равенства и делаем выводы о возможной или невозможной замене знака «+» на знак «•».

Задание 2

Какие выражения нельзя заменить суммой, чтобы ответ не изменился:

0 • 4 =
1 • 0 =
1 • 1 =
1 • 6 =
0 • 9 =
7 • 0 =
5 • 2 =
2 • 2 =

Подсказка:

Вспомните, каким правилом следует пользоваться при умножении на ноль.

Задание 3

Решите примеры:

45 : 5 + 1 =
45 : 5 • 1 =
543 — 5 • 1 =
(543 — 5) • 1 =
423 + 7 • 0 =
(423 + 7) • 1 =
10 — 0 + 4 =
10 • 0 + 4 =

Задание 4

Из каждого выражения на умножение составьте выражения на деление:

6 • 8 =
7 • 1 =
4 • 0 =
0 • 3 =
4 • 9 =

Подсказка

6 • 8 = 48
48 : 8 = 6
48 : 8 = 6

Задание 5

Какое значение имеют следующие выражение:

а : а =
а : 1 =
0 : а =
а : 0 =

Задание 6

Решите примеры:

(596 + 374) • 1 =
596 + 374 • 1 =
(596 + 374) • 0 =
596 + 374 + 0 =
0 • 320 : 1 =
0 + 320 : 1 =

Обязательно повторите с ребенком правила умножения и деления числа на единицу и умножения или деления числа на ноль, а также особенности деления ноля на любое число. Часто именно в этих примерах дети делают ошибки, которые влекут за собой дальнейшее неправильное решение примеров, выражений и задач.

Задание 7 (задача)

В оздоровительный лагерь привезли фрукты: 7 ящиков винограда и 5 ящиков персиков. Масса привезенных персиков составляет 40 килограммов. Какая масса винограда, если ящик винограда на 1 килограмм весит больше, чем ящик персиков.

Решение

Найдем, сколько весит один ящик персиков. Известно, что общая масса персиков составляет 40 кг, а всего ящиков – 5.

Первое действие:
40 : 5 = 8 (кг) весит один ящик персиков.

Теперь найдем, сколько весит один ящик винограда, если известно, что он тяжелее на 1 кг, чем ящик персиков.

Второе действие:
8 + 1 = 9 (кг) весит один ящик винограда.

Теперь находим общую массу всего винограда, если известно, что один ящик весит 9 кг, а всего винограда – 7 ящиков.

Третье действие:
9 • 7 = 63 (кг) – общая масса винограда.

Ответ: масса привезенного винограда составляет 63 кг.

Задание 8

Сосна может расти 600 лет, береза – 350 лет. А ива – в 6 раз меньше от сосны. Что может расти дольше береза или ива? И насколько лет?

Решение

Вначале рассчитаем, сколько лет может расти ива, если известно, что она растет в 6 раз меньше, чем сосна.

Первое действие:
600 : 6 = 100 (лет) может расти ива.

Теперь, когда известно, что ива может расти 100 лет, сравним продолжительность «жизни» березы и ивы. Известно, что береза растет 350 лет, а ива – 100. 350 больше чем 100, значит береза может расти дольше ивы. Чтобы рассчитать, на сколько береза может расти дольше ивы, решаем равенство.

Второе действие:
350 — 100 = 250 (лет) – на столько береза может расти дольше ивы

Ответ: береза может расти дольше ивы на 250 лет.

Важно! Если задачу можно решить несколькими способами, обязательно сообщите об этом ребенку. Пусть потренирует логику и начертит все возможные схем решения задачи, т.е. составить схематическое условие. Ведь правильно составленное условие задачи – это 90% успешного решения.

Задание 9

В понедельник гусеница начала ползти вверх по дереву высотой 9 метров. За день она поднялась вверх на 5 метров, а за ночь – опустилась на 2 метра. На какой день гусеница достигнет верхушки дерева?

Решение

Для начала рассчитаем, на сколько метров поднимается гусеница вверх за один день, с учетом того, что ночью на опускается.

Первое действие:
5 — 2 = 3 (м) гусеница проползает за сутки вверх.

Теперь найдем количеств дней, необходимых на преодоление расстояния 9 метров вверх по дереву.

Второе действие:
9 : 3 = 3 (дня) нужно гусенице, чтобы достичь вершины дерева.

Ответ: 3 дня нужно гусенице, чтобы достичь вершины дерева.

Задание 10

В коробке было 18 килограммов печенья. Сначала из нее взяли 13 килограммов печенья, потом досыпали в 4 раза больше, чем оставалось. Сколько килограммов печенья стало в коробке.

Решение

Сначала найдем, сколько килограммов печенья осталось в коробке, после того, как из нее забрали 13 килограммов.

Первое действие:
18 — 13 = 5 (кг) печенья осталось в коробке

Теперь рассчитаем сколько килограммов печенья досыпали в коробку.

Второе действие:
5 • 4 = 20 (кг) досыпали

Сложим тот вес, который оставался в коробке, и тот, который досыпали, чтобы найти, сколько килограммов печения стало в коробке.

Третье действие:
5 + 20 = 25 (кг) стало

Ответ: 25 килограммов печения стало в коробке.

Задание 11

За лето хозяйка вырастила 208 домашних птиц. Кур и уток было 129, а уток и гусей – 115. Сколько кур, уток и гусей вырастила хозяйка за лето?

Решение

Известно, что кур и уток было 129, а всего птиц – 208. Значит, можно найти количество гусей.

Первое действие:
208 (птиц) – 129 (уток + кур) = 79 гусей

Также известно, что уток и гусей всего 115, значит мы можем найти, сколько было кур.

Второе действие:
208 (птиц) – 115 (уток + гусей) = 93 кур

Теперь, когда мы знаем количество гусей и кур, а также общее количество домашних птиц, мы можем найти количество уток.

Третье действие:
208 — (79 + 93) = 36 уток

Ответ: за лето хозяйка вырастила 79 гусей, 93 кур и 36 уток.

Второй вариант решения

Известно, что кур и уток было 129, а всего птиц – 208. Значит, можно найти количество гусей.

Первое действие:
208 (птиц) – 129 (уток + кур) = 79 гусей

Также известно, что уток и гусей всего 115, значит мы можем найти, сколько было уток

Второе действие:
115 (уток + гусей) – 79 (гусей) = 36 уток

Теперь, когда мы знаем количество гусей и уток по отдельности, а также общее количество домашних птиц, мы можем найти количество кур.

Третье действие:
208 – (79 + 36) = 208 – 115 = 93 кур

Ответ: за лето хозяйка вырастила 79 гусей, 93 кур и 36 уток.

Примеры и задачи по математике на сложение и вычитание

Основной задачей заданий и примеров по математике на сложение и вычитание в третьем классе является популяризация математических знаний и идей, поддержка и развитие математических знаний школьников, стимулирование и мотивация учеников в изучении естественно-математический предметов.

Задание 1

Реши уравнения:

Х – 40 = 60
Х + 4 = 61
Х – 16 = 25
Х + 25 = 84
Х – 45 = 251
Х + 56 = 106
Х + 78 = 301

Задание 2

Расставьте скобки так, чтобы ответом выражения в первом случае было 6, а в втором – 2:

12 : 2 + 2 • 2 =

Подсказка

12 : (2 + 2) • 2 = 6
12 : (2 + 2 • 2) = 2

Важно! Некоторые условия составлены таким образом, чтобы ребенок включал логическое мышление. Прорешивая такие задания он мыслит, делает предположения, размышляет, и находит правильное решение задания.

Задание 3

Перевести в одну систему измерения и решить выражения:

1 м – 5 дм =
1 м – 5 см =
6 м 5 дм – 8 дм =
5 см + 5 см =
15 см + 5 дм =
3 дм – 6 см =
3 дм 5 см – 15 см =
1 дм 2 см – 3 см =
1 м 6 дм – 8 дм =

Задание 4

Из каждого выражения произведения отнять 15 и записать новые выражение и решить их:

7 • 3 =
7 • 6 =
7 • 9 =
8 • 6 =
8 • 4 =
3 • 9 =
4 • 4 =
5 • 7 =

Подсказка

Если 7 • 3 = 21, то 21 – 15 = 6

Задание 5

Решить примеры:

7 • 6 + 7 • 4 =
21 : 3 – 6 =
(35 – 28) • 5 =
(68 – 26) : 7 =
7 + (6 : 2) =
3 – 14 : 2 =
60 – 63 : 7 =
81 – 56 : 7 =
50 + 42 : 7 =

Задание 6 (задача)

В шести одинаковых бочонках 24 литра воды. Сколько литров воды в сети таких же бочонках, на сколько литров больше во втором случае, чем в первом?

Решение

Вначале найдем, сколько воды вмещается в один бочонок.

Первое действие:
24 : 6 = 4 (л) в одном бочонке

Теперь рассчитаем, сколько воды в семи одинаковых бочонках

Второе действие:
4 • 7 = 28 (л) в сети одинаковых бочонках

Найдем ответ на главный вопрос задачи, на сколько литров больше во втором случае, чем в первом.

Третье действие:
28 – 24 = 4 (л) на столько литров больше во втором случае, чем в первом

Ответ: на 4 литра воды больше во втором случае, чем в первом

Задание 7

Отец и сын купили на рынке картошку в 6 одинаковых сетках. Отец принес домой 4 сетки, а сын 2. Всего получилось 18 килограммов картошки. Сколько килограммов принес отец? Сколько килограммов принес сын? На сколько больше килограммов картошки принес отец?

Решение

Рассчитаем, сколько картошки было в одной сетке, если известно, то всего принести 18 килограммов в 6 одинаковых сетках.

Первое действие:
18 : 6 = 3 (кг) в одной сетке.

Теперь узнаем сколько килограммов принес отец и сколько килограммов принес сын.

Второе действие:
3 • 4 = 12 (кг) принес отец

Третье действие:
3 • 2 = 6 (кг) принес сын

Найдем искомую разницу.

Четвертое действие:
12 – 6 = 6 (кг) на столько больше принес отец.

Ответ: Отец принес на 6 килограммов больше картошки, чем сын.

Задание 8

За 5 часов работы двигателя было израсходовано 30 литров бензина. Сколько бензина будет израсходовано за 8 часов работы двигателя. На сколько больше двигатель израсходует бензина за разницу во времени?

Решение

Рассчитаем, сколько бензина расходует двигатель за час своей работы.

Первое действие:
30 : 5 = 6 (л) за один час работы

Рассчитаем, сколько составляет разница во времени?

Второе действие:
8 – 5 = 3 (ч) разница во времени

Теперь можно рассчитать, сколько бензина израсходовано за оставшиеся 3 часа.

Третье действие:
3 • 6 = 18 (л) потрачено за 3 часа.

Ответ: за 3 часа двигатель истратил 18 литров бензина

Второй способ решения

Рассчитаем, сколько бензина расходует двигатель за час своей работы.

Первое действие:
30 : 5 = 6 (л) за один час работы

Рассчитаем, сколько бензина будет израсходовано за 8 часов работы двигателя.

Второе действие:
8 • 6 = 48 (л) израсходовано за 8 часов работы двигателя

Теперь можно рассчитать разницу потраченного топлива.

Третье действие:
48 – 30 = 18 (л) разница потраченного топлива

Ответ: за 3 часа двигатель истратил 18 литров бензина

Важно! Задания на сложение и вычитание не исключают в своем условии или решении возможность других математических действий, например, умножения или деления. Ученик третьего класса уже должен уметь различать в условии требования к сложению и умножению, делению и вычитанию. Именно потому задания по математике для этого класса часто носят смешанный характер.

Задание 9

В двух прудах плавало 56 уток. Когда из первого пруда во второй перелетело 7 уток, то в нем осталось 25. Сколько уток с самого начала плавало во втором пруду?

Решение

Известно, что после того, как из первого пруда улетело 7 уток, в нем осталось 25. Находим количество уток в первом пруду с самого начала.

Первое действие:
7 + 25 = 32 (утки) было в первом пруду.

Теперь можем найти, сколько уток плавало во втором пруду с самого начала.

Второе действие:
56 – 32 = 24 (утки) было во втором пруду.

Ответ: с самого начала во втором пруду было 24 утки.

Задание 10

С первого куста собрали 9 килограммов ягод. Со второго куста собрали на 3 килограммов больше, чем с первого, а с третьего – на 2 килограммов больше, чем со второго. Сколько килограммов ягод собрали с третьего куста? Сколько всего ягод собрали?

Решение

Вначале найдем, сколько килограммов ягод собрали со второго куста.

Первое действие:
9 + 3 = 12 (кг) ягод со второго куста

Теперь определяем, сколько килограммов ягод собрали с третьего куста

Второе действие:
12 + 2 = 14 (кг) год с третьего куста

Когда все составляющие известны, находим ответ на главный вопрос задачи.

Третье действие:
9 + 12 + 14 = 35 (кг) ягод всего

Ответ: всего собрали 35 килограммов ягод.

Вместо заключения

Уделяйте математике достаточно внимания уже с начальной школы. Этот предмет не только тренируем мозг в устном счете, но и умении логически мыслить, развивать смекалку. Постепенно привыкая к выполнению дополнительных и основных заданий, ребенок учится учиться, выполнять требования учителя, грамотно планировать свое время, распределять время для учебы и досуга.

Математические задания для третьеклассников моно составлять самостоятельно по приведенным нами аналогии, это не составит особого труда. Зато ваш ученик сможет больше тренироваться в математике, выполнять задания на каникулах и выходных, а также заниматься дополнительно после школы.

Порядок решения примеров с умножением и делением. Учебно-методический материал по математике (3 класс) на тему: Примеры на порядок действий

Числовые,буквенные выражения и выражения с переменными в своей записи могут содержать знаки различных арифметических действий. При преобразовании выражений и вычислении значений выражений действия выполняются в определенной очередности, иными словами, нужно соблюдать порядок выполнения действий .

В этой статье мы разберемся, какие действия следует выполнять сначала, а какие следом за ними. Начнем с самых простых случаев, когда выражение содержит лишь числа или переменные, соединенные знаками плюс, минус, умножить и разделить. Дальше разъясним, какого порядка выполнения действий следует придерживаться в выражениях со скобками. Наконец, рассмотрим, в какой последовательности выполняются действия в выражениях, содержащих степени, корни и другие функции.

Навигация по странице.

В школе дается следующее правило, определяющее порядок выполнения действий в выражениях без скобок :

  • действия выполняются по порядку слева направо,
  • причем сначала выполняется умножение и деление, а затем – сложение и вычитание.
  • Озвученное правило воспринимается достаточно естественно. Выполнение действий по порядку слева направо объясняется тем, что у нас принято вести записи слева направо. А то, что умножение и деление выполняется перед сложением и вычитанием объясняется смыслом, который в себе несут эти действия.

    Рассмотрим несколько примеров применения этого правила. Для примеров будем брать простейшие числовые выражения, чтобы не отвлекаться на вычисления, а сосредоточиться именно на порядке выполнения действий.

    Выполните действия 7−3+6 .

    Исходное выражение не содержит скобок, а также оно не содержит умножения и деления. Поэтому нам следует выполнить все действия по порядку слева направо, то есть, сначала мы от 7 отнимаем 3 , получаем 4 , после чего к полученной разности 4 прибавляем 6 , получаем 10 .

    Кратко решение можно записать так: 7−3+6=4+6=10 .

    Укажите порядок выполнения действий в выражении 6:2·8:3 .

    Чтобы ответить на вопрос задачи, обратимся к правилу, указывающему порядок выполнения действий в выражениях без скобок. В исходном выражении содержатся лишь действия умножения и деления, а согласно правилу, их нужно выполнять по порядку слева направо.

    сначала 6 делим на 2 , это частное умножаем на 8 , наконец, полученный результат делим на 3.

    Вычислите значение выражения 17−5·6:3−2+4:2 .

    Сначала определим, в каком порядке следует выполнять действия в исходном выражении. Оно содержит и умножение с делением, и сложение с вычитанием. Сначала слева направо нужно выполнить умножение и деление. Так 5 умножаем на 6 , получаем 30 , это число делим на 3 , получаем 10 . Теперь 4 делим на 2 , получаем 2 . Подставляем в исходное выражение вместо 5·6:3 найденное значение 10 , а вместо 4:2 — значение 2 , имеем 17−5·6:3−2+4:2=17−10−2+2 .

    В полученном выражении уже нет умножения и деления, поэтому остается по порядку слева направо выполнить оставшиеся действия: 17−10−2+2=7−2+2=5+2=7 .

    На первых порах, чтобы не перепутать порядок выполнения действий при вычислении значения выражения, удобно над знаками действий расставить цифры, соответствующие порядку их выполнения. Для предыдущего примера это выглядело бы так: .

    Этого же порядка выполнения действий – сначала умножение и деление, затем сложение и вычитание — следует придерживаться и при работе с буквенными выражениями.

    Действия первой и второй ступени

    В некоторых учебниках по математике встречается разделение арифметических действий на действия первой и второй ступени. Разберемся с этим.

    Действиями первой ступени называют сложение и вычитание, а умножение и деление называют действиями второй ступени .

    В этих терминах правило из предыдущего пункта, определяющее порядок выполнения действий, запишется так: если выражение не содержит скобок, то по порядку слева направо сначала выполняются действия второй ступени (умножение и деление), затем – действия первой ступени (сложение и вычитание).

    Порядок выполнения арифметических действий в выражениях со скобками

    Выражения часто содержат скобки, указывающие порядок выполнения действий. В этом случае правило, задающее порядок выполнения действий в выражениях со скобками , формулируется так: сначала выполняются действия в скобках, при этом также по порядку слева направо выполняется умножение и деление, затем – сложение и вычитание.

    Итак, выражения в скобках рассматриваются как составные части исходного выражения, и в них сохраняется уже известный нам порядок выполнения действий. Рассмотрим решения примеров для большей ясности.

    Выполните указанные действия 5+(7−2·3)·(6−4):2 .

    Выражение содержит скобки, поэтому сначала выполним действия в выражениях, заключенных в эти скобки. Начнем с выражения 7−2·3 . В нем нужно сначала выполнить умножение, и только потом вычитание, имеем 7−2·3=7−6=1 . Переходим ко второму выражению в скобках 6−4 . Здесь лишь одно действие – вычитание, выполняем его 6−4=2 .

    Подставляем полученные значения в исходное выражение: 5+(7−2·3)·(6−4):2=5+1·2:2 . В полученном выражении сначала выполняем слева направо умножение и деление, затем – вычитание, получаем 5+1·2:2=5+2:2=5+1=6 . На этом все действия выполнены, мы придерживались такого порядка их выполнения: 5+(7−2·3)·(6−4):2 .

    Запишем краткое решение: 5+(7−2·3)·(6−4):2=5+1·2:2=5+1=6 .

    Бывает, что выражение содержит скобки в скобках. Этого бояться не стоит, нужно лишь последовательно применять озвученное правило выполнения действий в выражениях со скобками. Покажем решение примера.

    Выполните действия в выражении 4+(3+1+4·(2+3)) .

    Это выражение со скобками, это означает, что выполнение действий нужно начинать с выражения в скобках, то есть, с 3+1+4·(2+3) . Это выражение также содержит скобки, поэтому нужно сначала выполнить действия в них. Сделаем это: 2+3=5 . Подставив найденное значение, получаем 3+1+4·5 . В этом выражении сначала выполняем умножение, затем – сложение, имеем 3+1+4·5=3+1+20=24 . Исходное значение, после подстановки этого значения, принимает вид 4+24 , и остается лишь закончить выполнение действий: 4+24=28 .

    Вообще, когда в выражении присутствуют скобки в скобках, то часто бывает удобно выполнение действий начинать с внутренних скобок и продвигаться к внешним.

    Например, пусть нам нужно выполнить действия в выражении (4+(4+(4−6:2))−1)−1 . Сначала выполняем действия во внутренних скобках, так как 4−6:2=4−3=1 , то после этого исходное выражение примет вид (4+(4+1)−1)−1 . Опять выполняем действие во внутренних скобках, так как 4+1=5 , то приходим к следующему выражению (4+5−1)−1 . Опять выполняем действия в скобках: 4+5−1=8 , при этом приходим к разности 8−1 , которая равна 7 .

    Порядок выполнения действий в выражениях с корнями, степенями, логарифмами и другими функциями

    Если в выражение входят степени, корни, логарифмы, синус, косинус, тангенс и котангенс, а также другие функции, то их значения вычисляются до выполнения остальных действий, при этом также учитываются правила из предыдущих пунктов, задающие порядок выполнения действий. Иными словами, перечисленные вещи, грубо говоря, можно считать заключенными в скобки, а мы знаем, что сначала выполняются действия в скобках.

    Рассмотрим решения примеров.

    Выполните действия в выражении (3+1)·2+6 2:3−7 .

    В этом выражении содержится степень 6 2 , ее значение нужно вычислить до выполнения остальных действий. Итак, выполняем возведение в степень: 6 2 =36 . Подставляем это значение в исходное выражение, оно примет вид (3+1)·2+36:3−7 .

    Дальше все понятно: выполняем действия в скобках, после чего остается выражение без скобок, в котором по порядку слева направо сначала выполняем умножение и деление, а затем – сложение и вычитание. Имеем (3+1)·2+36:3−7=4·2+36:3−7= 8+12−7=13 .

    Другие, в том числе и более сложные примеры выполнения действий в выражениях с корнями, степенями и т.п., Вы можете посмотреть в статье вычисление значений выражений.

    cleverstudents.ru

    Онлайн игры,тренажеры,презентации,уроки,энциклопедии,статьи

    Post navigation

    Примеры со скобками, урок с тренажерами.

    Мы рассмотрим в этой статье три варианта примеров:

    1. Примеры со скобками (действия сложения и вычитания)

    2. Примеры со скобками (сложение, вычитание, умножение, деление)

    3. Примеры, в которых много действий

    1 Примеры со скобками (действия сложения и вычитания)

    Рассмотрим три примера. В каждом из них порядок действий обозначен цифрами красного цвета:

    Мы видим, что порядок действий в каждом примере будет разный, хотя числа и знаки одинаковые. Это происходит потому, что во втором и третьем примере есть скобки.

  • Если в примере нет скобок , мы выполняем все действия по порядку, слева направо.
  • Если в примере есть скобки , то сначала мы выполняем действия в скобках, и лишь потом все остальные действия, начиная слева направо.
  • *Это правило для примеров без умножения и деления. Правила для примеров со скобками, включающих действия умножения и деления мы рассмотрим во второй части этой статьи.

    Чтобы не запутаться в примере со скобками, можно превратить его в обычный пример, без скобок. Для этого результат, полученный в скобках, записываем над скобками, далее переписываем весь пример, записывая вместо скобок этот результат, и далее выполняем все действия по порядку, слева направо:

    В несложных примерах можно все эти операции производить в уме. Главное — сначала выполнить действие в скобках и запомнить результат, а затем считать по порядку, слева направо.

    А теперь — тренажеры!

    1) Примеры со скобками в пределах до 20.

    Онлайн тренажер.

    2) Примеры со скобками в пределах до 100. Онлайн тренажер.

    3) Примеры со скобками. Тренажер №2

    4) Вставь пропущенное число — примеры со скобками. Тренажер

    2 Примеры со скобками (сложение, вычитание, умножение, деление)

    Теперь рассмотрим примеры, в которых кроме сложения и вычитания есть умножение и деление.

    Сначала рассмотрим примеры без скобок:

  • Если в примере нет скобок , сначала выполняем действия умножения и деления по порядку, слева направо. Затем — действия сложения и вычитания по порядку, слева направо.
  • Если в примере есть скобки , то сначала мы выполняем действия в скобках, затем умножение и деление, и затем — сложение и вычитание начиная слева направо.
  • Есть одна хитрость, как не запутаться при решении примеров на порядок действий. Если нет скобок, то выполняем действия умножения и деления, далее переписываем пример, записывая вместо этих действий полученные результаты. Затем выполняем сложение и вычитание по порядку:

    Если в примере есть скобки, то сначала нужно избавиться от скобок: переписать пример, записывая вместо скобок полученный в них результат. Затем нужно выделить мысленно части примера, разделенные знаками «+» и «-«, и посчитать каждую часть отдельно. Затем выполнить сложение и вычитание по порядку:

    3 Примеры, в которых много действий

    Если в примере много действий, то удобнее будет не расставлять порядок действий во всем примере, а выделить блоки, и решить каждый блок отдельно. Для этого находим свободные знаки «+» и «–» (свободные — значит не в скобках, на рисунке показаны стрелочками).

    Эти знаки и будут делить наш пример на блоки:

    Выполняя действия в каждом блоке не забываем про порядок действий, приведенный выше в статье. Решив каждый блок, выполняем действия сложения и вычитания по порядку.

    А теперь закрепляем решение примеров на порядок действий на тренажерах!

    1. Примеры со скобками в пределах чисел до 100, действия сложения, вычитания, умножения и деления. Онлайн тренажер.

    2. Тренажер по математике 2 — 3 класс «Расставь порядок действий (буквенные выражения).»

    3. Порядок действий (расставляем порядок и решаем примеры)

    Порядок действий в математике 4 класс

    Начальная школа подходит к концу, скоро ребёнок шагнёт в углубленный мир математики. Но уже в этот период школьник сталкивается с трудностями науки. Выполняя простое задание, ребёнок путается, теряется, что в результате приводит к отрицательной отметке за выполненную работу. Чтобы избежать подобных неприятностей, нужно при решении примеров, уметь ориентироваться в порядке, по которому нужно решать пример. Не верно распределив действия, ребёнок не правильно выполняет задание. В статье раскрываются основные правила решения примеров, содержащих в себе весь спектр математических вычислений, включая скобки. Порядок действий в математике 4 класс правила и примеры.

    Перед выполнением задания попросите своё чадо пронумеровать действия, которые он собирается выполнить. Если возникли затруднения – помогите.

    Некоторые правила, которые необходимо соблюдать при решении примеров без скобок:

    Если в задании необходимо выполнить ряд действий, нужно сначала выполнить деление или умножение, затем сложение. Все действия выполняются по ходу письма. В противном случае, результат решения будет не верным.

    Если в примере требуется выполнить сложение и вычитание, выполняем по порядку, слева направо.

    27-5+15=37 (при решении примера руководствуемся правилом. Сначала выполняем вычитание, затем – сложение).

    Научите ребёнка всегда планировать и нумеровать выполняемые действия.

    Ответы на каждое решённое действие записываются над примером. Так ребёнку гораздо легче будет ориентироваться в действиях.

    Рассмотрим ещё один вариант, где необходимо распределить действия по порядку:

    Как видим, при решении соблюдено правило, сначала ищем произведение, после — разность.

    Это простые примеры, при решении которых, необходима внимательность. Многие дети впадают в ступор при виде задания, в котором присутствует не только умножение и деление, но и скобки. У школьника, не знающего порядок выполнения действий, возникают вопросы, которые мешают выполнить задание.

    Как говорилось в правиле, сначала найдём произведение или частное, а потом всё остальное. Но тут же есть скобки! Как поступить в этом случае?

    Решение примеров со скобками

    Разберём конкретный пример:

  • При выполнении данного задания, сначала найдём значение выражения, заключённого в скобки.
  • Начать следует с умножения, далее – сложение.
  • После того, как выражение в скобках решено, приступаем к действиям вне их.
  • По правилам порядка действий, следующим шагом будет умножение.
  • Завершающим этапом станет вычитание.
  • Как видим на наглядном примере, все действия пронумерованы. Для закрепления темы предложите ребёнку решить самостоятельно несколько примеров:

    Порядок, по которому следует вычислять значение выражения уже расставлен. Ребёнку останется только выполнить непосредственно решение.

    Усложним задачу. Пусть ребёнок найдёт значение выражений самостоятельно.

    7*3-5*4+(20-19) 14+2*3-(13-9)
    17+2*5+(28-2) 5*3+15-(2-1*2)
    24-3*2-(56-4*3) 14+12-3*(21-7)

    Приучите ребёнка решать все задания в черновом варианте. В таком случае, у школьника будет возможность исправить не верное решение или помарки. В рабочей тетради исправления не допустимы. Выполняя самостоятельно задания, дети видят свои ошибки.

    Родители, в свою очередь, должны обратить внимание на ошибки, помочь ребёнку разобраться и исправить их. Не стоит нагружать мозг школьника большими объёмами заданий. Такими действиями вы отобьёте стремление ребёнка к знаниям. Во всём должно быть чувство меры.

    Делайте перерыв. Ребёнок должен отвлекаться и отдыхать от занятий. Главное помнить, что не все обладают математическим складом ума. Может из вашего ребёнка вырастет знаменитый философ.

    detskoerazvitie.info

    Урок по математике 2 класс Порядок действий в выражениях со скобками.

    Успейте воспользоваться скидками до 50% на курсы «Инфоурок»

    Цель: 1.

    2.

    3. Закрепить знание таблицы умножения и деления на 2 – 6, понятия делителя и

    4. Учить работать в парах с целью развития коммуникативных качеств.

    Оборудование * : + — (), геометрический материал.

    Раз, два – выше голова.

    Три, четыре – руки шире.

    Пять, шесть – всем присесть.

    Семь, восемь – лень отбросим.

    Но сначала придется узнать его название. Для этого нужно выполнить несколько заданий:

    6 + 6 + 6 … 6 * 4 6 * 4 + 6… 6 * 5 – 6 14 дм 5 см… 4 дм 5 см

    Пока мы вспоминали о порядке действий в выражениях, с замком происходили чудеса. Мы были только что у ворот, а теперь попали в коридор. Смотрите, дверь. А на ней замок. Откроем?

    1. Из числа 20 вычесть частное чисел 8 и 2.

    2. Разность чисел 20 и 8 разделить на 2.

    — Чем отличаются результаты?

    — Кто сможет назвать тему нашего урока?

    (на массажных ковриках)

    По дорожке, по дорожке

    Скачем мы на правой ножке,

    Скачем мы на левой ножке.

    По тропинке побежим,

    Наше предположение было полностью правильно7

    Где выполняются действия сначала, если в выражении есть скобки?

    Смотрите перед нами «живые примеры». Давайте «оживим» их.

    * : + — ().

    m – c * (a + d) + x

    k: b + (a – c) * t

    6. Работа в парах.

    Для их решения вам понадобиться геометрический материал.

    Учащиеся выполняют задания в парах. После выполнения проверка работы пар у доски.

    Что нового вы узнали?

    8. Домашнее задание.

    Тема: Порядок действий в выражениях со скобками.

    Цель: 1. Вывести правило порядка действий в выражениях со скобками, содержащих все

    4 арифметических действия,

    2. Формировать способность к практическому применению правила,

    4.Учить работать в парах с целью развития коммуникативных качеств.

    Оборудование : учебник, тетради, карточки со знаками действий * : + — (), геометрический материал.

    1 .Физминутка.

    Девять, десять – тихо сесть.

    2. Актуализация опорных знаний.

    Сегодня мы с вами отправляемся в очередное путешествие по стране Знаний городу математика. Нам предстоит посетить один дворец. Что-то я забыла его название. Но не будем расстраиваться, вы сами сможете мне подсказать его название. Пока я переживала, мы подошли к воротам дворца. Войдем?

    1. Сравните выражения:

    2. Расшифруй слово.

    3. Постановка проблемы. Открытие нового.

    Так как же называется дворец?

    А когда в математике мы говорим о порядке?

    Что вы уже знаете о порядке выполнения действий в выражениях?

    — Интересно, нам предлагают записать и решить выражения (учитель читает выражения, учащиеся записывают их и решают).

    20 – 8: 2

    (20 – 8) : 2

    Молодцы. А что интересного в этих выражениях?

    Посмотрите на выражения и их результаты.

    — Что общего в записи выражений?

    — Как вы думаете, почему получились разные результаты, ведь числа были одинаковые?

    Кто рискнет сформулировать правило выполнения действий в выражениях со скобками?

    Правильность этого ответа мы сможем проверить в другой комнате. Отправляемся туда.

    4. Физминутка.

    И по этой же дорожке

    До горы мы добежим.

    Стоп. Немножко отдохнем

    И опять пешком пойдем.

    5. Первичное закрепление изученного.

    Вот мы и пришли.

    Нам нужно решить еще два выражения, чтобы проверить правильность нашего предположения.

    6 * (33 – 25) 54: (6 + 3) 25 – 5 * (9 – 5) : 2

    Для проверки правильности предположения откроем учебники на стр. 33 и прочитаем правило.

    Как нужно выполнять действия после решения в скобках?

    На доске написаны буквенные выражения и лежат карточки со знаками действий * : + — (). Дети выходят к доске по одному, берут карточку с тем действием, которое нужно сделать сначала, потом выходит второй ученик и берет карточку со вторым действием и т. д.

    а + (а –в)

    а * (в +с) : d t

    m c * ( a + d ) + x

    k : b + ( a c ) * t

    (a – b)

    : t + d

    6. Работа в парах.

    Знание порядка действий необходимо не только для решения примеров, но и при решении задач мы тоже сталкиваемся с этим правилом. Сейчас вы в этом убедитесь работая в парах. Вам нужно будет решить задачи из № 3 стр. 33.

    7. Итог.

    По какому дворцу мы с вами сегодня путешествовали?

    Вам понравился урок?

    Как нужно выполнять действия в выражениях со скобками?

    • Можно ли оформить договор купли-продажи квартиры, купленной за материнский капитал? В настоящей момент каждой семье, в которой родился или которая усыновила второго ребенка, государство предоставляет возможность […]
    • Особенности бухгалтерского учета субсидий Государство стремится поддержать малое и среднее предпринимательство. Такая поддержка наиболее часто выражается в форме предоставления субсидий – безвозмездных выплат из […]
    • Работа вахтой в Москве — свежие вакансии прямых работодателей логистические компании; склады; Дополнительный плюс работы вахтовым методом заключается в том, что работник получает от компании проживание (в […]
    • Ходатайство об уменьшении размера исковых требований Один из видов уточнения иска — ходатайство об уменьшении размера исковых требований. Когда истец неправильно определил цену иска. Или ответчик частично исполнил […]
    • Как правильно париться в бане Банная процедура с парением — это целая наука. Основные правила парильщика: не торопиться, наибольшее удовольствие от бани — когда можно не спеша несколько раз зайти в парилку с […]
    • Школьная Энциклопедия Nav view search Login Form Законы Кеплера о движении планет Подробности Категория: Этапы развития астрономии Опубликовано 20.09.2012 13:44 Просмотров: 25396 «Он жил в эпоху, когда ещё не […]

    Конспект урока по математике на тему «Внетабличное умножение и деление» (3 класс)

    МИНИСТЕРСТВО ОБЩЕГО И ПРОФЕССИОНАЛЬНОГО ОБРАЗОВАНИЯ СВЕРДЛОВСКОЙ ОБЛАСТИ

    Государственное бюджетное образовательное учреждение

    Среднего профессионального образования Свердловской области

    «СВЕРДЛОВСКИЙ ОБЛАСТНОЙ ПЕДАГОГИЧЕСКИЙ КОЛЛЕДЖ»

    Конспект урока на тему:

    «Внетабличное умножение и деление»

    3 класс

    УМК «Школа России», М. И.Моро

    Выполнила:

    студентка 30Н группы

    Гринькина Е.К.

    Методист____________ Липатникова И. Г.

    Педагог-психолог___________ Манякова П.О.

    Учитель____________ Мугадова Л.Н.

    г. Екатеринбург

    2017

    Тема урока: «Внетабличное деление и умножение»

    Перспективная цель: формирование способности к использованию внетабличного умножения при решении примеров и задач.

    Актуальная цель: закрепить правила внетабличного умножения и деления чисел в пределах 100 в процессе решения примеров и задач.

    Задачи урока:

    Обучающая: моделировать способ и запись решения текстовой задачи; формировать способность к исправлению ошибок; закреплять навык применения правила порядка действий в выражениях со скобками и без скобок; решать примеры на умножение, деление и деление с остатком, делить двузначное число на двузначное методом подбора; отрабатывать навыки таблицы умножения.

    Развивающая: развивать внимательность, зрительную и слуховую память, математическую речь.

    Воспитательная: воспитывать уважительное отношение к мнению друг друга и интерес к математике.

    Технологическая карта

    Предмет, Тема, Класс, УМК

    Методы

    Тип, вид урока. Результат

    Планируемые результаты

    Виды учебной деятельности

    Система средств обучения

    Личностные

    Метапредметные

    Предметные

    Регулятивные

    Познавательные

    Коммуникативные

    Математика, «Внетабличное умножение и деление», 3 класс, УМК «Школа России»

    Словесный, наглядный, практический, игровой.

    Урок рефлексия

    понимание значения математики в повседневной жизни; умение объяснять причины успеха или неуспеха в своей учёбе.

    умение ставить цель; умение выполнять действие самоконтроля при вычислениях; осуществление самоконтроля.

    умение ставить цель на урок, находить пробелы в знаниях, уметь оформлять свои мысли в устной форме, умение ориентироваться в своей системе знаний.

    развитие коммуникативных способностей учащихся, стремление к более точному выражению собственного мнения; проявление инициативы и активности; умение работать в группе и умение вести диалог.

    Знание таблицы умножения, формулирование правил деления с остатком, двузначного числа на двузначное, знание компонентов деления и умножения, умение решать уравнения, развитие математической речи, умение строить математическую модель записи решения задачи.

    Самостоятельная работа, работа с презентацией, решение примеров и задач.

    Учебник, презентация, раздаточный материал.

    Конспект урока

    Прозвенел звонок на урок.

    -Ребята, пожалуйста, проверьте свою готовность на уроке. Итак, сначала тихо сядут девочки. А теперь тихо сядут мальчики.

    -Вспомните, чем мы занимались на прошлом уроке?

    -Верно, сегодня знания умножения и деления с остатком помогут выполнять действия при решении примеров и задач. Продолжим эту тему. Те задания, которые мы будем выполнять, постарайтесь выполнить без ошибок.

    Готовятся к уроку. Встают.

    Садятся девочки.

    Садятся мальчики.

    -Умножали и делили числа. Деление с остатком.

    Коммуникативные УУД: учатся слушать и понимать учителя.

    Личностные: формирование умения организовывать свою деятельность.

    Регулятивные: способность к саморегуляции.

    Актуализация учащихся и пробного учебного действия

    -Запишите сегодняшнюю дату и слова «Классная работа».

    Перед вами ряд чисел, но там пропущены три числа:

    …, 80, …, 60, 24, …, 48.

    Чтобы их найти, вам необходимо письменно решить следующие примеры:

    84:2, 76:38, 96:8.

    -Что интересного вы заметили?

    -Как правильно разделить двузначное число на двузначное?

    -Правильно, ребята. Скажите, какие числа у нас получились?

    -Эти числа заполняют пропуски, получился ряд: 42, 80, 2, 60, 24, 12, 48.

    -Устно расставьте числа в порядке возрастания.

    -Что можете сказать про эти числа?

    -Вспомните, какие есть компоненты умножения?

    -Назовите два множителя, произведение которых равно 12, 24, 42, 48?

    Перед вами три примера: 15 : 6; 56 :9; 60 :5.

    Вам необходимо письменно их решить.

    -Что вы можете сказать об этих примерах?

    -Что можно сказать об остатке?

    -Какое правило помогает понять, правильно ли выполнено деление с остатком?

    -Как проверить правильность выполнения действия?

    -Правильно, ребята. Сейчас решите следующие примеры:

    15:2 25:4

    57:9 44:8

    45:7 33:5

    59:6 27:5

    Каждый пример записываем в строчку и устно проговариваем, как решили.

    -Как узнать, сколько раз по 2 содержится в 15?

    Слушают учителя. Записывают дату.

    Решают примеры

    84:2=42

    76:38=2

    96:8=12

    -Пример 76:38 отличается, так как там мы делим двузначное число на двузначное.

    -Чтобы разделить, нужно найти частное способом подбора.

    Чтобы 76 разделить на 38, нужно подобрать такое число, которое при умножении на 38 даст 76.

    -42, 2, 12.

    — 2, 12, 24, 42, 48, 60, 80.

    — Они четные.

    -Первый множитель, второй множитель, произведение.

    — 12=6 *2; 24=8*3; 42= 6*7; 48=6*8.

    -15 разделить на 6, получится 2 и 3 в остатке.

    56 разделить на 9, получится 6 и 2 в остатке.

    60 разделить на 5, получится 11 и 5 в остатке.

    -Это деление с остатком.

    — Остаток меньше делителя.

    -Если остаток при делении меньше делителя, то деление с остатком выполнено правильно, если остаток при делении больше делимого, то мы допустили ошибку.

    -Частное умножить на делитель и прибавить остаток.

    Решают примеры в тетради.

    15 разделить на 2, получится 7 и 1 в остатке.

    Запись в тетради: 15:2=7 (ост.1)

    -В 15 содержится 7 раз по 2, и еще останется 1.

    57 разделить на 9, получится 6 и 3 в остатке.

    Запись в тетради: 57:9=6 (ост.3)

    45 разделить на 7, получится 6 и 3 в остатке.

    Запись в тетради: 45:7=6 (ост.3)

    59 разделить на 6, получится 9 и 5 в остатке.

    Запись в тетради: 59:6=9 (ост.5)

    25 разделить на 4, получится 6 и 1 в остатке.

    Запись в тетради: 25:4=6 (ост.1)

    44 разделить на 8, получится 5 и 4 в остатке.

    Запись в тетради: 44:8=5 (ост.4)

    33 разделить на 5, получится 6 и 3 в остатке.

    Запись в тетради: 33:5=6 (ост.3)

    27 разделить на 5, получится 5 и 2 в остатке.

    Запись в тетради: 27:5=5 (ост.2)

    Предметные: Развивают математическую речь, знание таблицы умножения, умение формулировать правила деления с остатком и двузначного числа на двузначное, знание компонентов деления и умножения.

    Регулятивные: составление последовательности действий.

    Коммуникативные: умение слушать и вступать в диалог.

    Локализация индивидуальных затруднений и действие целеполагания

    -После выполнения задания, сравните свою работу с образцом на доске.

    Образец:

    15:2=7 (ост.1) 25:4=6 (ост.1)

    57:9=6 (ост.3) 44:8=5 (ост.4)

    45:7=6 (ост.3) 33:5=6 (ост.3)

    59:6=9 (ост.5) 27:5=5 (ост.2)

    -Сейчас проверим, как вы выполнили задание. Какую ошибку вы могли допустить? Почему?

    -Проверьте, кто допустил ошибки, найдите их и исправьте. Какая цель нашего урока?

    -Правильно, а также понять, почему произошли ошибки.

    Сравнивают свою работу с образцом.

    Ответы детей:

    -Могли допустить ошибки при делении или умножении, при нахождении вычитания и сложения.

    -Найти ошибки и исправить их.

    Регулятивные: учатся ставить учебную задачу, поиск пробелов в знаниях.

    Познавательные: самостоятельное выделение и формулирование цели.

    Построение проекта коррекции выявленных затруднений и реализация его

    -Скажите, пожалуйста, как вы будете исправлять ошибку?

    -Какой алгоритм мы можем применить при делении с остатком?

    -Назовите компоненты действия деления с остатком.

    Дети рассказывают, какие ошибки они допустили и как будут их исправлять:

    -Неправильно выполнили деление, значит нужно вспомнить таблицу умножения и пересчитать еще раз.

    Вспомнить алгоритм деления с остатком.

    Алгоритм деления с остатком:

    1.Находим наибольшее число, которое можно разделить на делитель без остатка.

    2.Данное число делим на делитель. Это значение частного.

    3.Вычитаем разделившееся число из делителя – это остаток.

    -Делимое, делитель, частное, остаток.

    Дети, выполнившие все задания правильно, могут проанализировать свою работу с точки зрения оформления, аккуратности записи.

    Познавательные: формулирование пробелов в знаниях.

    Регулятивные: способность к саморегуляции, планирование действий.

    Обобщение причин затруднений во внешней речи

    Учитель помогает детям понять их ошибки.

    -Повторим компоненты деления и умножения.

    -Те ребята, которые не допустили ошибок, могут выполнить следующие задания

    1)№ 16 на странице 35.

    Выполните деление с остатком:

    36:7 44:5 60:8 80:12 44:18

    2)Решите уравнения и выполните проверку:

    12*х=60 х:14=6

    Дети обращают внимание на то задание, где допустили ошибки и обсуждают вместе с учителем.

    -Компоненты деления: делимое, делитель, частное.

    -Компоненты умножения: первый множитель, второй множитель, произведение.

    -Компоненты деления с остатком: делимое, делитель, частное, остаток.

    Дети, которые не допустили ошибок, выполняют эти задания в тетради и устно проговаривают решение:

    1) 36 разделить на 7, получится 5 и 1 в остатке.

    Запись в тетради: 36:7 =5(ост.1)

    44 разделить на 5, получится 8 и 4 в остатке.

    Запись в тетради: 44:5=8 (ост.4)

    60 разделить на 8, получится 7 и 4 в остатке.

    Запись в тетради: 60:8=7 (ост.4)

    80 разделить на 12, получится 6 и 8 в остатке.

    Запись в тетради: 80:12=6 (ост.8)

    44 разделить на 18, получится 2 и 8 в остатке.

    Запись в тетради: 44:18 =2 (ост. 8)

    2) Один ученик решает уравнение у доски и объясняет решение, все остальные записывают в тетради.

    Запись в тетради:

    12*Х=60 Х:14=6

    Х=60:12 Х=14*6

    Х=5 Х=84

    12*5=60 84:14=6

    60=60 6=6

    Регулятивные: внесение необходимых дополнений и корректив, выделение и осознание того, что уже усвоено, оценивание качества и уровня усвоения.

    Предметные: умение решать уравнения, знание таблицы умножения, умение делить с остатком.

    Самостоятельная работа с самопроверкой по эталону

    -Итак, я предлагаю вам выполнить следующие задания самостоятельно. После выполнения сравните работу с образцом.

    Задание №1

    Выполни деление с остатком, после, назовите делимое, делитель, частное и остаток.

    7:6; 30:8; 40:7; 41:7; 65:9; 46:9; 46:8; 39:10; 48:10.

    -Задание №2 решим устно.

    Надо упаковать 86 кубиков в коробки, по 10 штук в каждую. Сколько потребуется таких коробок? Сколько кубиков останется?

    -Подумайте, как можно записать решение задачи.

    -Раскрасьте следующий рисунок. Чтобы узнать цвет, вам необходимо правильно решить примеры.

    Выполняют письменно в тетради.

    Запись в тетради: 7:6=1 (ост.1)

    Делимое 7, делитель 6, частное 1 и остаток 1.

    Запись в тетради: 30:8=3 (ост.6)

    Делимое 30, делитель 8, частное 3 и остаток 6.

    Запись в тетради: 40:7=5 (ост.5)

    Делимое 40, делитель 7, частное 5 и остаток 5.

    Запись в тетради: 41:7=5 (ост.6)

    Делимое 41, делитель 7, частное 5 и остаток 6.

    Запись в тетради: 65:9=7 (ост.3)

    Делимое 65, делитель 9, частное 7 и остаток 3.

    Запись в тетради: 46:9=5 (ост.1)

    Делимое 46, делитель 9, частное 5 и остаток 1.

    Запись в тетради: 46:8=5 (ост.6)

    Делимое 46, делитель 8, частное 5 и остаток 6.

    Запись в тетради: 39:10=3 (ост.9)

    Делимое 39, делитель 10, частное 3 и остаток 9.

    Запись в тетради: 48:10=4 (ост.8)

    Делимое 48, делитель 10, частное 4 и остаток 8.

    Решают задание.

    -Чтобы узнать, сколько понадобится коробок и сколько останется кубиков, необходимо 86 разделить на 10, получится 8 коробок и 6 кубиков останется.

    Один из учеников делает запись на доске: 86:10=8 (ост.6)

    Решают примеры, раскрашивают рисунок.

    Регулятивные: составление последовательности действий.

    Регулятивные: способность к саморегуляции, структурирование знаний.

    Личностные: проявление интереса к математике.

    Предметные: знание таблицы умножения, умение решать задачи на деление с остатком, умение строить математическую модель записи решения задачи.

    Итог урока. Рефлексивно оценочные действия

    -Итак, ребята, мы выполнили все задания.

    -Чему научились на уроке?

    -Какой материал повторили?

    -Где и когда вы можете применить свои знания?

    Отвечают на вопросы, высказывают свое мнение.

    -Закрепили навыки умножения и деления. Повторили таблицу умножения, алгоритм деления двузначного числа на двузначное.

    Высказывают свое мнение.

    Коммуникативные: умение высказывать свою точку зрения и умение слушать одноклассников.

    Инструктаж по выполнению домашнего задания

    -Наш урок подходит к концу. Мы сегодня поработали очень хорошо.

    Запишите домашнее задание:

    Учебник стр.

    Записывают домашнее задание.

    Коммуникативные УУД: учатся слушать и понимать учителя.

    Проверочная работа по математике «Умножение и деление трёхзначных чисел» 3 класс

    Проверочная работа по теме:

    «Умножение и деление трёхзначных чисел»

     

    1 – вариант

     

    №1. Реши примеры, делая подробную запись в строчку:

    31 · 3=                                    39 :3=

    342 · 2=                                  84 : 6=

    123 · 4=                                  351 : 3=

     

    №2. Реши примеры, выполняя вычисления в столбик:

    32 · 3=          142 · 6=          69 : 3=            864 : 2=

    214 · 4=        73 · 3=            408 : 4=          459 :3=

     

    №3.  Реши систему неравенств, сделай чертёж и запиши ответ:

    х>6

      х<11

     

    №4. Реши неравенство, используя решение соответствующего уравнения. Сделай проверку:

    в + 53 > 87

     

    №5. Реши задачу. Запиши решение сложным выражением.

    В магазин привезли 8 ящиков груш по 32 кг в каждом. Сколько кг груш продали, если осталось 58 кг груш.

     

    №6**Заполни клетки квадрата числами так, чтобы он стал «магическим»:

     

    156

     

     

     

     

    164

     

     

     

     

    158

     

     

    152

     

     

     

     

     

     

    Проверочная работа по теме:

    «Умножение и деление трёхзначных чисел»

     

    2 – вариант

     

    №1. Реши примеры, делая подробную запись в строчку:

    21 · 3=                                    69 :3=

    243 · 2=                                  84 : 7=

    223 · 4=                                  357 : 3=

     

    №2. Реши примеры, выполняя вычисления в столбик:

    23 · 3=          162 · 6=          96 : 3=            684 : 2=

    114 · 4=        83 · 3=            606 : 6=          456 :3=

     

    №3.  Реши систему неравенств, сделай чертёж и запиши ответ:

    х>7

      х<12

     

    №4. Реши неравенство, используя решение соответствующего уравнения. Сделай проверку:

    в + 37 < 96

     

    №5. Реши задачу. Запиши решение сложным выражением.

    В магазин привезли 7 ящиков груш по 34 кг в каждом.

    Сколько кг груш продали, если осталось 63 кг груш.

     

    №6**Заполни клетки квадрата числами так, чтобы он стал «магическим»:

     

    156

     

     

     

     

    164

     

     

     

     

    158

     

     

    152

     

     

     

     

     

    Таблица умножения и игра, чтобы быстро выучить

    С лучшей бесплатной игрой таблица умножения учится очень быстро. Проверьте это сами!

    Учить таблицу умножения — игра

    Попробуйте нашу обучающую электронную игру. Используя её, вы уже завтра сможете решать математические задачи в классе у доски без ответов, не прибегая к табличке, чтобы умножить числа. Стоит только начать играть, и уже минут через 40 будет отличный результат. А для закрепления результата тренируйтесь несколько раз, не забывая о перерывах. В идеале – каждый день (сохраните страницу, чтобы не потерять). Игровая форма тренажера подходит как для мальчиков, так и для девочек.

    Таблица умножения – таблица, где строки и столбцы озаглавлены множителями (1, 2, 3, 4, 5…), а ячейки таблицы содержат их произведение. Применяется таблица для обучения умножению. Здесь есть игра и картинка для печати. Для скачивания игры с таблицей на компьютер, сохраните страницу (Ctrl+S). Также посмотрите таблицу деления.

    Смотрите ниже шпаргалки в полной форме.

    Распечатать таблицу умножения

    Умножение прямо на сайте (онлайн)

    *

    https://uchim. org/matematika/tablica-umnozheniya — uchim.org


    Таблица умножения (числа от 1 до 20)
     ×1234567891011121314151617181920
    11234567891011121314151617181920
    2246810121416182022242628303234363840
    33691215182124273033363942454851545760
    448121620242832364044485256606468727680
    55101520253035404550556065707580859095100
    66121824303642485460667278849096102108114120
    7714212835424956637077849198105112119126133140
    881624324048566472808896104112120128136144152160
    9918273645546372819099108117126135144153162171180
    10102030405060708090100110120130140150160170180190200
    11112233445566778899110121132143154165176187198209220
    121224364860728496108120132144156168180192204216228240
    1313263952657891104117130143156169182195208221234247260
    1414284256708498112126140154168182196210224238252266280
    15153045607590105120135150165180195210225240255270285300
    16163248648096112128144160176192208224240256272288304320
    171734516885102119136153170187204221238255272289306323340
    181836547290108126144162180198216234252270288306324342360
    191938577695114133152171190209228247266285304323342361380
    2020406080100120140160180200220240260280300320340360380400

    Как умножать числа столбиком (видео по математике)

    Чтобы потренироваться и быстро выучить, можно также попробовать умножать числа столбиком.

    Нужно распечатать таблицу умножения? Просто нажмите на ссылку печать таблицы умножения. Либо скопируйте картинку (первая таблица) в Ворд (Microsoft Office Word) и распечатайте с помощью сочетания клавиш Ctrl+P. Смотрите также таблицу квадратов.

    Всё для учебы » Математика в школе » Таблица умножения и игра, чтобы быстро выучить

    Порядок действий в Математике

    Основные операции в математике

    Основные операции, которые используют в математике — это сложение, вычитание, умножение и деление. Помимо этих операций есть ещё операции отношения, такие как равно (=), больше (>), меньше (<), больше или равно (≥), меньше или равно (≤), не равно (≠).

    Операции действия:

    • сложение (+)
    • вычитание (-)
    • умножение (*)
    • деление (:)

    Операции отношения:

    • равно (=)
    • больше (>)
    • меньше (<)
    • больше или равно (≥)
    • меньше или равно (≤)
    • не равно (≠)

    Сложение — операция, которая позволяет объединить два слагаемых.

    • Запись сложения: 5 + 1 = 6, где 5 и 1 — слагаемые, 6 — сумма.

    Вычитание — действие, обратное сложению.

    • Запись вычитания: 10 — 1 = 9, где 10 — уменьшаемое, 1 — вычитаемое, 9 — разность.

    Если разность 9, сложить с вычитаемым 1, то получится уменьшаемое 10. Операция сложения 9 + 1 = 10 является контрольной проверкой вычитания 10 — 1 = 9.

    Умножение — арифметическое действие в виде краткой записи суммы одинаковых слагаемых.

    • Запись: 3 * 4 = 12, где 3 — множимое, 4 — множитель, 12 — произведение.
    • 3 * 4 = 3 + 3 + 3 + 3

    В случае, если множимое и множитель поменять ролями, произведение остается одним и тем же. Например: 5 * 2 = 5 + 5 = 10.

    Поэтому и множитель, и множимое называют сомножителями.

    Деление — арифметическое действие обратное умножению.

    • Запись: 30 : 6 = 5 или 30/6 = 5, где 30 — делимое, 6 — делитель, 5 — частное.4 = 81 — возведение числа 3 в четвертую степень дает 81 (проверка извлечения корня).
    • 2√16 = 4 — корень второй степени называется — квадратным.

    При знаке квадратного корня показатель корня принято опускать: √16 = 4.

    3√8 = 2 — корень третьей степени называется — кубическим.

    Сложение и вычитание, умножение и деление, возведение в степень и извлечение корня попарно представляют обратные друг другу действия. Далее узнаем порядок выполнения арифметических действий.

     

    Порядок вычисления простых выражений

    Есть однозначное правило, которое определяет порядок выполнения действий в выражениях без скобок:

    • действия выполняются по порядку слева направо
    • сначала выполняется умножение и деление, а затем — сложение и вычитание.

    Из этого правила становится яснее, какое действие выполняется первым. Универсального ответа нет, нужно анализировать каждый пример и подбирать ход решения самостоятельно.

    Что первое, умножение или деление? — По порядку слева направо.

    Сначала умножение или сложение? — Умножаем, потом складываем.

    Порядок выполнения действий в математике (слева направо) можно объяснить тем, что в нашей культуре принято вести записи слева направо. А необходимость сначала умножить или разделить объясняется самой сутью этих операций.

    Рассмотрим порядок арифметических действий в примерах.

    Пример 1. Выполнить вычисление: 11- 2 + 5.

    Как решаем:

    В нашем выражении нет скобок, умножение и деление отсутствуют, поэтому выполняем все действия в указанном порядке. Сначала вычтем два из одиннадцати, затем прибавим к остатку пять и в итоге получим четырнадцать.

    Вот запись всего решения: 11- 2 + 5 = 9 + 5 = 14.

    Ответ: 14.

    Пример 2. В каком порядке выполнить вычисления в выражении: 10 : 2 * 7 : 5?

    Как рассуждаем:

    Чтобы не ошибиться, перечитаем правило для выражений без скобок. У нас есть только умножение и деление — значит сохраняем записанный порядок вычислений и считаем последовательно слева направо.

    Сначала выполняем деление десяти на два, результат умножаем на семь и получившееся в число делим на пять.

    Запись всего решения выглядит так: 10 : 2 * 7 : 5 = 5 * 7 : 5 = 35 : 5 = 7.

    Ответ: 7.

    Пока новые знания не стали привычными, чтобы не перепутать последовательность действий при вычислении значения выражения, удобно над знаками арифметический действий расставить цифры, которые соответствуют порядку их выполнения.

    Например, в такой последовательности можно решить пример по действиям:


    Действия первой и второй ступени

    В некоторых учебниках по математике можно встретить разделение арифметических действий на действия первой и второй ступени.

    • Действиями первой ступени называют сложение и вычитание, а умножение и деление — действиями второй ступени.

    С этими терминами правило определения порядка выполнения действий звучит так:

    Если выражение не содержит скобок, то по порядку слева направо сначала выполняются действия второй ступени (умножение и деление), затем — действия первой ступени (сложение и вычитание).


    Порядок вычислений в выражениях со скобками

    Иногда выражения могут содержать скобки, которые подсказывают порядок выполнения математических действий. В этом случае правило звучит так:

    Сначала выполнить действия в скобках, при этом также по порядку слева направо выполняется умножение и деление, затем — сложение и вычитание.

    Выражения в скобках рассматриваются как составные части исходного выражения. В них сохраняется уже известный нам порядок выполнения действий.

    Рассмотрим порядок выполнения действий на примерах со скобками.

    Пример 1. Вычислить: 10 + (8 — 2 * 3) * (12 — 4) : 2.

    Как правильно решить пример:

    Выражение содержит скобки, поэтому сначала выполним действия в выражениях, которые заключены в эти скобки.

    Начнем с первого 8 — 2 * 3. Что сначала, умножение или вычитание? Мы уже знаем правильный ответ: умножение, затем вычитание. Получается так:

    8 — 2 * 3 = 8 — 6 = 2.

    Переходим ко второму выражению в скобках 12 — 4. Здесь только одно действие – вычитание, выполняем: 12 — 4 = 8.

    Подставляем полученные значения в исходное выражение:

    10 + (8 — 2 * 3) * (12 — 4) : 2 = 10 + 2 * 8 : 2.

    Порядок действий: умножение, деление, и только потом — сложение. Получится:

    10 + 2 * 8 : 2 = 10 + 16 : 2 = 10 + 8 = 18.

    На этом все действия выполнены.

    Ответ: 10 + (8 — 2 * 3) * (12 — 4) : 2 = 18.

    Можно встретить выражения, которые содержат скобки в скобках. Для их решения, нужно последовательно применять правило выполнения действий в выражениях со скобками. Удобнее всего начинать выполнение действий с внутренних скобок и продвигаться к внешним. Покажем на примере.

    Пример 2. Выполнить действия в выражении: 9 + (5 + 1 + 4 * (2 + 3)).

    Как решаем:

    Перед нами выражение со скобками. Это значит, что выполнение действий нужно начать с выражения в скобках, то есть, с 5 + 1 + 4 * (2 + 3). Но! Это выражение также содержит скобки, поэтому начнем сначала с действий в них:

    2 + 3 = 5.

    Подставим найденное значение: 5 + 1 + 4 * 5. В этом выражении сначала выполняем умножение, затем — сложение:

    5 + 1 + 4 * 5 = 5 + 1 + 20 = 26.

    Исходное значение, после подстановки примет вид 9 + 24, и остается лишь выполнить сложение: 9 + 26 = 35.

    Ответ: 4 + (3 + 1 + 4 * (2 + 3)) = 35.

     

    Порядок вычисления в выражениях со степенями, корнями, логарифмами и иными функциями

    Если в выражение входят степени, корни, логарифмы, синус, косинус, тангенс и котангенс, а также другие функции — их значения нужно вычислить до выполнения остальных действий. При этом важно учитывать правила из предыдущих пунктов, которые задают очередность действий в математике.

    Другими словами, перечисленные функции по степени важности можно приравнивать к выражению в скобках.

    И, как всегда, рассмотрим, как это работает на примере.

    Пример 1. Вычислить (4 + 1) * 3 + 62 : 3 — 7.

    Как решаем:

    В этом выражении есть степень 62. И нам нужно найти ее значение до выполнения остальных действий. Выполним возведение в степень: 62 = 36.

    Подставляем полученное значение в исходное выражение:

    (4 + 1) * 3 + 36 : 3 — 7.

    Дальше нам уже все знакомо: выполняем действия в скобках, далее по порядку слева направо выполняем сначала умножение, деление, а затем — сложение и вычитание. Ход решения выглядит так:

    (4 + 1) * 3 + 36 : 3 — 7 = 3 * 3 + 36 : 3 — 7 = 9 + 12 — 7 = 14.

    Ответ: (3 + 1) * 2 + 62 : 3 — 7 = 14.

    У нас есть статья «знаки больше, меньше или равно», она может быть полезной для тебя!

    3 класс, часть 1 – 2 Консультация 3. Уроки 1 – 13.

    3 класс, часть 1 – 2

    Консультация 3. Уроки 1 – 13.

    На уроках 1 – 5 систематизируются знания учащихся о единицах измерения длины и массы, вводятся новые единицы измерения массы: грамм, центнер, тонна, закрепляются соотношения между единицами измерения длины, массы, умение выражать значения величин в разных единицах измерения. Также повторяются и закрепляются нумерация и действия с многозначными числами, решение текстовых задач, уравнений, примеров на порядок действий, умножение чисел в столбик, измерение отрезков и построение отрезков данной длины, понятие объема прямоугольного параллелепипеда, отрабатываются вычислительные навыки.

    На уроке 1 воспроизводится таблица, устанавливающая соотношение между единицами длины, с которой учащиеся уже встречались раньше:

    Теперь область применения этой таблицы существенно расширяется. В 1, стр. 95 проговариваются все возможные соотношения между этими единицами. Например, устанавливается, что 1 км = 1000 м = 10 000 дм = 100 000 см = 1 000 000 мм и т. д. При этом надо вспомнить правило: при переходе к меньшим меркам выполняется умножение, а при переходе к большим меркам – деление. Соответствующие коэффициенты перехода (числа, на которые надо умножать или делить при переходе от одной единицы измерения к другой) записаны под дугами.

    В 2–4, стр. 95 учащиеся используют установленные соотношения и аналогию с десятичной системой записи чисел для перевода длин из одних единиц измерения в другие. Решение примеров записывается в тетради в клетку и проговаривается вслух. Способ обоснования может быть различным – на основе установленного правила либо на основе аналогии с десятичной системой записи чисел, например:

    а) 7 м = 700 см, так как в 1 метре 100 сантиметров, а 100 · 7 = 700,

    или

    7 м = 700 см, так как 7 метров – это 7 сотен сантиметров;

    б) 16 000 мм = 1600 см, так как в 1 сантиметре 10 миллиметров, а

    16 000 : 10 = 1600,

    или

    16 000 мм = 1600 см, так как в 16 000 содержится 1600 десятков;

    в) 12 км 50 м = 12 050 м, так как в 1 километре 1000 метров, значит,

    в 12 км – 12 000 м, да еще 50 м, всего получится 12 050 метров,

    или

    12 км 50 м = 12 050 м, так как 12 км 50 м – это 12 тысяч 50 метров.

    Основным способом является первый, так как он универсальный и используется, например, и при преобразовании единиц времени, где соотношения между единицами не являются десятичными. Однако акцент на аналогию системы мер длины и массы с десятичной системой записи чисел не только поможет закрепить знание нумерации, но и покажет связь изучения чисел с практическими задачами. Каждый из учеников может выбрать тот способ обоснования, который ему удобен, а в классе должны звучать оба способа.

    Перед выполнением заданий 5–6, стр. 96 надо повторить с учащимися правило о том, что величины можно сравнивать, складывать и вычитать только тогда, когда они выражены в одних и тех же единицах измерения. Поэтому для сравнения, сложения и вычитания величин в этих заданиях надо их сначала выразить в одинаковых мерках.

    На уроке 2 в 1–2, стр. 98 учащиеся решают практические задачи, связанные с построением отрезков и измерением их длин. В 1 они устанавливают, что если точки A, B и C лежат на одной прямой, то длина AC равна сумме длин AB и BC, а если нет, то длина AC меньше суммы длин AB и . Другими словами, прямая линия, соединяющая две точки A и C, короче ломаной ABC. В 2 они строят планы земельных участков треугольной и четырехугольной формы и вычисляют их периметры. Таким образом, их внимание еще раз обращается на то, что числа возникли для решения практических задач, поэтому естественно, что соотношения между единицами измерения величин аналогичны принципу нумерации. Эта аналогия еще раз подчеркивается в 3, стр. 98. В заданиях 4–5, стр. 98 рассматриваются более сложные случаи перевода единиц длины.

    На уроках 3–4 аналогичным образом рассматриваются единицы массы и соотношения между ними:

    Правило перевода единиц и способы перевода остаются прежними, изменяются лишь названия единиц и переводные коэффициенты. Кроме того, рассматриваются виды гирь, которые обычно используются при взвешивании, и способы уравновешивания предметов на чашечных весах.

    Хотим отметить, что при выполнении 10, стр. 99 следует обратить внимание на некоторые моменты. К настоящему времени дети уже знают, что одни и те же математические выражения могут описывать разнообразные жизненные ситуации. Так, выражение 2 + 3 может быть суммой игрушек, ручек, тракторов и еще чего угодно, в том числе «шклидулок». И от того, что мы не знаем, что такое «шклидулка», суть вычислений не изменится – мы все равно получим в ответе 5.

    В задаче предлагается вымышленная ситуация – о шклидулках и бримазятах. Математическая структура задачи не представляет для учеников труда, но здесь они должны суметь перенести ее на абстрактное для них содержание и провести рассуждения во всей полноте.

    – Чтобы ответить на первый вопрос задачи, можно сложить шклидулки, которые нашли бримазище и бримазенок. (Ищем целое.) Для этого сначала из 96 вычтем 64 и узнаем, сколько шклидулок нашел бримазенок. Чтобы узнать, во сколько раз больше шклидулок нашел бримазище, чем бримазенок, надо первое число разделить на второе.

    1) 96 – 64 = 32 (ш.) – нашел бримазенок.

    2) 96 + 32 = 128 (ш.).

    3) 96 : 32 = 3 (раза).

    Ответ: вместе они нашли 128 шклидулок, бримазище – в 3 раза больше бримазенка.

    При выполнении 12, стр. 103 следует рассуждать так:

    Р – 70 Г – 200 С – 40

    И – 80 К – 5400 Б – 400

    П – 50 О – 4800 Н – 100

    СПРИНГБОК. Один из интереснейших видов газелей, обитающий в Южной Африке. Верхняя сторона тела – желто‑коричневая, нижняя сторона – белая, на границе проходит контрастная буровато‑черная полоса. Но самая замечательная особенность спрингбока – обширная продольная кожная складка на спине. Когда животное спокойно, складку не видно. Но, почувствовав опасность, спрингбок начинает подпрыгивать на месте, отталкиваясь одновременно всеми ногами, без видимых усилий, как резиновый мяч.

    Прыжки спрингбока колоссальны: до 2 м в высоту. При этом края кожной складки расходятся, и выстилающий ее белый мех начинает ослепительно сверкать. Для всех обитателей саванны прыжки спрингбока служат сигналом опасности.

    Спрингбок знаменит своими странствиями. К сожалению, говорить о них приходится лишь в прошедшем времени: они прекратились вместе с резким уменьшением численности спрингбока. Во время последнего крупного переселения спрингбоков в 1896 году животные плотной массой покрывали участок шириной около 25 км, а длина колонны составляла 220 км!

    Во второй части учебника закрепляются нумерация, сложение и вычитание многозначных чисел, вводится умножение и деление многозначного числа на однозначное, рассматриваются некоторые преобразования на плоскости (параллельный перенос, симметрия), меры времени и календарь, на основе некоторых логических понятий (высказывание, истинное и ложное высказывание) уточняется понятие уравнения и рассматриваются новые их виды. Учащиеся знакомятся с понятиями переменной и выражения с переменной, учатся находить значения выражений с переменной, строить формулы зависимостей между величинами.

    На уроках 6 – 9 у учащихся формируется умение умножать многозначные числа на однозначные и умножать круглые числа в случаях, сводящихся к умножению на однозначное число, учатся решать задачи на нахождение значений величин по их сумме и разности. Ученики повторяют и закрепляют нумерацию, сложение и вычитание многозначных чисел, решение текстовых задач, решение уравнений с комментированием по компонентам действий, сравнение выражений, действия с единицами длины и массы.

    Простейшие случаи умножения многозначного числа на однозначное (27 · 5, 140 · 3 и т. д.) и их запись в столбик уже встречались учащимся. На данном этапе обучения они должны распространить известный им способ умножения в столбик на общий случай умножения многозначного числа на однозначное, и отработать его для сложных случаев. Работа ведется, как обычно, деятельностным методом.

    На уроке 6 на этапе актуализации знаний с учащимися нужно вспомнить распределительное свойство умножения. Для этого можно рассмотреть с ними различные способы нахождения площади прямоугольников для случаев, когда прямоугольник разбит на 2 части и на 3 части:

    По данным рисункам ставятся вопросы:

    1) Чем похожи и чем отличаются эти задачи? (В первой задаче прямоугольник разбит на две части, а во второй – на три. )

    2) Как называется первое равенство? (Правило умножения суммы на число, или распределительное свойство умножения.)

    3) Можно ли распространить это правило на сумму трех слагаемых? (Из второго равенства следует, что да.)

    4) Можно ли его распространить на сумму большего числа слагаемых? (Да, ведь прямоугольник можно разбить на большее число частей.)

    Чтобы поставить проблему, учащимся можно сначала предложить решить в тетрадях в клетку следующие примеры и выявить в них закономерности:

    Ученики могут заметить, что:

    1) все примеры – на умножение;

    2) первый множитель увеличивается, а второй не изменяется;

    3) с увеличением первого множителя произведение увеличивается;

    4) если первый множитель увеличивается в 10 раз, то и все произведение

    увеличивается в 10 раз.

    Затем учитель предлагает, воспользоваться тем же вычислительным приемом и решить пример

    При решении примера, вероятно, возникнет затруднение: могут получиться разные ответы, кто‑то из детей не решит его и т. д. Возникшая проблемная ситуация и мотивирует поиск нового способа действий.

    В случае, если с последним примером справятся все обучающиеся, можно попросить их обосновать решение. Главное – дети должны заметить, что для решения данного примера используется другой вычислительный прием. Этот признак отличия они должны проговорить вслух: в первых четырех примерах требуется умножить двузначное число на однозначное, а в последнем примере – трехзначное на однозначное.

    После этого цель урока может быть сформулирована следующим образом: установить, как умножается любое многозначное число на однозначное. Если последний пример выполнят все ученики, то цель урока мотивируется необходимостью обосновать правомерность используемого приема.

    Этап «открытия» нового знания начинается с выбора метода рассуждений. Рассмотренная в начале урока задача о вычислении площадей прямоугольников должна помочь учащимся вспомнить, что алгоритм умножения двузначного числа на однозначное был установлен на основе правила умножения суммы на число (распределительного свойства умножения), и сориентироваться на это свойство.

    В 1, стр. 1 еще раз проговаривается формулировка правила умножения суммы на число и возможность его распространения на любое число слагаемых. Затем в 2 (а), стр. 1 данное число 576 разбивается на удобные слагаемые 500 + 70 + 6 и на основе этого правила выполняются преобразования:

    Очевидно, что такая запись является слишком громоздкой, неудобной, – это учащиеся скажут сразу. Тогда ставится задача найти более короткий способ записи по аналогии с умножением на двузначное число. Если самостоятельно ученики затруднятся это сделать, можно предложить им проанализировать слагаемые суммы по рисунку 2 (б), стр. 1. Дети должны заметить, что при вычислении суммы сначала подсчитывается число единиц, затем число десятков и число сотен (нули при сложении результата не изменяют). И поскольку все эти числа всегда являются двузначными (значения табличных произведений), то удобнее число единиц следующего разряда, которое «запоминается», писать вверху над соответствующим разрядом первого множителя, как при умножении двузначных чисел. Подвести учащихся к этому выводу можно следующей последовательностью вопросов:

    1) Как получили слагаемые суммы? (6 единиц умножили на 9, потом 7 десятков умножили на 9, а потом 5 сотен умножили на 9.)

    2) Всегда ли во втором слагаемом на конце будет нуль? Почему? (Всегда, так как считаем число десятков.)

    3) Всегда ли в третьем слагаемом на конце 2 нуля? Почему? (Всегда, так как считаем число сотен.)

    4) Почему во втором столбике нули зачеркнуты? (Они не изменяют значение суммы.)

    5) Может ли число единиц, десятков или сотен «заходить» не на один следующий разряд, а на 2 или 3 разряда? (Нет, перемножаем однозначные числа, поэтому в произведении не может быть больше двух знаков.)

    6) Сравните запись умножения во втором и третьем столбике – какая из записей удобнее? (В третьем столбике.)

    7) Догадайтесь, как она получается из предыдущей? (Сначала умножаем единицы: 6 · 9 = 54, 4 единицы пишем, а 5 десятков запоминаем – записываем над числом десятков первого множителя. Потом умножаем десятки: 7 · 9 = 63, 63 + 5 = 68, 8 десятков пишем, а 6 сотен запоминаем. А потом умножаем сотни: 5 · 9 = 45, 45 + 6 = 51, записываем 51 сотню. – «Открытие».)

    Пишу: множитель 9 под разрядом единиц множителя 576.

    Умножаю единицы: 6 · 9 = 54 ед., пишу 4 в разряде единиц,

    а 5 д. запоминаю.

    Умножаю десятки: 7 · 9 = 63 д., 63 + 5 = 68 д., пишу 8 в разряде

    десятков, а 6 с. запоминаю.

    Умножаю сотни: 5 · 9 = 45 с., 45 + 6 = 51 с., пишу 1 в разряде

    сотен, а 5 – в разряде тысяч.

    Ответ: 5184.

    В завершение учитель спрашивает у детей, изменятся ли рассуждения при умножении на однозначное число четырехзначного, пятизначного, шестизначного и т. д. числа. Как правило, дети легко распространяют полученный вывод на любое многозначное число. Тогда в тетради в клетку надо записать, решить и прокомментировать (с возможной помощью учителя) более сложный случай умножения, например, 5 · 20 156. Внимание детей обращается на порядок множителей и на то, что в данном случае также удобно писать однозначный множитель под разрядом единиц многозначного множителя.

    Если у учащихся все же возникнет сомнение в правомерности распространения полученного вывода на случай умножения любого многозначного числа на однозначное, то можно рассмотреть аналогичным образом умножение четырехзначного числа на однозначное или предложить учащимся сделать это дома самостоятельно.

    Примеры для этапа первичного закрепления подбираются в зависимости от уровня подготовленности класса. Можно, например, решить с подробным комментированием в громкой речи 3 (а), стр. 1, а для этапа самоконтроля использовать 3 (б), стр. 1. После выполнения самостоятельной работы ученики сопоставляют свое решение с образцом, предъявленным учителем, и убеждаются в том, что новый вычислительный прием ими освоен. Напомним, что при изучении нового материала первостепенное значение имеет создание ситуации успеха для каждого ребенка. Возможные ошибки должны здесь же исправляться, а материалы дорабатываться индивидуально, пока остальные учащиеся класса решают задачи на повторение.

    На этапе повторения новое знание включается в систему знаний, а также решаются задания, обеспечивающие непрерывность развития содержательно‑методических линий курса. Так, на рассматриваемом уроке умножение многозначного числа на однозначное встречается при решении текстовых задач 4–5, стр. 2, в уравнении 6, стр. 2 и при работе с буквенными выражениями в 7, стр. 2. Далее в задании 8, стр. 2 повторяется правило порядка действий в выражениях и отрабатываются вычислительные навыки. В 9, стр. 2 повторяются действия с многозначными числами, в 10–11, стр. 2 – понятия равенства и пересечения множеств, которые связываются с рисованием геометрических фигур и перебором вариантов, а в 12, стр. 2 предлагается логическая задача. Учитель на уроке введения нового знания выбирает для оставшихся 5–10 минут урока из этих заданий те, в которых учащиеся его класса испытывают больше затруднений.

    Сделать этот выбор более осознанным и обоснованным позволяют «Электронные приложения к учебникам».

    С другой стороны, методическим приемом, который позволяет существенно увеличить число решенных в классе примеров без перегрузки детей, является решение задач по выбору учащихся. Так, например, на данном уроке учитель может предложить учащимся на этапе повторения решить по выбору одно из заданий 5–9, стр. 2. Учащиеся в течение 3–4 минут решают по одному выбранному ими заданию, а затем проговаривают их решение в течение следующих 5 минут. Таким образом, все задания воспроизведены в памяти детей, т. е. цель повторения достигнута. При этом в классе создается атмосфера психологической комфортности, так как каждый ребенок решает задание, которое он выбрал сам, а значит, то, которое ему больше понравилось. Задачи по выбору можно предлагать и для домашней работы.

    При подведении итога урока учитель обсуждает с учениками вопросы:

    – Что нового узнали? (Научились умножать любое многозначное число на однозначное. )

    – Какое математическое свойство для этого использовали? (Распределительное свойство умножения.)

    – Кто уже чувствует себя уверенно в решении новых примеров?

    – Что повторили? Что больше всего понравилось?

    – Кто сегодня нам помогал на уроке?

    – Как оцениваете свою работу?

    Для домашней работы можно предложить учащимся придумать и решить свой пример на умножение многозначного числа на однозначное, решить задачу 4, стр. 2 и по желанию – одно из заданий 10–12, стр. 2. Таким образом, обязательное задание не займет у обучающихся больше 10–15 мин самостоятельной работы. При таком подходе исключена перегрузка детей, каждому из них обеспечивается возможность успешного усвоения необходимого минимума, и в то же время каждому предоставляется возможность обучения на высоком уровне за счет активного включения в деятельность на уроке и решения дополнительных развивающих заданий.

    На уроках 7–8 рассматриваются более сложные случаи умножения многозначного числа на однозначное и случаи умножения круглых чисел, сводящиеся к ним. Так, в 1, стр. 6 учащиеся распространяют на множество многозначных чисел изученное ранее правило: чтобы умножить круглые числа, надо выполнить умножение, не глядя на нули, а потом к полученному произведению приписать столько нулей, сколько в обоих множителях вместе. На основании этого правила при записи умножения круглых чисел в столбик для удобства вычислений нули мысленно отбрасываются и полученное однозначное число записывается в разряде единиц многозначного множителя:

    На последующих уроках умножение многозначного числа на однозначное отрабатывается в основном в процессе выполнения проверки примеров на деление.

    На уроке 8 рассматривается новый тип задач – задачи на нахождение величин по их сумме и разности. На основе предметных действий с моделями полосками ученики догадываются, что при вычитании из суммы двух чисел их разности получается удвоенное меньшее число, а при сложении суммы и разности – удвоенное большее число. Поэтому решить задачу, например, 1, стр. 8 можно двумя способами:

    Для этапа первичного закрепления предназначены задания 3–4, стр. 8–9, а для этапа самостоятельной работы с самопроверкой в классе – 2, стр. 8. Дома можно предложить им придумать и решить свои задачи на нахождение величин по их сумме и разности.

    На всех данных и последующих уроках особое внимание уделяется комментированию решения уравнений по компонентам действий ( 6, стр. 2; 6, стр. 4; 6, стр. 9; 7, стр. 18; 5, стр. 20; 4, стр. 25 и т. д.). Это связано с подготовкой детей к изучению темы «Уравнения» на уроке 27 данной части учебника. К этому времени обучающиеся должны не только уметь на автоматизированном уровне верно находить неизвестные компоненты действий, но и комментировать решение по образцу, приведенному на стр. 77 учебника.

    На уроках 9 – 12 формируется умение делить многозначные числа на однозначные и делить круглые числа, сводящиеся к делению на однозначное число, умение делать проверку деления умножением, а также повторяются и закрепляются нумерация, сложение и вычитание многозначных чисел, умножение многозначного числа на однозначное, решение текстовых задач. Учащиеся решают уравнения с комментированием по компонентам действий, повторяют понятие периметра треугольника, понятие числового луча, действия с единицами длины и массы, читают и записывают выражения.

    При изучении внетабличного деления в пределах 100 учащиеся знакомились с правилом деления суммы на число. Сейчас это правило используется для построения алгоритма деления многозначного числа на однозначное. В итоге обсуждения учащиеся должны выявить и осмыслить основную идею, основной принцип деления многозначных чисел: сначала делится более крупная счетная единица, затем остаток дробится и делится следующая по величине счетная единица и так далее до конца. Новый материал вводится в обучение деятельностным методом.

    На уроке 9 на этапе актуализации знаний с учащимися нужно вспомнить взаимосвязь между умножением и делением (a : b = c b · c = a, b 0), алгоритм деления с остатком и правило деления суммы на число, распространив его, как и в предыдущем случае, на сумму трех и более слагаемых.

    На этапе постановки проблемы детям можно предложить в течение 2–3 минут в тетрадях в клетку самостоятельно решить примеры «по частям», т. е. используя правило деления суммы на число, и выявить в них закономерности:

    Учащиеся могут заметить, что:

    1) все примеры – на деление;

    2) делимое увеличивается, а делитель не изменяется;

    3) с увеличением делимого частное увеличивается;

    4) если делимое увеличивается в 10 раз, то и частное увеличивается в 10 раз.

    При решении последнего примера обычно возникает затруднение, которое мотивирует поиск нового способа действий (если и последний пример выполнят все ученики, можно попросить их найти лишний пример).

    Далее учитель подводит учащихся к выявлению существенного для данного урока признака отличия последнего примера от предыдущих: первые четыре примера сводятся к делению двузначного числа на однозначное, а в последнем примере – деление трехзначного числа на однозначное. Этот признак отличия учащиеся должны проговорить вслух.

    Таким образом, ставится цель урока установить, как делится многозначное число на однозначное. (Если затруднений в решении последнего примера у обучающихся не возникнет, слово установить заменяется словом обосновать – ведь подобные примеры в классе ранее не рассматривались.)

    На этапе «открытия» нового знания детям вначале предоставляется возможность выбрать метод рассуждений. Задания, рассмотренные в начале урока, должны сориентировать их на выбор правила деления суммы на число, распространенного на случай нескольких слагаемых. Для подбора слагаемых для вычисления частного 536 : 4 можно использовать графическую модель. Учитель рисует ее на доске, а учащиеся – в тетради:

    Рассматривая ее, ученики должны догадаться, что для нахождения частного вначале надо разделить сотни (коробки), затем оставшуюся сотню перевести в десятки и делить все имеющиеся десятки (пачки) и, наконец, оставшийся десяток раздробить в единицы (штуки) и делить единицы. В менее подготовленных классах поиск решения целесообразно сопровождать не только графическим моделированием, но и предметным – работой с конкретными коробками, пачками и единицами предметов.

    Получившиеся группы обводятся овалами – это «удобные слагаемые»:

    Из приведенных рассуждений следует, что каждый получил 1 сотню, 3 десятка и 4 штуки, или 134 штуки предметов. На математическом языке проведенные рассуждения можно записать так:

    536 : 4 = (400 + 120 + 16) : 4 = 400 : 4 + 120 : 4 + 16 : 4 = 100 + 30 + 4 = 134.

    Эта цепочка преобразований записывается в тетрадь, и еще раз проговаривается полученный вывод: чтобы разделить многозначное число на однозначное, можно делимое разбить на сумму «удобных» слагаемых и делить «по частям», то есть по правилу деления суммы на число.

    Применение этого способа действий весьма ограничено, но проведенные рассуждения помогут учащимся в дальнейшем осмыслить общий принцип деления многозначных чисел. Для перехода к делению углом надо показать им неудобство построенного способа действий, предложив, например, найти частное 11 768 : 4.

    Понятно, что попытки найти «удобные» слагаемые вряд ли закончатся успехом, и тогда можно попросить детей еще раз вернуться к рисунку:

    – Рассмотрите, с каких единиц мы начинали деление – с мелких или с крупных? (С крупных.)

    – Конечно, ведь удобнее сначала раздать более крупные счетные единицы – коробки. Но вот у нас 1 коробка осталась, что нам пришлось сделать? (Достать пачки и делить уже пачки.)

    – Правильно, нам пришлось раздробить сотни в десятки. А когда и десятки у нас закончились, что мы сделали? (Стали делить единицы.)

    – Кто теперь догадается, как можно делить любое многозначное число, не подбирая слагаемые? (Делить сначала самые крупные счетные единицы, затем остаток дробить и делить более мелкие единицы.)

    На доске в процессе беседы учитель кратко записывает суть выполняемых преобразований:

    1) 5 с. : 4 = 1 с. (ост. 1 с.)

    2) 13 д. : 4 = 3 д. (ост. 1 д.)

    3) 16 ед. : 4 = 4 ед. Итак, 536 : 4 = 134.

    Аналогично записывается решение примера 11 768 : 4, предложенного учителем:

    1) 11 т. : 4 = 2 т. (ост. 3 т.)

    2) 37 с. : 4 = 9 с. (ост. 1 с.)

    3) 16 д. : 4 = 4 д.

    4) 8 ед. : 4 = 2 ед. Итак, 11 768 : 4 = 2942.

    Таким образом, поставленная проблема решена: найден общий способ деления многозначного числа на однозначное. Он заключается в делении с остатком возможно более крупных счетных единиц и последовательном переходе к делению более мелких счетных единиц. Однако остается проблема записи деления. На вопрос учителя: «Удобная ли запись деления?» – ответ всегда одинаковый: неудобная, громоздкая. Тогда можно предложить учащимся попробовать придумать свою запись, более короткую и удобную. Для этой цели лучше использовать первый пример – 536 : 4.

    Только после того как дети предложат свои версии, следует показать им «свернутый» способ записи приведенных рассуждений – уголком, и прокомментировать его:

    Проверку деления удобно делать умножением на основании взаимосвязи:

    Так, для проверки выполненного деления можно число 2942 умножить на 4.

    Учитель обращает внимание учащихся на то, что при комментировании примеров надо вначале указать первое неполное делимое, потом определить число цифр в частном, а затем рассказать, как находятся цифры в каждом разряде частного. При этом надо постоянно помнить о том, что на каждом шаге мы фактически выполняем деление с остатком, и поэтому получаемые остатки должны быть меньше делителя. Проверку решения удобно делать умножением.

    Алгоритм письменного деления фиксируется с помощью блок-схемы:

    Проблема разрешена.

    Для проведения этапа первичного закрепления можно использовать задания 3–6, стр. 11–12, которые решаются с проговариванием в громкой речи. В 3 учащиеся находят частное всеми тремя рассмотренными способами. В 4 внимание детей еще раз фиксируется на том, что остаток от деления всегда должен быть меньше делителя, проговариваются основные этапы деления многозначного числа на однозначное, выделенные в рамке на стр. 11. Примеры 5–6 записываются в тетради в клетку и решаются по выбору. Здесь возможно комментирование в паре, в группе, создание игровых ситуаций. Достаточно, если каждый ребенок решит 2–3 примера. Параллельно проговаривается способ проверки деления умножением, зависимость между компонентами деления.

    Задание 2, стр. 10 целесообразно использовать на этапе самостоятельной работы с самопроверкой в классе. Оно менее сложное, чем примеры, решенные на предыдущем этапе урока, и содержит наглядную опору, которая поможет обучающимся лучше представить каждый этап деления.

    На этапе повторения по выбору можно решить задания 7 (а), стр. 12 и 9 (а), стр. 12.

    При подведении итога урока обсуждаются вопросы:

    – Что нового узнали? (Научились делить многозначное число на однозначное, записывать деление «углом».)

    – Какой прием используется для устного деления? (Деление «по частям».)

    – С каких единиц начинаем письменное деление? (С самых крупных. ) А потом? (Делим по очереди более мелкие единицы.)

    – Кто сегодня нам хорошо помогал?

    – Кто доволен своей работой?

    – Что повторили? Что больше всего понравилось?

    В домашней работе можно предложить учащимся самостоятельно составить и решить пример на деление трехзначного числа на однозначное, построить его графическую модель и выполнить деление тремя способами по аналогии с тем, как это сделано в учебнике. Кроме того, решить по собственному выбору одно из заданий 7 (б), 9 (б), стр. 12. В качестве дополнительного задания, которое выполняется по желанию, – одно из заданий 8, 10, стр. 12.

    На последующих уроках рассматриваются более сложные случаи деления: делимое содержит большее число цифр (урок 10), в частном получаются нули в середине и на конце (уроки 11–13).

    Чтобы переварить знания, надо поглощать их с аппетитом.

    (А. Франц)

    Желаем Вам удачи и творческих успехов!

    Мы вместе, значит, у нас все получится!

    Соединение фактов умножения и деления — элементарная математика

    Назначение

    Для идентификации и предоставления связанных фактов умножения и деления

    Материалы

    Нет

    Обзор

    Дайте учащимся два множителя (например, 3 x 7), попросите учащихся дать произведение (21), а затем попросите одного учащегося указать связанный факт деления (например, 21 ÷ 7 = 3). Чтобы учащиеся были внимательны, укажите факторы и попросите учащихся отреагировать на продукт, прежде чем выбрать учащегося, чтобы сообщить соответствующий факт. Чтобы не терять темп, вы можете случайным образом выбирать учеников, используя колоду именных карточек.

    О последовательности

    Часть 1 просит студентов представить произведение двух факторов, за которым следует частное соответствующего факта деления, используя коэффициенты, меньшие или равные 5. В части 2 используются коэффициенты до 10, а в расширении предлагается практика с коэффициентами до 12.

    Часть 1

    Давайте попробуем наши факты умножения. Я приведу набор факторов, и вместе мы назовем продукт. Затем доброволец предоставит один связанный факт деления. Например, если я скажу 2 x 4, произведение будет 8, и один связанный факт деления будет 8 ÷ 4 = 2 (или 8 ÷ 2 = 4). Вот так!

    Примеры:

    • 2 × 5 (10, связанный факт деления 10 ÷ 5 = 2 или 10 ÷ 2 = 5)
    • 3 × 2 (6, связанный факт деления 6 ÷ 3 = 2 или 6 ÷ 2 = 3
    • 3 × 3 (9, связанный факт деления 9 ÷ 3 = 3)
    • 2 × 2 (4, связанный факт деления 4 ÷ 2 = 2)
    • 4 × 3 (12, связанный факт деления 12 ÷ 3 = 4 или 12 ÷ 4 = 3)
    • 4 × 4 (16, связанный факт деления 16 ÷ 4 = 4)
    • 5 × 3 (15, связанный факт деления 15 ÷ 5 = 3 или 15 ÷ 3 = 5)
    • 5 × 4 (20, связанный факт деления 20 ÷ 4 = 5 или 20 ÷ 5 = 4)

    Пока дети наслаждаются развитием мастерства, не стесняйтесь повторять. Когда дети хотят большего, попробуйте Часть 2.

    Часть 2

    Давайте продолжим работу над нашими связанными фактами умножения и деления, но на этот раз мы пойдем еще быстрее. (В какой-то момент вы можете позволить ученикам вести это задание.)

    Примеры:

    • 5 × 8 (40, связанный факт деления 40 ÷ 8 = 5 или 40 ÷ 5 = 8)
    • 9 × 5 (45, связанный факт деления 45 ÷ 9 = 5 или 45 ÷ 5 = 9)
    • 7 × 6 (42, связанный факт деления 42 ÷ 6 = 7 или 42 ÷ 7 = 6)
    • 6 × 10 (60, связанный факт деления 60 ÷ 6 = 10 или 60 ÷ 10 = 6)
    • 10 × 10 (100, связанный факт деления 100 ÷ 10 = 10)
    • 6 × 8 (48, связанный факт деления 48 ÷ 8 = 6 или 48 ÷ 6 = 8)
    • 9 × 4 (36, связанный факт деления 36 ÷ 4 = 9 или 36 ÷ 9 = 4)
    • 9 × 9 (81, связанный факт деления 81 ÷ 9 = 9)

    Как всегда, когда детям кажется, что новая задача взволнована, двигайтесь дальше.

    добавочный номер

    А теперь давайте найдем еще несколько продуктов и связанных с ними фактов о разделении.

    Примеры:

    • 11 × 10 (110, связанный факт деления 110 ÷ 10 = 11 или 110 ÷ 11 = 10)
    • 11 × 8 (88, связанный факт деления 88 ÷ 8 = 11 или 88 ÷ 11 = 8)
    • 11 × 5 (55, связанный факт деления 55 ÷ 5 = 11 или 55 ÷ 11 = 5)
    • 12 × 8 (48, связанный факт деления 48 ÷ 8 = 12 или 48 ÷ 12 = 8)
    • 11 × 3 (33, связанный факт деления 33 ÷ 3 = 11 или 33 ÷ 11 = 3)
    • 12 × 5 (60, связанный факт деления 60 ÷ 5 = 12 или 60 ÷ 12 = 5)
    • 12 × 7 (84, связанный факт деления 84 ÷ 7 = 12 или 84 ÷ 12 = 7)
    • 11 × 9 (99, связанный факт деления 99 ÷ 9 = 11 или 99 ÷ 11 = 9)
    • 12 × 3 (36, связанный факт деления 36 ÷ 3 = 12 или 36 ÷ 12 = 3)

    Порядок операций — PEMDAS

    Операции

    «Операции» означают такие вещи, как сложение, вычитание, умножение, деление, возведение в квадрат и т. Д.Если это не число, это, вероятно, операция.

    Но, когда вы видите что-то вроде …

    7 + (6 × 5 2 + 3)

    … какую часть нужно рассчитать в первую очередь?

    Начать слева и пойти направо?
    Или идти справа налево?

    Предупреждение: вычислите их в неправильном порядке, и вы можете получить неправильный ответ!

    Итак, давным-давно люди согласились следовать правилам при расчетах, а это:

    Порядок действий

    Действия, указанные в скобках, сначала

    4 × (5 + 3) = 4 × 8 =

    32

    4 × (5 + 3) = 20 + 3 =

    23

    (неправильно)

    Показатели (степени, корни) перед умножением, делением, сложением или вычитанием

    5 × 2 2 = 5 × 4 =

    20

    5 × 2 2 = 10 2 =

    100

    (неправильно)

    Умножьте или разделите перед сложением или вычитанием

    2 + 5 × 3 = 2 + 15 =

    17

    2 + 5 × 3 = 7 × 3 =

    21

    (неправильно)

    В противном случае просто идите слева направо

    30 ÷ 5 × 3 = 6 × 3 =

    18

    30 ÷ 5 × 3 = 30 ÷ 15 =

    2

    (неправильно)

    Как я все это помню…? ПЕМДАС!

    п

    P первые скобки

    E

    E xponents (т. е. степени, квадратные корни и т. Д.)

    MD

    M ultiplication и D ivision (слева направо)

    AS

    A ddition и S ubtraction (слева направо)

    Разделение и Умножение ранжируются одинаково (и идут слева направо).

    Сложить и вычесть ранг одинаково (и идти слева направо)

    Так сделай так:

    После того, как вы сделали «P» и «E», просто идите слева направо, выполняя любую «M» или «D», как вы их найдете.

    Затем идите слева направо, выполняя любые «A» или «S», когда найдете их.


    Вы можете вспомнить, сказав « P lease E xcuse M y D ear A unt S ally».
    Или … Пухлые эльфы могут потребовать перекус
    Попкорн Каждый понедельник Пончики Всегда воскресенье
    Съешьте, пожалуйста, вкусные яблочные штрудели мамы
    Везде люди принимали решения по суммам

    Примечание: в Великобритании говорят BODMAS (скобки, заказы, деление, умножение, сложение, вычитание), а в Канаде говорят BEDMAS (скобки, экспоненты, деление, умножение, сложение, вычитание). Все это означает одно и то же! Неважно, как вы это запомните, главное, чтобы вы все поняли правильно.

    Примеры

    Пример: как вычислить

    3 + 6 × 2 ?

    M ultiplication до A ddition:

    Сначала 6 × 2 = 12 , затем 3 + 12 = 15


    Пример: как вычислить

    (3 + 6) × 2 ?

    P первая цифра:

    Сначала (3 + 6) = 9 , затем 9 × 2 = 18


    Пример: как вы работаете

    12/6 × 3/2 ?

    M ultiplication и D ivision ранжируются одинаково, поэтому просто идите слева направо:

    Сначала 12/6 = 2 , затем 2 × 3 = 6 , затем 6/2 = 3

    Практический пример:

    Пример: Сэм бросил мяч прямо вверх со скоростью 20 метров в секунду, как далеко он улетел за 2 секунды?

    Сэм использует эту особую формулу, которая включает эффекты гравитации:

    высота = скорость × время — (1/2) × 9. 8 × время 2

    Сэм устанавливает скорость 20 метров в секунду и время 2 секунды:

    высота = 20 × 2 — (1/2) × 9,8 × 2 2

    Теперь о расчетах!

    Начать с: 20 × 2 — (1/2) × 9,8 × 2 2

    Сначала скобки: 20 × 2 — 0,5 × 9,8 × 2 2

    Тогда экспоненты (2 2 = 4): 20 × 2 — 0,5 × 9,8 × 4

    Затем умножается: 40 — 19,6

    Вычесть и СДЕЛАНО! 20.4

    Мяч достигает 20,4 метра за 2 секунды

    Показатели экспоненты …

    А как насчет этого примера?

    4 3 2

    Показатели — особые: идут сверху вниз (сначала экспонента сверху). Итак, вычисляем так:

    Начать с: 4 3 2
    3 2 = 3 × 3: 4 9
    4 9 = 4 × 4 × 4 × 4 × 4 × 4 × 4 × 4 × 4: 262144

    Так 4 3 2 = 4 (3 2 ) , а не (4 3 ) 2

    И, наконец, как насчет примера с самого начала?

    Начать с: 7 + (6 × 5 2 + 3)

    Сначала скобки , а затем Показатели : 7 + (6 × 25 + 3)

    Затем Умножить : 7 + (150 + 3)

    Затем Добавьте : 7 + (153)

    Скобки завершены: 7 + 153

    Последняя операция — это Добавить : 160

    Умножение и деление

    Четвертый класс использует то, что было изучено в третьем классе. Третий класс вводит и обучает всем фактам умножения и деления от нуля до двенадцати. Как часто говорят, ученики ИЗУЧАЮТ факты в третьем классе, чтобы они могли ИСПОЛЬЗОВАТЬ факты в четвертом и всю оставшуюся жизнь.

    Мы начинаем с нуля и единицы, затем с двоек и троек. В самом начале мы узнаем, что факторы можно переключать, а не изменять продукт. Это называется коммутативным свойством . Затем мы учим пятерки и десятки, а затем четверки и девятки.После этого узнаем остальное по порядку. Причина этого двоякая. От нуля до трех вводится теория умножения. Это идея о том, что умножение — это «повторное сложение», а деление — это обратное умножению. Факты следуют шаблону как умножения, так и деления. Затем мы переходим к пятеркам и десяткам, потому что эти закономерности сразу распознаются как счет пропусков. Далее идут четверки, потому что они увеличивают диапазон используемых факторов.Следом идут девятки из-за уникального рисунка изделий. Студентов также учат, что, просто комбинируя уже изученные уравнения, они могут найти ответ на любое из тех, которые им еще не известны. Это распределительная собственность . Например, 3×6 плюс 4×6 дает тот же ответ, что и 7×6, без заучивания семерок. Это работает и для деления.

    Мы изучаем свойства умножения, многие из которых похожи на свойства сложения.С помощью этих свойств мы можем узнать, «почему» умножение работает, и как применять уравнения для решения многих повседневных событий. Знакомя с алгеброй, например, решая неизвестное число, представленное буквой, например «х», дети узнают, что любое уравнение можно сбалансировать, просто зная две из трех переменных и какой процесс использовать. Уравнение типа 3 x X = 15 просто превращается в задачу деления 15 ÷ 3 = X.

    Символы умножения и деления, выражения и отношения

    Назначение

    Этот модуль развивает понимание умножения и деления, в том числе обратной связи между этими двумя операциями, а также того, когда и как их использовать в ситуациях решения проблем.Студенты изучают правила представления операций умножения и деления в виде уравнений.

    Конкретные результаты обучения

    • Прочтите, запишите и поймите символы умножения и деления, знак равенства и язык, связанный с этими символами.
    • Напишите контекст истории для заданных уравнений умножения и деления.
    • Признайте, что операция умножения коммутативна.
    • Определите связанные факты умножения и деления («семейства фактов»).
    • Распознайте обратную связь между операциями умножения и деления.
    • Признайте, что деление не коммутативно.
    • Используйте слова «фактор» и «продукт» надлежащим образом и определите факторы заданных сумм.

    Описание математики

    Эта последовательность уроков устанавливает связь между повторным сложением и умножением. Он вводит деление и исследует взаимосвязь между операциями умножения и деления.

    В рамках этих уроков развиваются три основных понимания.

    1. Учащимся необходимо понимать отношения между величинами, которые представлены уравнениями умножения и деления. Например, 4 x 5 = 20 может означать, что «четыре количества из пяти равны 20» или «20 в четыре раза больше 5».
    2. Учащимся необходимо выучить словарный запас, связанный с умножением и делением, а также значение этих слов.Важный словарь включает в себя факторы (умножаемые числа), произведение (ответ на умножение), умножение на (увеличение одного количества в x раз), равенство (одинаковость количества).
    3. Умножение можно также представить в пространстве. Массивы — это мощный способ показать структуру и шаблон нескольких групп, и, в этом случае, прочно увязать умножение и деление с измерением.

    При изучении структуры и паттерна умножения и деления, основное внимание также уделяется раннему пониманию свойств числа .В этих уроках формально исследуется коммутативность умножения. Свойство распределения, при котором один или оба фактора разделяются (например, 12 x 55 = 10 x 55 + 2 x 55), является основополагающим для стратегий вычислений, включая письменные алгоритмы.

    При изучении поведения операций умножения и деления важно, чтобы учащиеся сделали обобщений , в которых они могли бы заявить, «что всегда происходит», когда предпринимаются определенные действия. Например, признание того, что правило «перевернуть» (коммутативное) равно , всегда верно для умножения, но это неверно для деления.

    Эта серия уроков посвящена однозначным множителям и делителям. Он признает, что для построения правильного понимания того, как мы используем символы и выражения умножения и деления для математического мышления и для выражения отношений, учащиеся должны иметь много возможностей для представления операций для решения словесных задач. Студенты также должны уметь создавать контексты, которые может выразить уравнение. Установление связей между языком и символами важно для развития правильного понимания математических идей и концепций.

    Ссылки на числовую структуру
    Ранняя добавка (стадия 5)
    Расширенная добавка (стадия 6)

    Возможности адаптации и дифференциации

    Возможности обучения в этом модуле можно дифференцировать, предоставляя или прекращая поддержку учащимся и изменяя требования к заданиям. Способы дифференциации включают:

    • Обеспечьте физические материалы, чтобы учащиеся могли предвидеть действия и обосновывать свои решения. Используйте такие материалы, как кубики и квадратные плитки, чтобы моделировать ситуации и связывать стратегии, используемые учащимися, с представленными количествами. Прогресс в создании диаграмм массивов на бумаге в квадрат.
    • Соедините символы и математический словарь, особенно символы для умножения и деления (x, ÷) и для равенства (=). Явно смоделируйте правильное использование уравнений и алгоритмов и обсудите значение символов в контексте.
    • Изменить сложность используемых чисел.Умножение на такие множители, как два, четыре, пять, десять и деление на те же делители, как правило, проще, чем на множители, такие как три, шесть, семь, восемь и девять.
    • Поощряйте студентов к сотрудничеству в небольших группах, а также к тому, чтобы делиться своими идеями и оправдывать их.
    • Используйте технологии, особенно калькуляторы, для прогнозирования, основанных на шаблонах, для оценки продуктов и коэффициентов, например Если ответ на 4 x 8 = 32, ответ на 32 ÷ 5 будет больше или меньше 8? Откуда вы знаете? Разрешите использование калькуляторов там, где вы хотите, чтобы учащиеся больше концентрировались на процессе получения разумного ответа или на обнаружении закономерностей, чем на отработке навыков вычислений.

    Контекст, используемый для этого устройства, — лоскутные одеяла и ткань тапа. Вы можете изменить контекст на ситуации, более соответствующие повседневной жизни, интересам или культурной самобытности ваших учеников. Массивы распространены в разных культурах и могут быть найдены в узорах плитки, текстиле, упаковках, сваях для домов и игровых досках для игр. Поощряйте учеников проявлять творческий подход, принимая различные стратегии от других и прося учеников создавать свои собственные проблемы для решения другими в значимых контекстах.

    Необходимые ресурсные материалы

    • Не менее двух прямоугольных стеганых одеял или ткани тапа
    • Цветные пластиковые квадратные плитки (или маленькие квадраты разноцветных карточек)
    • Квадратная бумага
    • Калькуляторы
    • Кубы Unifix
    • Карты игральные
    • Первая и вторая точки PowerPoints
    • Один, два и три копировальных аппарата

    Деятельность

    Сессия 1

    Деятельность 1

    1. Покажите ученикам два разных прямоугольных лоскутных одеяла. Или используйте PowerPoint One, чтобы показать фотографии подходящих одеял или тапа. Например:

      Предположим, что класс собирается сшить лоскутное одеяло или ткань тапа для детской палаты в местной больнице (или хосписе).
      Вовлеките студентов в обсуждение квилтинга, выясняя, как создаются рисунки.

    2. Спросите: «Какая математика содержится в этих лоскутных одеялах?» (например, лоскутное одеяло 3 x 3)
      Запишите идеи учащихся в таблицу класса. (Они могут включать число, геометрию, формулировки измерений: например, 3 + 3 + 3 = 9, 3 x 3 = 9, 9 квадратов, один большой квадрат, стороны одинаковой длины, 9, разделенные на 3 и т. Д.). Сравните количество квадратов в разных примерах.
    3. Выделите операцию , связь и символов (или слов), которые были записаны. Например:
    4. Напишите каждый символ на отдельном листе бумаги формата А4. Попросите пары учащихся взять один лист (один символ), и каждая по очереди запишет за 2 минуты , используя слова и изображения / диаграммы , мозговой штурм всего, что они знают об этом символе (или слове). Попросите студентов привести пример того, где можно использовать их символ.
    5. Попросите учащихся вернуться на коврик, сидя в отдельных двух группах: группа с операцией символов (+ — x ÷) и группа с соотношением символов ( =). Попросите выбранные пары учеников объяснить, почему они сидят там, где они находятся, и какие идеи они записали для своих символов.
      В этом обсуждении выделите используемый язык , получите представление о том, что такое операция с числами (математический процесс, который изменяет число или сумму), и рассмотрите значение , равного знаку .
      Сохраните листы мозгового штурма для использования в будущем.

    Деятельность 2

    1. Подготовьте пакеты из 12, 18, 20, 24 и 30 пластиковых плиток, маленьких цветных квадратов карт или тканевых квадратов. Сделайте это, карандаши и бумагу, доступными для студенческих пар.

      Задайте проблему. «Покажите, с помощью диаграмм и формул , сколькими различными способами вы можете расположить эти заплатки, чтобы сделать« мини-лоскутное одеяло »?»
      Попросите учащихся работать в парах, чтобы записать свои идеи.

    2. Попросите учеников поделиться своими идеями с парой, у которой было одинаковое количество плиток, и запишите все аранжировки, о которых они не думали.
    3. В классе делитесь идеями, исследуйте и фиксируйте ключевые моменты в таблице класса. Сохраните эту студенческую работу для Занятия 2.
      Например: Из пакета с 18 «заплатками» (плитками).

      В ходе обсуждения основывайтесь на идеях, изложенных в Задании 1 (выше), выделяя и записывая словами следующие идеи:

      • Аранжировки «патчей» могут быть записаны с использованием различных операций .
      • Умножение
      • с использованием символа x может показать ту же идею как повторное сложение (равных величин) с использованием символа + .
      • Символ для разделения или разделения на равные группы: ÷ . Он называется символом деления .
      • Это расположение с равными строками и столбцами называется массивом .
    4. Поза и запись: «9 + 9 = 6 x 3. Вы согласны или не согласны». Попросите пары учащихся обсудить это утверждение и подготовиться к обоснованию своей позиции (объясните, почему они согласны или не согласны, и откуда они знают, что они правы).
      Запишите обоснование учащегося, выделив отношение эквивалентности (оба равны 18, всего 18 патчей в обоих массивах). Выделите мультипликативные представления, такие как «9 равно 3 x 3, поэтому 9 + 9 равно 6 x 3».

    Деятельность 3

    Напишите в таблице классов два уравнения: одно умножение и одно деление.
    Например: 6 x 5 = 30 28 ÷ 4 = 7. Прочтите их вместе. Попросите каждого ученика нарисовать схему лоскутного одеяла или ткани тапа, которая представляет уравнение. Попросите их написать словами, как одеяло / ткань представляет уравнение.

    Деятельность 4

    Завершите сеанс, рассмотрев символы операций и отношений и их значения.

    Сессия 2

    Деятельность 1

    1. Начните с того, что по крайней мере два ученика поделятся своими схемами лоскутного одеяла / ткани с предыдущего занятия. Попросите других студентов записать уравнения, представленные на схеме. Подчеркните тот факт, что математику из реальной жизни можно представить с помощью диаграмм, слов и символов.
    2. Проведите мозговой штурм по таблице класса для других ситуаций в нашей жизни, в которых мы видим и используем умножение или деление. По мере того, как учащиеся обмениваются идеями, попросите их назвать конкретные числа.Запишите эти истории, используя схемы и слова.
      Например: Мы видим умножение, когда:
      • 12 пакетов по 20 изюмов ​​упакованы в большую пачку — четыре пакета в ряд и три ряда.
      • Вы покупаете три пачки жевательной резинки по десять штук в каждой пачке
      • Для спортивной игры по физкультуре делаем четыре команды по шесть человек.
    3. Прочтите рассказы еще раз вместе. Попросите учащихся использовать символы для записи уравнений для каждого из рассказов в своих книгах / на доске / бумаге.Те студенты, которые заканчивают быстро, могут придумать больше контекстных историй.
      Попросите учащихся поделиться своими уравнениями в парах. Если учащиеся записали, используя повторное сложение, попросите их также записать уравнения умножения.

    Деятельность 2

    1. Просмотрите информацию о символах из сеанса 1, выделив символы операций, + — x ÷, и символы взаимосвязи, равно (=), больше (>) и меньше ( Попросите учащихся поработать в парах, используя ситуации из предыдущего задания.Студенты должны обсудить ситуации и посмотреть, сколько уравнений или неравенств они могут написать, например:
      3 x 4 = 4 x 3
      3 x 4 4 x 6> 2 x 10> 4 x 3
      Им следует использовать диаграммы, чтобы показать, откуда они знают, что они верны.
    2. Попросите учеников поделиться своими работами в паре. При этом они должны по очереди прочитать вслух то, что они написали.

    Деятельность 3

    1. Вернитесь к лоскутным одеялам / тапам (фотографии).Объясните, что некоторым маленьким детям нравятся лоскутные одеяла с алфавитом, в которых на каждой нашивке изображено что-то, начинающееся с другой буквы алфавита. Поговорите о том, что некоторые из них могут быть. Например: A может изображать яблоко, B — бабочку, C — кошку и так далее.
    2. Раздайте ученикам бумагу, карандаши и фломастеры.
      Задайте задачу: Вы собираетесь сделать лоскутное одеяло / тапа с алфавитом для маленького ребенка. У вас есть до конца сегодняшнего занятия, чтобы спланировать свой дизайн и то, как вы расположите свои «квадратные пятна» .Где-то в проблеме может быть проблема. Вы, , решаете, как лучше всего решить эту проблему для своего дизайна лоскутного одеяла.
      Сколько букв в алфавите? (26)
      Почему сделать квилт с 26 квадратами может быть проблемой?
    3. Предложите учащимся поэкспериментировать с 26 квадратами. Они могут нарисовать возможные варианты использования квадратных плиток или кубиков.
      (26 образуют только массивы 1 x 26 или 2 x 13, что нежелательно для стеганого одеяла такого типа. Учащиеся столкнутся с «остатком» (6 x 4 + 2, 5 x 5 + 1) или найдут это какие-то «заплатки» короткие (7 x 4).Принимайте реалистичные решения для контекста. (например, одеяло 5 x 5: поместите 2 буквы на один патч, одеяло 6 x 4: сделайте его размером 7 x 4 и включите 2 романа или пустых патча.)
    4. Предложите: Если мы добавим патчи для каждой из цифр 0-9, сколько патчей у нас будет тогда? (26 + 10 = 36)
      Посмотри, какие лоскутные одеяла ты тогда сможешь сделать.
      Поищите учащихся, чтобы они нашли все возможные варианты:
      1 x 36 2 x 18 3 x 12 4 x 9 6 x 6
      Какая ткань является лучшей тканью для квилтинга / тапа? Почему?

    Сессия 3

    Деятельность 1

    1. Попросите учащихся поделиться своими эскизами лоскутных одеял с алфавитом для 36 заплат. Обсудите «оставшуюся проблему» и порекомендуйте творческие решения.
      Почему невозможно было изготовить лоскутное одеяло с пятью заплатами подряд?
      Запишите 36 ÷ 5 = 7 r 1 и спросите учащихся, что означает r 1 (остаток от 1).
      Укажите, что часто проблемы с разделением не решаются равномерно. Мы называем то, что осталось, остатком .
    2. Представьте, что у нас есть 26 патчей, и мы пытаемся разместить по шесть патчей в каждом ряду. Один из способов записать эту проблему — 26 ÷ 6 = 4 r2.
    3. На диаграмме классов быстро нарисуйте массивы, разработанные для 26 патчей.
    4. Обсудите «размеры» массива, введя слова факторы и продукт . Модель с примером:

      Попросите каждого ученика записать под своим дизайном лоскутного одеяла, что находится в поле выше, корректируя числа в соответствии с их собственным дизайном.

    Деятельность 2

    1. Напишите на доске 4 36 9.
      Вот еще три числа, которые связаны умножением и делением.
      Запишите набор уравнений умножения и деления, используя эти числа.

    2. Попросите учащихся работать в парах, чтобы разработать уравнения и создать массив, представляющий все четыре уравнения. Студенты должны быть готовы обосновать свою позицию (объяснить, откуда они знают, что они правы).
      4 x 9 = 36 9 x 4 = 36 36 ÷ 4 = 9 36 ÷ 9 = 4
      Свяжите каждое уравнение с массивом 9 x 4, который учащиеся должны распознать по дизайну как задание для квилтинга.Обратите особое внимание на разделение. Например, 36 ÷ 4 = 9 дает количество строк, созданных из 36 фрагментов (области), если каждый ряд состоит из четырех фрагментов.

    3. Обобщите полученные данные в таблице класса. Например:
      • Есть только четыре связанных факта, (семейство фактов) и не более.
        4 x 7 = 28 7 x 4 = 28 28 ÷ 4 = 7 28 ÷ 7 = 4
      • Умножение — это «оборотная» операция. Вы можете изменить порядок факторов, не меняя продукт. (Это похоже на сложение.)
        Мы говорим, что умножение (и сложение) коммутативны .
        4 х 7 = 7 х 4 = 28
      • Division не коммутативная, например 36 ÷ 4 = 9, но 4 ÷ 36 = 0,1111… (1/9). У делений разное частное (ответ).
        Мы говорим, что деление (и вычитание) не коммутативны .

    Деятельность 3

    1. Попросите учащихся сыграть в игру Умножь, нарисуй и напиши в парах .
      Им нужны игральные карты (с цифрами от 2 до 9), карандаш и бумага.
      Побеждает тот, у кого после десяти раундов больше всего пар карточек с одинаковыми товарами, но сделанными с разными факторами.
      Например: 6 x 4 = 8 x 3 = 24 или 4 x 4 = 2 x 8 = 16
      Как играть:
      Карты перемешиваются и кладутся лицом вниз в стопку между обоими игроками.
      Игроки по очереди переворачивают три карты из стопки. Это факторы.Игрок возвращает одну карту в конец стопки. Игрок должен записать факт / ы умножения для двух карт. Они также могут нарисовать массив и написать семейство фактов.
      Например:
    2. Учащиеся завершают занятие, записывая словесные сценарии для своих наборов уравнений (семейство фактов). Это не обязательно сценарии квилтинга.
      Например: «Было три мешка по пять яблок в каждом. Пятнадцать, разделенные на три сумки, составляют пять. Если эти пятнадцать яблок положить в пять пакетов, то в каждом будет по три.Это будет пять лотов из трех ».

    Сессия 4

    Деятельность 1

    Покажите альтернативное лоскутное одеяло или ткань тапа (PowerPoint Two). Например:

    Попросите четырех студентов записать по одному из связанных фактов.
    (6 x 5 = 30, 5 x 6 = 30, 30 ÷ 5 = 6, 30 ÷ 6 = 5) и объясните каждый факт со ссылкой на лоскутное одеяло, включая демонстрацию коммутативного (поворотного) свойства умножения. Поверните одеяло, чтобы продемонстрировать это.

    Деятельность 2

    1. Раздайте учащимся связующие кубики (или цветные фишки). Попросите пары учеников взять по 48 кубиков. Спросите, какие факторы могут дать 48. Запишите возможности, используя умножение; 1 x 48, 2 x 24, 3 x 16 и т. Д.
    2. Пусть по одному учащемуся из каждой пары учеников моделируют 4 x 12, соединяя кубики. Затем пусть их партнер использует те же кубики, чтобы моделировать 12 x 4. Обсудите, что происходит. (Им нужно было их перегруппировать). Повторите с 6 х 8 и 8 х 6.Подчеркните, что коммутативное свойство включает в себя те же факторы и продукт, но требует другого способа просмотра массива (т.е. строки или столбец образуют равные наборы).
    3. Поместите факты умножения 48 на карты (Копимастер Один). Сдержать 5 раз? и 7 х? Сопоставьте пары уравнений, которые показывают коммутативность.
      Как вы думаете, это все факты умножения на произведение 48? (Вы можете расположить карты по первому множителю.)
      Почему нет фактов 5 x и 7 x? (Используйте карточки. Учащиеся должны понимать, что 48 не входит в число, кратное 5 и 7. 48 не делится на 5 и 7).
      Воспользуйтесь калькулятором, чтобы показать, что 48 ÷ 5 = 9,6 и 48 ÷ 7 = 6,857142857…
      Как вы думаете, что показывает десятичная часть произведения? (остаток, поэтому 48 не делится на 5 и 7)
    4. Попросите учащихся изучить факты умножения с разным количеством кубиков, используя язык тех же факторов и продукта, уделяя особое внимание перегруппировке.Исследование может показать, что некоторые числа имеют только два делителя, например 17 и 31. Это простые числа.

    Деятельность 3

    1. Запишите одно знакомое уравнение умножения в таблицу классов. Например, 6 x 2 = 12. Попросите одного из учеников в каждой паре смоделировать это, составив 6 групп по 2 и соединив кубики в одну линию из 12.

      Запишите 12 ÷ 6 = 2. Другой Студент в паре разыграет это кубиками.

      Попросите учащихся описать то, что произошло, и записать такие идеи, как: это противоположное, деление без умножения, все наоборот, мы вернулись к тому, с чего начали.
      Спросите, Всегда ли это правда? Как мы можем узнать? Принимайте идеи студентов. Сюда должны входить учащиеся, изучающие больше примеров.

    2. Сделайте вывод, что невозможно проверить все факты умножения и деления. Скажем, идея «отмены» означает, что умножение и деление являются обратными операциями, как включение и выключение света.Отмена друг друга — это просто способ, которым ведут себя умножение и деление.

    3. Запишите обратную связь в таблице классов. Обсудите слова, похожие на обратное, например: перевернуть, отменить, вернуть, вернуть и их значение. Установите связь с обратной зависимостью между сложением и вычитанием. Выделите, что в каждой паре операций одна операция или действие отменяет другое.

    4. Вернитесь к лоскутному одеялу в упражнении 1 (выше) и к записанным уравнениям:
      (6 x 6 = 30, 5 x 6 = 30, 30 ÷ 5 = 6, 30 ÷ 6 = 5)
      Попросите учащихся объяснить « отмена »(снова обратная зависимость, применительно к лоскутному одеялу. (Это немного труднее увидеть, потому что этот массив физически невозможно «отменить». Однако вы можете создать ряды из шести кубов и отобразить 5 x 6, расположив пять рядов по вертикали. Сколько у меня патчей? Что произойдет, если я теперь разделите на пять? )

    5. Напишите в таблице класса:
      Знание того, что умножение и деление являются обратными операциями, полезно, потому что …… ..
      Попросите учащихся указать причины и записать их, в том числе:
      Мы можем использовать умножение, чтобы помочь нам решить задачи деления.
      Мы можем проверить операции деления с помощью умножения. (Как?)

    Операция 4

    Раздайте Copymaster Two студентам для работы. Подчеркните обратные операции, и необходимость для учащихся показать или объяснить , как умножение помогает решать задачи деления.

    Сессия 5

    Деятельность 1

    Просмотрите основные выводы занятия 4. Предложите учащимся поработать в парах, чтобы поделиться своими решениями задач лоскутного одеяла из занятия 4, занятие 4.Поощряйте их задавать вопросы друг другу.

    Деятельность 2

    1. Покажите несколько примеров стеганой ткани или ткани тапа с помощью PowerPoint One:
    2. Напишите в таблице класса:
      Одно лоскутное одеяло из шестнадцати заплат:
      Одно одеяло из тридцати заплат:
      Одно лоскутное одеяло из сорока пяти заплат:
      Если я расположу лоскутки в один ряд, как будет выглядеть лоскутное одеяло? (Больше похоже на длинный шарф)
    3. Попросите студентов записать уравнения умножения для каждого из этих утверждений.
      Одно лоскутное одеяло из шестнадцати заплат: 1 x 16 = 16
      Одно лоскутное одеяло из девяти заплат: 1 x 30 = 30
      Одно лоскутное одеяло из тридцати заплат: 1 x 45 = 45
      Если ваши уравнения верны, каковы ответы на 16 ÷ 1 = ☐, 30 ÷ 1 = ☐, 45 ÷ 1 = ☐?
    4. Попросите студентов обсудить свои идеи, а затем объяснить и обосновать свое мышление. Связаны ли они с делением с вопросом «Сколько столбцов в одном патче составляет в общей сложности 16, 30 или 45 патчей?»
      Если ваши уравнения верны, каковы ответы на вопросы: 16 ÷ 16 = ☐, 30 ÷ 30 = ☐, 45 ÷ 45 = ☐?
      Относят ли они разделение к следующему: «Сколько рядов по 16 фрагментов составляют в общей сложности 16 фрагментов и т. Д.?»
    5. Приведите другие примеры деления числа на единицу и само себя.Калькуляторы можно использовать для проверки ответов.

    Деятельность 3

    Попросите учащихся работать группами от 2 до 4 человек по телефону Это факт? (Копимастер Три (Цель: различать правильные и неправильные уравнения и выражения умножения и деления и уметь объяснять, почему, обосновывая свое решение)

    Учащиеся по очереди выбирают утверждение и объясняют остальным в группе, является ли утверждение фактом и почему оно неверно (верно или неверно) и почему.

    Попросите учащихся создать свои собственные факты или не факты, которые включают умножение и деление, например 8 x 9 = 72, поэтому 72 ÷ 18 = 4. Обменивайтесь фактами и не-фактами между учащимися.

    Деятельность 4

    Завершите это занятие, проанализировав обучение, полученное за пять занятий.

    Домашняя ссылка

    Дорогие родители и ванау,

    На этой неделе по алгебре мы изучали числовые операции умножения и деления и взаимосвязь между ними.

    Один из способов закрепить то, что они узнали, — это прочитать истинные и ложные утверждения, а также определить и объяснить, какие из них неверны и почему.

    Ваш ребенок может захотеть поиграть на Это факт? игра с вами. По очереди очень важно, чтобы каждый человек объяснил , почему утверждение является верным или нет.

    Надеемся, вам понравится этот вызов.

    Спасибо.

    Попробуйте эти примеры:

    1 x 25 = 25, поэтому 25 ÷ 25 = 1 Верно или неверно

    28 ÷ 4 = 7, поэтому 4 ÷ 28 = 7 Верно или неверно

    Делитель десятичный.Недвижимость раздела

    Урок 11 Раздел 3

    Вернуться в раздел 1

    Вернуться в раздел 2

    Дивиденд ÷ Делитель = Частное

    Дивиденды
    Делитель
    = Частное

    (Разделительный стержень. )

    1. Число не изменится, если мы умножим его, а затем разделим произведение
    на то же число:
    (45 × 100) ÷ 100 = 45;
    или если мы разделим, а затем умножим частное на то же число.
    (4500 ÷ 100) × 100 = 4500.

    Ниже мы объясним, почему это так.

    Из-за этого свойства мы говорим, что умножение и деление являются обратными операциями.

    2. Частное не изменится, если мы умножим и делимое, и делитель
    на одно и то же число; или если мы разделим на одно и то же число.
    8
    2
    = 4.
    3 × 8
    3 × 2
    = 24
    6
    = 4.
    5 × 8
    5 × 2
    = 40
    10
    = 4.

    О делении делимого и делителя на одно и то же число:

    36 000
    12 000
    = 36,000 ÷ 1000
    12,000 ÷ 1000
    = 36
    12
    = 3.
    360
    120
    = 360 ÷ 10
    120 ÷ 10
    = 36
    12
    = 3.

    Таким образом, мы видим, что когда делимое и делитель оканчиваются одним и тем же числом нулей, мы можем проигнорировать их и разделить оставшиеся числа.

    Пример 1. 1200 ÷ 30 = 120 ÷ 3 = 40.
    Чек: 40 × 30 = 1200.
    Пример 2. 1 . 2
    30
    =?

    Ответ . Чтобы упростить ситуацию, мы можем сделать делимое целым числом, умножив его на 10. Но тогда мы должны умножить делитель также на 10:

    .

    Затем разделите на 3. И, наконец, разделите на 100. (Урок 4.)

    12
    300
    = 4
    100
    = . 04

    Теперь мы увидим, как применить свойство 2, когда делитель является десятичным. Потому что делитель не может быть десятичным. Делитель должен быть целым числом

    Пример 3. Делитель десятичной дроби. Сколько раз . 2 содержится
    в 6?

    Ответ . . 2 — делитель: 6
    . 2
    . Но чтобы разделить,

    Делитель должен быть целым числом.

    Следовательно, мы должны сделать . 2 в целое число. Для этого умножьте его на 10. Но тогда мы должны также умножить 6 на 10:

    6
    . 2
    = 10 × 6
    10 × 90 10 8. 2
    = 60
    2
    = 30.

    Этот пример показывает, что, хотя мы говорим, что делим десятичные числа, мы действительно можем делить только целые числа, а затем правильно размещать десятичную точку. Мы еще раз увидим это в Уроке 13.

    Пример 4. 1 . 8
    . 03
    = 1 . 8 × 100
    . 03 × 100
    = 180
    3
    = 60.

    Почему мы умножили на 100? Потому что это делает делитель . 03 в целое число 3.

    Другими словами:

    Если делитель десятичный, превратите его в целое число
    , умножив его на 10, 100, 1000 и т. Д. В соответствии с
    на количество десятичных цифр. Затем умножьте дивиденд
    на ту же степень 10.

    В Уроке 12 мы увидим, что делать, когда дивиденд является десятичным.

    3. Когда деление обозначено полосой деления, и делимое состоит из факторов, тогда мы можем разделить любые на факторов.
    1) 12 × 8
    2
    = 6 × 8 = 48,

    при делении 12 на 2.

    Или,

    2) 12 × 8
    2
    = 12 × 4 = 48,

    при делении 8 на 2.

    Или, наконец, при умножении сначала:

    3) 12 × 8
    2
    = 96
    2
    = 48.

    Эти три возможности подразумевают:

    Когда деление обозначается полосой деления, то порядок
    , в котором мы умножаем или делим, не имеет значения.Мы можем либо сначала разделить, либо сначала умножить.
    Однако более умело сначала разделить, потому что тогда у нас есть меньшие числа для умножения.

    *

    Кроме того, если мы разложим делимое на множители, то иногда мы сможем найти кратное делителю.

    72
    4
    = 8 × 9
    4
    = 2 × 9 = 18.
    700
    25
    = 100 × 7
    25
    = 4 × 7 = 28.

    Объяснение того, почему свойство 3 верно, см. Ниже.

    Разъяснение свойств раздела

    Арифметика — первая наука. Мы смотрим на сами факты. Мы объясним свойства деления, посмотрев на арифметические значения умножения и деления и отношения между ними. Это не алгебра.

    1. Число не изменится, если мы умножим его, а затем разделим произведение
    на это же число; или если мы разделим его, а затем умножим частное
    на это же число.

    Чтобы проиллюстрировать это, давайте начнем с 5, а затем умножим его на 3:

    3 × 5

    А теперь разделим на 3. Давайте многократно вычтем тройки. Но поскольку 15 теперь состоит из 5, как мы можем это сделать?

    Согласно свойству порядка умножения,

    3 × 5 = 5 × 3.

    Итак, произведение — это , состоящее из троек — оно состоит из пяти троек.

    15 ÷ 3 = 5.

    Итак, мы вернулись туда, откуда начали, на 5.

    Ученик должен понять, что таблица умножения — 3 × 5 = 15 — здесь не главное. Дело в том, чтобы понять — для см. — что, хотя 3 × 5 является суммой 5, мы все же можем вычесть 3.

    Это свойство сохраняется для любых чисел.

    (206 × 19) ÷ 19 = 206.

    Понимание, не имеющее ничего общего со знанием «ответа» на 206 × 19

    Теперь давайте начнем с 15 и разделим его на 5:

    Получим 3, потому что

    3 × 5 = 15.

    А согласно приказу собственности,

    5 × 3 = 15.

    Следовательно, если теперь умножить 3 на 5, мы вернемся к 15.

    2. Частное не изменится, если мы умножим и делимое, и делитель
    на одно и то же число; или если мы разделим их на одно и то же число.

    Частное — это количество раз, когда делитель входит в состав делимого.

    Таким образом, столько же раз, когда мы увеличиваем или уменьшаем дивиденд — но не меняем делитель — частное будет увеличиваться или уменьшаться в то же количество раз.

    Если мы удвоим дивиденд, делитель будет вдвое больше, то есть частное удвоится. Если мы утроим дивиденд, частное утроится.

    С другой стороны, если мы разделим дивиденд на 2, то есть возьмем его половину —

    — тогда и частное будет половинным.И так далее.

    Затем, столько раз, сколько мы увеличиваем или уменьшаем делитель — и не меняем дивиденд — тогда, наоборот, частное будет уменьшаться или увеличиваться в то же количество раз.

    Если мы удвоим делитель, частное будет половиной. Если мы утроим делитель, частное будет на одну треть меньше.

    С другой стороны, если взять половину делителя —

    — уйдет вдвое больше.Если разделить делитель на 3, частное утроится. И так далее.

    Следовательно, если мы увеличим или уменьшим делимое и делитель одинаковое количество раз, частное не изменится.

    Другими словами: частное не изменится, если мы умножим делимое и делитель на одно и то же число или если мы разделим их на одно и то же число.

    3. Когда дивиденд состоит из факторов, мы можем разделить любые на факторов.

    (3 × 20) ÷ 5 = 3 × (20 ÷ 5)

    То есть, чтобы разделить 3 × 20 на 5, мы можем сначала разделить 5 на 20, а затем умножить на 3.

    Вот (3 × 20) ÷ 5 — три 20, разделенные на 5:

    Но это то же самое, что (20 ÷ 5) —

    — добавлено трижды. Следовательно,

    (3 × 20) ÷ 5 = 3 × (20 ÷ 5).

    Пожалуйста, «переверните» страницу и сделайте что-нибудь Проблемы.

    или

    Переходите к следующему уроку.

    Раздел 1 этого урока: Значение раздела

    Раздел 2: Мысленный расчет

    Введение | Главная | Содержание


    Авторские права © 2021 Лоуренс Спектор

    Вопросы или комментарии?

    Эл. Почта: [email protected]


    Умножение и деление десятичных знаков на 10, 100 и 1000 (степени десяти)

    Это полный урок с видео и упражнениями, показывающими, в первую очередь, общий ярлык для умножения и деления десятичных знаков на степень десяти: вы перемещаете десятичную точку на столько шагов, сколько нулей в числе 10, 100, 1000. и т.п.

    Затем я также показываю , где происходит от этого ярлыка, с помощью диаграмм значений разряда. На самом деле перемещение десятичной точки является своего рода иллюзией, и вместо этого цифр числа перемещаются внутри диаграммы разряда. Это объяснение действительно может помочь студентам понять причину «трюка» с перемещением десятичной точки.

    Урок ниже объясняет этот ярлык более подробно, а также содержит различные виды упражнений, задачи со словами и даже забавную загадку для учащихся.


    1. Умножить.

    а. 10 × 0,04 = ________

    г. 100 × 0,04 = ________

    г. 1000 × 0,04 = ________

    г. 10 × 0,56 = ________

    эл. 100 × 0,56 = ________

    ф. 1000 × 0,56 = ________

    г. 10 × 0,048 = ________

    ч. 100 × 0,048 = ________

    и. 1000 × 0,048 = _______

    Еще один полезный ярлык! Поскольку 100 × 2 = 200, очевидно, что ответ на
    100 × 2.105 будет немного больше 200. Следовательно, вы можете просто написать цифры
    2105 и поставьте десятичную точку так, чтобы ответ был 200 с чем-то: 210,5 .

    2. Давайте еще немного попрактикуемся.

    а. 100 × 5,439 = ________

    г. 100 × 4,03 = ________

    г. 1000 × 3,06 = ________

    г. 100 × 30,54 = ________

    эл. 30,73 × 10 = ________

    ф. 93,103 × 100 = _______

    10 5 = 100000 имеет пять нулей. Снова напишите дополнительные нули, чтобы десятичная дробь точка может «перепрыгнуть» в эти места.

    3. А теперь попрактикуемся в использовании степени десяти.

    а. 10 2 × 0,007 = _____________

    10 3 × 2,01 = _____________

    10 5 × 4,1 = ______________

    б. 10 5 × 41,59 = _____________

    3,06 × 10 4 = ______________

    0. 046 × 10 6 = _____________

    4. Разделить.

    а. 0,4 ÷ 10 = ________

    0,4 ÷ 100 = ________

    4,4 ÷ 100 = ________

    г. 15,4 ÷ 100 = ________

    21.03 ÷ 10 = ________

    0,39 ÷ 10 = ________

    г. 5.6 ÷ 10 = ________

    34,9 ÷ 100 = ________

    230 ÷ 1000 = ________

    5. Теперь попрактикуемся в использовании степени десяти.

    а. 0,7 ÷ 10 2 = _____________

    45,3 ÷ 10 3 = _____________

    568 ÷ 10 5 = _____________

    г. 2,1 ÷ 10 4 = _____________

    4,500 ÷ 10 6 = _____________

    9,13 ÷ 10 3 = _____________

    Почему работает этот БЫСТРЫЙ ВЫКЛ?

    Когда 0,01 (сотая) умножается на десять, мы получаем десятые сотые, что равняется одной десятой. Или 10 × 0,01 = 0,1.

    Все число переместилось на одну «ячейку» влево на диаграмме разряда.Этот выглядит как , перемещающий десятичную запятую в числе вправо.

    Сто умножить на два десятые — это как умножение каждой десятой на 10, а на 10 очередной раз. Десять раз по две десятых дает нам два, и в десять раз больше 20.

    Опять же, это похоже на перемещение числа на два «слота» влево на диаграмме значений разряда или перемещение десятичной точки в 0.2, два шага вправо.

    Т O т h чт
    0 0 . 2
    Т O т h чт
    2 0 .
    Когда умножается на 3,915 на 100 получаем 391,5. Каждая часть число (3, 9 десятых, 1 сотых, 5 тысячных) умножается на 100, поэтому каждый из них перемещает два «слота» в диаграмма размеченной стоимости. Этот это то же самое, что думать, что десятичная точка перемещается на два шаги вправо.
    H Т O т h чт
    3 . 9 1 5
    H Т O т h чт
    3 9 1 . 5
    Аналогичный ярлык для деления работает потому что деление — это , противоположное операции умножения — оно «Отменяет» умножение. Если мы переместим десятичную запятую в справа при умножении на 10, 100, 1000 и так далее, то это Вполне естественно, что правило деления сработает «наоборот».

    Дроби против деления. Если мы переместим десятичную точку решать 6 ÷ 100, получаем:

    0 0 6. 0 ÷ 100 = 0,060 = 0,06

    Давайте запишем 6 ÷ 100, используя дробную черту: это 6/100 или 6 сотых, что написано 0.06 в виде десятичной дроби. Следовательно, в этом случае вам не нужен как «ярлык», вы можете просто думать о дробях и десятичных дробях. Такие «связи» делают математику такой изящной!

    6. Разделить. Подумайте о дробях в десятичные дроби или воспользуйтесь ярлыком. Сравните проблемы в каждой коробке!

    7. Мешок с орехами весом 10 фунтов стоит. 72 доллара.
    Сколько стоит один фунт?

    8. Найдите цену 100 шары для пинг-понга, если один мяч для пинг-понга стоит 0 долларов.89.

    Больше думать о дробях и десятичных числах

    Если разделить любое целиком число на 1000, в ответе будет тысячных или три десятичные цифры. Это упрощает деление целых чисел на 1000: просто скопируйте дивиденд в качестве ответа (без запятых), а затем сделать его тремя десятичными цифрами :

    Примеры:

    819,302

    1000

    = 819.302

    41,300

    1000

    = 41,300 = 41,3

    8,000

    1000

    = 8. 000 = 8

    Обратите внимание: в последних двух случаях мы можем упростить результаты: с 41,300 до 41,3 и с 8,000 до 8.

    9. Разделите целые числа на 1000. Упростите окончательный ответ, отбросив все конечные десятичные дроби. нули.

    Аналогично:

    • Если вы разделите любое целое число на 10 , скопируйте делимое и сделайте его равным одна десятичная цифра .
    • Если вы разделите любое целое число на 100 , скопируйте делимое и сделать двумя десятичными цифрами .

    Примеры:

    72

    10

    = 7,2

    3,090

    100

    = 30,90 = 30,9

    74,992

    100

    = 749,92

    82,000

    10

    = 8200. 0 = 8 200

    10. Разделите целые числа на 10 и 100.

    11. Найдите десятую часть …

    а. $ 8

    г. 25,50 $

    г. $ 126

    12. Найдите одну сотую из …

    а. $ 78

    г. $ 4

    г. $ 390

    13. Пара обуви стоимостью 29 долларов была снижена на 3/10 ее цены.Какова новая цена? ( Подсказка: сначала найдите 1/10 цены. )

    14. Найдите цену со скидкой:

    а. На велосипед стоимостью 126 долларов скидка составляет 2/10 его цены.

    б. Сотовый телефон за 45 долларов со скидкой 5/100 от его цены.
    ( Подсказка: Сначала найдите 1/100 цены. )

    15. Одна сотая определенного числа составляет 0,03.Какой номер?

    16. Какой пылесос оказывается дешевле? Модель
    A, с начальной ценой 86,90 долларов, имеет скидку 3/10 от ее цены.

    Model B сейчас стоит 75 долларов, но вы получите скидку 1/4 его цены.

    Важный совет

    В проблеме ____ × 3,09 = 309, число 3 становится 300, поэтому очевидно, что
    недостающий коэффициент равен 100. Вам даже не нужно рассматривать десятичную дробь точка!

    То же самое и с делением. В проблеме 7,209 ÷ знак равно 7.209, недостающий делитель
    равен тысяче, потому что значение цифры 7 было первых 7000, а потом стало 7.

    Конечно, в некоторых проблемах это будет легче думать о «Перемещение десятичной точки».

    17. Пришло время для некоторых заключительная практика. Найдите недостающие числа. Сопоставьте букву каждой задачи с правильным ответом в квадратах и решить загадку. Есть два набора ящиков. Первые блоки относятся к первому набору упражнений, а последние блоки относятся ко второму набору.

    Почему 7 не понял, о чем идет речь в 3.14?

    E ____ × 0,04 = 40

    D ____ × 9,381 = 938,1

    H 1000 × 4,20 =

    D ____ × 7,31 = 731

    Т ____ × 0.075 = 0,75

    I 10 × 3,55 = ______

    N 100 × ______ = 4,2

    S 1000 × ______ = 355

    E ____ × 60,15 = 60,150

    4,200 1000 100 35,5 100 0,042 10 0. 355 1000 1000

    Т _____ ÷ 100 = 0,42

    П _____ ÷ 10 = 2,3

    N _____ ÷ 1000 = 4.2

    H

    100

    = 2,3
    I

    10

    = 0,42

    S 0,31 ÷ _____ = 0,031

    O 4,360 ÷ _____ = 4,36

    I 304,5 ÷ _____ = 3,045

    230 100 10 23 1000 4. 2 4 200 42



    Я также предлагаю бесплатные рабочие листы:
    Рабочие листы для умножения десятичных знаков на степени десяти
    Рабочие листы для деления десятичных знаков на степени десяти .



    Этот урок взят из книги Марии Миллер Math Mammoth Decimals 2 и размещен на сайте www.HomeschoolMath.net с разрешения автора. Авторские права © Мария Миллер.



    Математика Мамонт Десятичные 2

    Самообучающийся рабочий текст для 5-6 классов, который охватывает четыре операции с десятичными знаками до трех десятичных знаков, уделяя особое внимание десятичному умножению и делению. В книге также рассматриваются разряды, сравнение, округление, сложение и вычитание десятичных знаков. Есть много проблем с умственной математикой.

    Загрузить ($ 6.25) . Также доступен в печатном виде.

    => Узнайте больше и посмотрите бесплатные образцы!


    План урока по умножению и делению

    Оценка: 03

    CCSS.Math.Content.3.OA.A.2

    Интерпретация целочисленных частных целых чисел, например.g., интерпретируйте 56 ÷ 8 как количество объектов в каждой доле, когда 56 объектов разделены поровну на 8 долей, или как количество долей, когда 56 объектов разделены на равные доли по 8 объектов в каждом. Например, опишите контекст, в котором количество акций или групп может быть выражено как 56 ÷ 8.

    Оценка: 03

    CCSS.Math.Content.3.OA.A.3

    Используйте умножение и деление в пределах 100 для решения задач со словами в ситуациях, связанных с равными группами, массивами и измеряемыми величинами, например, используя рисунки и уравнения с символом неизвестного числа для представления проблемы.1

    Оценка: 03

    CCSS. Math.Content.3.OA.A.4

    Определите неизвестное целое число в уравнении умножения или деления, связывающего три целых числа. Например, определить неизвестное число, которое делает уравнение истинным в каждом из уравнений 8 ×? = 48, 5 = _ ÷ 3, 6 × 6 =?

    Оценка: 03

    CCSS.Math.Content.3.OA.B.5

    Применяйте свойства операций как стратегии умножения и деления. 2 Примеры: если известно 6 × 4 = 24, то также известно 4 × 6 = 24. (Коммутативное свойство умножения.) 3 × 5 × 2 можно найти как 3 × 5 = 15, затем 15 × 2 = 30 или 5 × 2 = 10, затем 3 × 10 = 30. (Ассоциативное свойство умножения. ) Зная, что 8 × 5 = 40 и 8 × 2 = 16, можно найти 8 × 7 как 8 × (5 + 2) = (8 × 5) + (8 × 2) = 40 + 16 = 56.(Распределительная собственность.)

    Оценка: 03

    CCSS.Math.Content.3.OA.B.6

    Поймите разделение как проблему с неизвестным фактором. Например, найдите 32 ÷ 8, найдя число, которое дает 32 при умножении на 8.

    Оценка: 03

    CCSS.Math.Content.3.OA.C.7

    Плавно умножайте и делите в пределах 100, используя такие стратегии, как взаимосвязь между умножением и делением (например,g. , зная, что 8 × 5 = 40, мы знаем 40 ÷ 5 = 8) или свойства операций. К концу 3 класса выучить по памяти все произведения двух однозначных чисел.

    Оценка: 03

    CCSS.Math.Content.3.OA.D.8

    Решите двухэтапные задачи со словами, используя четыре операции.Представьте эти проблемы, используя уравнения с буквой, обозначающей неизвестную величину. Оцените разумность ответов с помощью мысленных вычислений и стратегий оценки, включая округление.3

    Оценка: 03

    CCSS.Math.Content.3.OA.D.9

    Определите арифметические шаблоны (включая шаблоны в таблице сложения или таблице умножения) и объясните их, используя свойства операций.Например, заметьте, что четырехкратное число всегда четно, и объясните, почему четырехкратное число можно разложить на два равных слагаемых.

    Класс: 04

    CCSS.Math.Content.4.NBT.B.5

    Умножьте целое число до четырех цифр на однозначное целое число и умножьте два двузначных числа, используя стратегии, основанные на разрядах и свойствах операций. Проиллюстрируйте и объясните расчет с помощью уравнений, прямоугольных массивов и / или моделей площадей.

    Класс: 04

    CCSS.Math.Content.4.NBT.B.6

    Найдите целочисленные частные и остатки с четырехзначными дивидендами и однозначными делителями, используя стратегии, основанные на разряде, свойствах операций и / или взаимосвязи между умножением и делением.Проиллюстрируйте и объясните расчет с помощью уравнений, прямоугольных массивов и / или моделей площадей.

    Класс: 04

    CCSS.Math.Content.4.OA.A.1

    Интерпретируйте уравнение умножения как сравнение, e.g., интерпретируйте 35 = 5 × 7 как утверждение, что 35 в 5 раз больше 7 и в 7 раз больше 5. Представьте словесные утверждения мультипликативных сравнений как уравнения умножения.

    Класс: 04

    CCSS.Math.Content.4.OA.A.2

    Умножайте или делите для решения словесных задач, связанных с мультипликативным сравнением, например.g., используя рисунки и уравнения с символом неизвестного числа для представления проблемы, отличая мультипликативное сравнение от аддитивного. 1

    Класс: 04

    CCSS.Math.Content.4.OA.A.3

    Решите многоступенчатые задачи со словами, поставленные с целыми числами и получив ответы с целыми числами, используя четыре операции, включая задачи, в которых необходимо интерпретировать остатки.Представьте эти проблемы, используя уравнения с буквой, обозначающей неизвестную величину. Оцените разумность ответов с помощью мысленных вычислений и стратегий оценки, включая округление.

    Оценка: 05

    CCSS.Math.Content.5.NBT.B.5

    Плавно умножайте многозначные целые числа по стандартному алгоритму.

    Оценка: 05

    CCSS.Math.Content.5.NBT.B.6

    Находите частные целых чисел с дивидендами до четырех и двузначными делителями, используя стратегии, основанные на разряде, свойствах операций и / или взаимосвязи между умножением и делением.Проиллюстрируйте и объясните расчет с помощью уравнений, прямоугольных массивов и / или моделей площадей.

    Оценка: 05

    CCSS.Math.Content.5.NBT.B.7

    Сложить, вычесть, умножить и разделить десятичные дроби до сотых, используя конкретные модели или чертежи и стратегии, основанные на разряде, свойствах операций и / или взаимосвязи между сложением и вычитанием; свяжите стратегию с письменным методом и объясните используемую аргументацию.

    примеры на сложение, вычитание, умножение и деление — РОСТОВСКИЙ ЦЕНТР ПОМОЩИ ДЕТЯМ № 7

    Содержание

    Примеры табличного умножения и деления, 3 класс (карточка). | Материал по математике (3 класс) на тему:

    8 * 3          7 * 4          56 : 8                                      6 * 9           3 * 4             56 : 7

    5 * 6          9 * 7          24 : 4                                      5 * 7           8 * 4             24 : 3

    3 * 9          6 * 6          35 : 5                                      4 * 6           7 * 7             25 : 5

    2 * 8          4 * 9          18 : 9                                      9 * 5           2 *9              16 : 8

    7 * 6          8 * 4          64 : 8                                      8 * 6           5 * 6              42 : 6

    8 * 3          7 * 4          56 : 8                                      6 * 9           3 * 4             56 : 7

    5 * 6          9 * 7          24 : 4                                      5 * 7           8 * 4             24 : 3

    3 * 9          6 * 6          35 : 5                                      4 * 6           7 * 7             25 : 5

    2 * 8          4 * 9          18 : 9                                      9 * 5           2 *9              16 : 8

    7 * 6          8 * 4          64 : 8                                      8 * 6           5 * 6              42 : 6

    8 * 3          7 * 4          56 : 8                                      6 * 9           3 * 4             56 : 7

    5 * 6          9 * 7          24 : 4                                      5 * 7           8 * 4             24 : 3

    3 * 9          6 * 6          35 : 5                                      4 * 6           7 * 7             25 : 5

    2 * 8          4 * 9          18 : 9                                      9 * 5           2 *9              16 : 8

    7 * 6          8 * 4          64 : 8                                      8 * 6           5 * 6              42 : 6

    8 * 3          7 * 4          56 : 8                                      6 * 9           3 * 4             56 : 7

    5 * 6          9 * 7          24 : 4                                      5 * 7           8 * 4             24 : 3

    3 * 9          6 * 6          35 : 5                                      4 * 6           7 * 7             25 : 5

    2 * 8          4 * 9          18 : 9                                      9 * 5           2 *9              16 : 8

    7 * 6          8 * 4          64 : 8                                      8 * 6           5 * 6              42 : 6

    8 * 3          7 * 4          56 : 8                                      6 * 9           3 * 4             56 : 7

    5 * 6          9 * 7          24 : 4                                      5 * 7           8 * 4             24 : 3

    3 * 9          6 * 6          35 : 5                                      4 * 6           7 * 7             25 : 5

    2 * 8          4 * 9          18 : 9                                      9 * 5           2 *9              16 : 8

    7 * 6          8 * 4          64 : 8                                      8 * 6           5 * 6              42 : 6

    8 * 3          7 * 4          56 : 8                                      6 * 9           3 * 4             56 : 7

    5 * 6          9 * 7          24 : 4                                      5 * 7           8 * 4             24 : 3

    3 * 9          6 * 6          35 : 5                                      4 * 6           7 * 7             25 : 5

    2 * 8          4 * 9          18 : 9                                      9 * 5           2 *9              16 : 8

    7 * 6          8 * 4          64 : 8                                      8 * 6           5 * 6              42 : 6

    8 * 3          7 * 4          56 : 8                                      6 * 9           3 * 4             56 : 7

    5 * 6          9 * 7          24 : 4                                      5 * 7           8 * 4             24 : 3

    3 * 9          6 * 6          35 : 5                                      4 * 6           7 * 7             25 : 5

    2 * 8          4 * 9          18 : 9                                      9 * 5           2 *9              16 : 8

    7 * 6          8 * 4          64 : 8                                      8 * 6           5 * 6              42 : 6

    Таблица умножения и игра, чтобы быстро выучить

    С лучшей бесплатной игрой таблица умножения учится очень быстро. Проверьте это сами!

    Учить таблицу умножения — игра

    Попробуйте нашу обучающую электронную игру. Используя её, вы уже завтра сможете решать математические задачи в классе у доски без ответов, не прибегая к табличке, чтобы умножить числа. Стоит только начать играть, и уже минут через 40 будет отличный результат. А для закрепления результата тренируйтесь несколько раз, не забывая о перерывах. В идеале – каждый день (сохраните страницу, чтобы не потерять). Игровая форма тренажера подходит как для мальчиков, так и для девочек.

    Таблица умножения – таблица, где строки и столбцы озаглавлены множителями (1, 2, 3, 4, 5…), а ячейки таблицы содержат их произведение. Применяется таблица для обучения умножению. Здесь есть игра и картинка для печати. Для скачивания игры с таблицей на компьютер, сохраните страницу (Ctrl+S). Также посмотрите таблицу деления.

    Смотрите ниже шпаргалки в полной форме.

    Распечатать таблицу умножения

    Умножение прямо на сайте (онлайн)

    *

    https://uchim. org/matematika/tablica-umnozheniya — uchim.org


    Таблица умножения (числа от 1 до 20)
     ×1234567891011121314151617181920
    11234567891011121314151617181920
    2246810121416182022242628303234363840
    33691215182124273033363942454851545760
    448121620242832364044485256606468727680
    55101520253035404550556065707580859095100
    66121824303642485460667278849096102108114120
    7714212835424956637077849198105112119126133140
    881624324048566472808896104112120128136144152160
    9918273645546372819099108117126135144153162171180
    10102030405060708090100110120130140150160170180190200
    11112233445566778899110121132143154165176187198209220
    121224364860728496108120132144156168180192204216228240
    1313263952657891104117130143156169182195208221234247260
    1414284256708498112126140154168182196210224238252266280
    15153045607590105120135150165180195210225240255270285300
    16163248648096112128144160176192208224240256272288304320
    171734516885102119136153170187204221238255272289306323340
    181836547290108126144162180198216234252270288306324342360
    191938577695114133152171190209228247266285304323342361380
    2020406080100120140160180200220240260280300320340360380400

    Как умножать числа столбиком (видео по математике)

    Чтобы потренироваться и быстро выучить, можно также попробовать умножать числа столбиком.

    Нужно распечатать таблицу умножения? Просто нажмите на ссылку печать таблицы умножения. Либо скопируйте картинку (первая таблица) в Ворд (Microsoft Office Word) и распечатайте с помощью сочетания клавиш Ctrl+P. Смотрите также таблицу квадратов.

    Всё для учебы » Математика в школе » Таблица умножения и игра, чтобы быстро выучить

    «3000 примеров по математике. 3 класс. Табличное умножение и деление» Узорова Ольга Васильевна, Нефедова Елена Алексеевна — описание книги | 3000 примеров для начальной школы

    Алтайский край

    Альметьевск

    Амурская область

    Ангарск

    Астрахань

    Белгород

    Богучар

    Братск

    Брянск

    Владивосток

    Владимирская область

    Волгоград

    Волгоградская область

    Воронеж

    Воронежская область

    Грозный

    Губкин

    Екатеринбург

    Ивановская область

    Иркутск

    Кабардино-Балкарская Республика

    Калач

    Калужская

    Кемерово

    Кемеровская область

    Киров

    Краснодарский край

    Красноярск

    Красноярский край

    Курганская

    Курск

    Липецк

    Москва

    Московская область

    Нижегородская область

    Нижний Новгород

    Нижний Тагил

    Новосибирск

    Новосибирская область

    Омск

    Оренбург

    Оренбургская область

    Орловская область

    Пенза

    Пермь

    Поворино

    Республика Адыгея

    Республика Башкортостан

    Республика Бурятия

    Республика Крым

    Республика Мордовия

    Республика Северная Осетия — Алания

    Республика Татарстан

    Республика Хакасия

    Россошь

    Ростов-на-Дону

    Ростовская область

    Рязань

    Самара

    Самарская область

    Саратов

    Свердловская область

    Севастополь

    Смоленск

    Ставрополь

    Ставропольский край

    Старый Оскол

    Тамбовская область

    Томск

    Тула

    Тулун

    Тюмень

    Улан‑Удэ

    Ульяновск

    Ульяновская область

    Хабаровск

    Ханты-Мансийский автономный округ

    Челябинск

    Челябинская область

    Чита

    Чувашская Республика

    Энгельс

    Ярославль

    Примеры умножение и деление на 3 распечатать.

    Умножение. Деньги и мышление миллионера

    С лучшей бесплатной игрой учится очень быстро. Проверьте это сами!

    Учить таблицу умножения — игра

    Попробуйте нашу обучающую электронную игру. Используя её, вы уже завтра сможете решать математические задачи в классе у доски без ответов, не прибегая к табличке, чтобы умножить числа. Стоит только начать играть, и уже минут через 40 будет отличный результат. А для закрепления результата тренируйтесь несколько раз, не забывая о перерывах. В идеале – каждый день (сохраните страницу, чтобы не потерять). Игровая форма тренажера подходит как для мальчиков, так и для девочек.

    Смотрите ниже шпаргалки в полной форме.


    Умножение прямо на сайте (онлайн)

    *

    Таблица умножения (числа от 1 до 20)
    ×1234567891011121314151617181920
    11234567891011121314151617181920
    2246810121416182022242628303234363840
    33691215182124273033363942454851545760
    448121620242832364044485256606468727680
    55101520253035404550556065707580859095100
    66121824303642485460667278849096102108114120
    7714212835424956637077849198105112119126133140
    881624324048566472808896104112120128136144152160
    9918273645546372819099108117126135144153162171180
    10102030405060708090100110120130140150160170180190200
    11112233445566778899110121132143154165176187198209220
    121224364860728496108120132144156168180192204216228240
    1313263952657891104117130143156169182195208221234247260
    1414284256708498112126140154168182196210224238252266280
    15153045607590105120135150165180195210225240255270285300
    16163248648096112128144160176192208224240256272288304320
    171734516885102119136153170187204221238255272289306323340
    181836547290108126144162180198216234252270288306324342360
    191938577695114133152171190209228247266285304323342361380
    2020406080100120140160180200220240260280300320340360380400

    Как умножать числа столбиком (видео по математике)

    Чтобы потренироваться и быстро выучить, можно также попробовать умножать числа столбиком.

    Ни для кого не секрет, как важно знание таблицы умножения и деления, в частности при выполнении арифметических расчётов и решении примеров по математике .

    Однако, что если ребёнка пугает этот огромный набор цифр, именующийся «Таблицей умножения и деления », а уж знать его наизусть, представляется совсем непосильной задачей?

    Тогда спешим успокоить – Выучить всю таблицу умножения очень просто! Для этого необходимо запомнить всего лишь 36 комбинаций чисел (связки трех чисел) . Здесь мы не учитываем умножение на 1 и 10, так как это является элементарным действием не требующим особых усилий в запоминании.

    Описание работы онлайн тренажера

    Данный тренажер работает на основе специально разработанного алгоритма повышения сложности примеров: начиная с самых простых цифр «2 x 2», постепенно повышая сложность до «9 x 9». Тем самым плавно завлекая в процесс изучения.

    Таким образом, запоминать таблицу умножения придётся небольшими порциями, что существенно снизит нагрузку, так как дети будут направлять своё внимание всего лишь на несколько примеров, забыв про весь «большой» объём.

    В Тренажере есть меню настроек для выбора режима изучения таблицы. Имеется возможность выбора дейстия — «Умножение» или «Деление», диапазона примеров «Вся таблица» или «На какое-то число». Все это является рассширенным функционалом сайта и доступно после оплаты .

    Каждый новый пример сопровождается справочной подсказкой , так ребёнку будет легче начать своё изучение и запоминать новые неизвестные ему комбинации.

    Если же по ходу обучения, какой либо пример вызывает трудность, можно быстро напомнить себе его результат, воспользовавшись дополнительной подсказкой , это поможет эффективнее справляться с запоминанием трудных примеров.

    Процентная шкала быстро даст вам понять каким уровнем знания таблицы умножения Вы обладаете.

    Пример считается полностью выученным, если правильный ответ был дан 4 раза подряд . Однако при достижении 100% , призываем не бросать изучение, а вернуться на следующий день и освежить свои знания, повторно пройдя все примеры. Ведь именно регулярные занятия развивают память и закрепляют навыки!

    Описание интерфейса онлайн тренажера

    Во-первых, в тренажере присутствует «панель быстрого доступа», включающая в себя 4 кнопки. Они позволяют: перейти на главную страницу сайта, включить или отключить звуковые сигналы, сбросить результаты обучения (начать изучение сначала), а также попать на страницу отзывов и комментариев.

    Во-вторых, это основная структура программы.

    Выше всех находится процентная шкала , отобржающая примерный уровень знания таблицы умножения.

    Ниже идет поле с примером , на который необходимо дать ответ. Во время ответа оно будет изменять свой цвет: станет красным — если был дан неверный ответ, зеленым — в случае правильного, голубым — после использования подсказки, и желтоватым — во время показа нового примера.

    Следом располагается строка сообщений . В ней выводятся текстовая информация об ошибках, правильных ответах, а также справочной и дополнительной подсказками.

    В конце находится экранная клавиатура , содержащая только необходимые для работы кнопки: все цифры, «забой» — если нужно исправить ответ, кнопки «Проверить» и «Дополнительная подсказка».

    Мы уверены, что данный тренажер «Таблица умножения за 20 минут», поможет .

    И умножение. Как раз об операции умножения и пойдет речь в этой статье.

    Умножение чисел

    Умножение чисел осваивается детьми во втором классе, и ничего в этом сложного нет. Сейчас мы рассмотрим умножение на примерах.

    Пример 2*5 . Это значит либо 2+2+2+2+2, либо 5+5. Берем 5 два раза или 2 пять раз. Ответ, соответственно, 10.

    Пример 4*3 . Аналогично, 4+4+4 или 3+3+3+3. Три раза по 4 или четыре раза по 3. Ответ 12.

    Пример 5*3 . Делаем так же как и предыдущие примеры. 5+5+5 или 3+3+3+3+3. Ответ 15.

    Формулы умножения

    Умножение – это сумма одинаковых чисел, например, 2 * 5 = 2 + 2 + 2 + 2 + 2 или 2 * 5 = 5 + 5. 2)

    Запишитесь на курс «Ускоряем устный счет, НЕ ментальная арифметика», чтобы научиться быстро и правильно складывать, вычитать, умножать, делить, возводить числа в квадрат и даже извлекать корни. За 30 дней вы научитесь использовать легкие приемы для упрощения арифметических операций. В каждом уроке новые приемы, понятные примеры и полезные задания.

    Умножение дробей

    Рассматривая сложение и вычитание дробей, прозвучало правило, приведения дробей к общему знаменателю, чтобы выполнить расчет. При умножении этого делать не надо ! При умножении двух дробей, умножается знаменатель на знаменатель, а числитель на числитель.

    Например, (2/5) * (3 * 4). Умножим две трети на одну четверть. Умножаем знаменатель на знаменатель, а числитель на числитель: (2 * 3)/(5 * 4), тогда 6/20, совершаем сокращение, получаем 3/10.

    Умножение 2 класс

    Второй класс – это только начала изучения умножения, поэтому второклассники решают простейшие задачки на замену сложения умножением, умножают числа, учат таблицу умножения.Давайте рассмотрим задачи на умножение уровня второго класса:

      Олег живет в пяти этажном доме, на самом верхнем этаже. Высота одного этажа равняется 2 метрам. Какова высота дома?

      В коробке находятся 10 упаковок с печеньем. В каждой упаковке их 7 штук. Сколько печенья в коробке?

      Миша расставил свои игрушечные машинки в ряд. В каждом ряду их 7, а рядов всего 8. Сколько у Миши машинок?

      В столовой стоят 6 столов, а за каждым столом задвинуты 5 стульев. Сколько стульев в столовой?

      Мама с магазина принесла 3 пакета с апельсинами. В пакетах находятся по 22 апельсина. Сколько апельсиновпринесла мама?

      В саду растет 9 кустов клубники, а на каждом кустике растет 11 ягод. Сколько ягод растет на всех кустиках?

      Рома положил друг за другом 8 деталей трубы, одинакового размера по 2 метра. Какова длина полной трубы?

      В школу родители на первое сентября привезли детей. Приехало 12 машин, в каждой было по 2 ребенка. Сколькодетей привезли родители на этих машинах?

    Умножение 3 класс

    В третьем классе даются уже более серьезные задания. Помимо умножения будет так же проходиться Деление .

    Среди заданий на умножение будет: умножение двузначных чисел, умножение столбиком, замена сложения умножением и наоборот.

    Умножение столбиком:

    Умножение столбиком – самый простой способ перемножить большие числа. Рассмотрим данный метод на примередвух чисел 427 * 36.

    1 шаг . Запишем числа друг под другом, так чтобы 427 было на верху, а 36 внизу, то есть 6 под 7, 3 под 2.

    2 шаг . Умножение начинаем с крайней правой цифры нижнего числа. То есть порядок умножения таков: 6 * 7, 6 * 2, 6 * 4, затем так же с тройкой: 3 * 7, 3 * 2, 3 * 4.

    Итак, умножаем сначала 6 на 7, ответ:42. Записываем так: так как получилось 42, то 4 – десятки, а 2 – единицы, запись происходит аналогично сложению, а значит 2 записываем под шестеркой, а 4 прибавляем к двойке числа 427.

    3 шаг . Затем аналогично делаем с 6 * 2. Ответ: 12. Первый десяток, который прибавляется к четверке числа 427, а второй – единицы. Складываем полученную двойку с четверкой от предыдущего умножения.

    4 шаг . Умножаем 6 на 4. Ответа 24 и прибавляем 1 от предыдущего умножения. Получаем 25.

    Итак, умножив 427 на 6, получился ответ 2562

    ЗАПОМНИТЕ! Результат второго умножения нужно начать записывать под ВТОРОЙ цифрой первого результата!

    5 шаг . Совершаем аналогичные действия с цифрой 3. Получаем ответ умножения 427 * 3=1281

    6 шаг . Затем полученные ответы при умножении складываем и получаем итоговый ответ умножения 427 * 36. Ответ: 15372.

    Умножение 4 класс

    Четвертый класс – это уже умножение только больших чисел. Вычисление выполняются методом умножения в столбик. Метод описан выше доступным языком.

    Например, найти произведение следующих пар чисел:

    1. 988 * 98 =
    2. 99 * 114 =
    3. 17 * 174 =
    4. 164 * 19 =

    Презентация на умножение

    Скачайте презентацию на умножение с простейшими заданиями для второклассников. Презентация поможет детям лучше ориентироваться в этой операции, потому что она составлена красочно и в игровом стиле – в лучшем варианте для обучения ребенка!

    Таблица умножения

    Таблица умножения учится каждым школьником во втором классе. Ее обязан знать каждый!

    Запишитесь на курс «Ускоряем устный счет, НЕ ментальная арифметика», чтобы научиться быстро и правильно складывать, вычитать, умножать, делить, возводить числа в квадрат и даже извлекать корни. За 30 дней вы научитесь использовать легкие приемы для упрощения арифметических операций. В каждом уроке новые приемы, понятные примеры и полезные задания.

    Примеры на умножение

    Умножение на однозначное
    1. 9 * 5 =
    2. 9 * 8 =
    3. 8 * 4 =
    4. 3 * 9 =
    5. 7 * 4 =
    6. 9 * 5 =
    7. 8 * 8 =
    8. 6 * 9 =
    9. 6 * 7 =
    10. 9 * 2 =
    11. 8 * 5 =
    12. 3 * 6 =
    Умножение на двузначное
    1. 4 * 16 =
    2. 11 * 6 =
    3. 24 * 3 =
    4. 9 * 19 =
    5. 16 * 8 =
    6. 27 * 5 =
    7. 4 * 31 =
    8. 17 * 5 =
    9. 28 * 2 =
    10. 12 * 9 =
    Умножение двузначное на двузначное
    1. 24 * 16 =
    2. 14 * 17 =
    3. 19 * 31 =
    4. 18 * 18 =
    5. 10 * 15 =
    6. 15 * 40 =
    7. 31 * 27 =
    8. 23 * 25 =
    9. 17 * 13 =
    Умножение трехзначных чисел
    1. 630 * 50 =
    2. 123 * 8 =
    3. 201 * 18 =
    4. 282 * 72 =
    5. 96 * 660 =
    6. 910 * 7 =
    7. 428 * 37 =
    8. 920 * 14 =

    Игры на развитие устного счета

    Специальные развивающие игры разработанные при участии российских ученых из Сколково помогут улучшить навыки устного счета в интересной игровой форме.

    Игра «Быстрый счет»

    Игра «быстрый счет» поможет вам усовершенствовать свое мышление . Суть игры в том, что на представленной вам картинке, потребуется выбрать ответ «да» или «нет» на вопрос «есть ли 5 одинаковых фруктов?». Идите за своей целью, а поможет вам в этом данная игра.

    Игра «Математические матрицы»

    «Математические матрицы» великолепное упражнение для мозга детей , которое поможет вам развить его мыслительную работу, устный счет, быстрый поиск нужных компонентов, внимательность. Суть игры заключается в том, что игроку предстоит из предложенных 16 чисел найти такую пару, которая в сумме даст данное число, например на картинке ниже данное число «29», а искомая пара «5» и «24».

    Игра «Числовой охват»

    Игра «числовой охват» нагрузит вашу память во время занятий с данным упражнением.

    Суть игры – запомнить цифру, на запоминание которой отводится около трех секунд. Затем нужно ее воспроизвести. По мере прохождения этапов игры, количество цифр растет, начинаете с двух и далее.

    Игра «Угадай операцию»

    Игра «Угадай операцию» развивает мышление и память. Главная суть игры надо выбрать математический знак, чтобы равенство было верным. На экране даны примеры, посмотрите внимательно и поставьте нужный знак «+» или «-», так чтобы равенство было верным. Знак «+» и «-» расположены внизу на картинке, выберите нужный знак и нажмите на нужную кнопку. Если вы ответили правильно, вы набираете очки и продолжаете играть дальше.

    Игра «Упрощение»

    Игра «Упрощение» развивает мышление и память. Главная суть игры надо быстро выполнить математическую операцию. На экране нарисован ученик у доски, и дано математическое действие, ученику надо посчитать этот пример и написать ответ. Внизу даны три ответа, посчитайте и нажмите нужное вам число с помощью мышки. Если вы ответили правильно, вы набираете очки и продолжаете играть дальше.

    Игра «Быстрое сложение»

    Игра «Быстрое сложение» развивает мышление и память. Главная суть игры выбирать цифры, сумма которых равна заданной цифре. В этой игре дана матрица от одного до шестнадцати. Над матрицей написано заданное число, надо выбрать цифры в матрице так, чтобы сумма этих цифр была равна заданной цифре. Если вы ответили правильно, вы набираете очки и продолжаете играть дальше.

    Игра «Визуальная геометрия»

    Игра «Визуальная геометрия» развивает мышление и память. Главная суть игры быстро считать количество закрашенных объектов и выбрать его из списка ответов. В этой игре на экране на несколько секунд показываются синие квадратики, их надо быстро посчитать, потом они закрываются. Снизу под таблицей написаны четыре числа, надо выбрать одно правильное число и нажать на него с помощью мышки. Если вы ответили правильно, вы набираете очки и продолжаете играть дальше.

    Игра «Математические сравнения»

    Игра «Математические сравнения» развивает мышление и память. Главная суть игры сравнить числа и математические операции. В этой игре надо сравнить два числа. На верху, написан вопрос, прочитайте его и ответьте правильно на поставленный вопрос. Ответить можно при помощи кнопок расположенных внизу. Там нарисованы три кнопки «левое», «равно» и «правое». Если вы ответили правильно, вы набираете очки и продолжаете играть дальше.

    Развитие феноменального устного счета

    Мы рассмотрели лишь верхушку айсберга, чтобы понять математику лучше — записывайтесь на наш курс: Ускоряем устный счет.

    Из курса вы не просто узнаете десятки приемов для упрощенного и быстрого умножения, сложения, умножения, деления, высчитывания процентов, но и отработаете их в специальных заданиях и развивающих играх! Устный счет тоже требует много внимания и концентрации, которые активно тренируются при решении интересных задач.

    Скорочтение за 30 дней

    Увеличьте скорость чтения в 2-3 раза за 30 дней. Со 150-200 до 300-600 слов в минуту или с 400 до 800-1200 слов в минуту. В курсе используются традиционные упражнения для развития скорочтения, техники ускоряющие работу мозга, методика прогрессивного увеличения скорости чтения, разбирается психология скорочтения и вопросы участников курса. Подходит детям и взрослым, читающим до 5000 слов в минуту.

    Секреты фитнеса мозга, тренируем память, внимание, мышление, счет

    Мозгу, как и телу нужен фитнес. Физические упражнения укрепляют тело, умственные развивают мозг. 30 дней полезных упражнений и развивающих игр на развитие памяти, концентрации внимания, сообразительности и скорочтения укрепят мозг, превратив его в крепкий орешек.

    Деньги и мышление миллионера

    Почему бывают проблемы с деньгами? В этом курсе мы подробно ответим на этот вопрос, заглянем вглубь проблемы, рассмотрим наши взаимоотношения с деньгами с психологической, экономической и эмоциональных точек зрения. Из курса Вы узнаете, что нужно делать, чтобы решить все свои финансовые проблемы, начать накапливать деньги и в дальнейшем инвестировать их.

    Знание психологии денег и способов работы с ними делает человека миллионером. 80% людей при увеличении доходов берут больше кредитов, становясь еще беднее. С другой стороны миллионеры, которые всего добились сами, снова заработают миллионы через 3-5 лет, если начнут с нуля. Этот курс учит грамотному распределению доходов и уменьшению расходов, мотивирует учиться и добиваться целей, учит вкладывать деньги и распознавать лохотрон.

    Тема: Таблица умножения и деления на 2. (Урок закрепления)

    Цель: закрепление вычислительных навыков таблицы умножения и деления.

    Задачи урока:

    1. Закрепить знания таблицы умножения и деления; отрабатывать умение решать составные задачи; продолжать формировать вычислительные навыки.

    2. Развивать логическое и экономическое мышление; умение делать выводы, обобщать.

    3. Работая в группах, воспитывать такие качества личности, как сотрудничество, взаимовыручка, толерантность; уважение к труду и людям труда.

    Тип урока : урок совершенствования и закрепления навыков.

    Ход урока.

    1. Оргмомент. Психологический настрой учащихся.

    Прозвенел звонок, начинается урок.

    Ребята, представьте себе, что ваши ладошки- это маленькое зеркальце, посмотрите в него, улыбнитесь себе- вы видите, какие вы симпатичные и умные! Посмотрите друг на друга, улыбнитесь, и ваше настроение будет бодрым и приподнятым, вам захочется узнавать новое, ведь это так интересно!

    Жил мудрец, который знал всё. Один человек решил доказать, что мудрец знает не всё. Зажав в ладонях бабочку, он спросил: «Скажите, мудрец, какая бабочка у меня в руках: мёртвая или живая?» А сам думает: «Скажет живая — я ее умертвлю, скажет мёртвая — выпущу». Мудрец, подумав, ответил: «Все в твоих руках».

    Ваши знания тоже в ваших руках. Давайте мы это и докажем своей работой на уроке.

    (Слайд 1)

    II. Актуализация опорных знаний.

    Чтобы работать быстро и ловко

    Нам нужна для ума тренировка.

    а) Какое число лишнее? (Слайд 2)

    Какое задание нужно выполнить с числами? (Убрать лишнее число)

    7 14 21 27 28 35 42 49

    5 10 11 15 20 25 30 35

    4 8 12 16 17 20 24 28

    Знание чего вам понадобились, чтобы выполнить задание? (Талицы умножения)

    Оценивание.

    б) Назови слово.

    Я предлагаю вам по вопросам узнать тему сегодняшнего урока.

    1. Действие, которым можно заменить сумму одинаковых слагаемых (умножение)

    2. Число, на которое делят (делитель)

    3. Число, которое делят (делимое)

    4. Результат действия при умножении (произведение)

    5. Результат действия при делении (частное)

    6. Компонент действия умножения (множитель)

    Слайд 3. Оценивание.

    III. Самостоятельное формулирование темы и цели урока. Целевая установка на урок.

    Кто догадался, какая тема урока?

    Таблица умножения и деления.

    Ребята, какую цель поставим перед собой?

    Слайд 4

    Сегодня закрепим знание таблицы умножения и деления, будем применять таблицу для решения задач, уравнений, нахождения значения выражения.

    Проблемный вопрос.

    А как вы думаете, можно ли, повторяя и закрепляя, узнать что-то новое? Нам надо разобраться.

    4. Устный счет

    1. Постановка проблемы. Загадка.

    Чтобы узнать, о чем сегодня будет идти речь, вам надо будет отгадать русскую народную загадку “Лежит кучка поросят, кто ни тронет — завизжат”. Сомневаетесь в ответе? А мы сейчас решим эту проблему, выполнив вычисления.

    Слайд 5

    Что перед нами? (блок-схема)

    Как мы будем выполнять вычисления? (по алгоритму)

    Что такое алгоритм? (выполнение действий по порядку)

    Записанные числа 13, 4, 8, 17, 5 записать в порядке возрастания (4, 5, 8, 13, 17)

    Слайд 6

    Какое слово получилось? (пчёлы)

    О ком ещё будем говорить на уроке?

    Оценивание.

    Слайд 7

    Ребята, пчёлы — неутомимые труженики. А отрасль с/х — пчеловодство. Чем занимается эта отрасль? (разводом пчел)

    Человек, какой профессии занимается разводом пчел? (пчеловод).

    Ребята, а есть ли у вас в селе пчеловод?

    Как вы думаете, всё ли знает он о пчёлах? (да)

    Главное в этой профессии, что пчеловод должен знать всё о пчёлах.

    А что вы знаете о пчёлах?

    К сожалению все о пчелах мы знать не можем, но постараемся узнать как можно больше. Я уверена, что у вас все получится.

    Сегодня одна из пчел будет сопровождать нас на уроке. Итак, в путь за пчелой.

    Работа в парах. Нахождение значения выражений с переменными.

    Наша дорога начинается от улья. На пасеке обычно находится много ульев. В каждом улье есть свой вход — леток. Для того, чтобы открыть леток, нам нужно выполнить задание. Какую цель мы поставим выполняя это задание? (выполнить выражения переменной) -Что такое выражение с переменной?

    Оценивание. Взаимопроверка и самопроверка по эталону.

    Слайд 8

    Вы замечательно знаете таблицу умножения и деления, леток в ульях открыт и не случайно наши ульи оказались именно таких цветов. (Желтый, синий, белый). Других цветов пчела просто не различает. Но зато она видит ультрафиолетовые лучи, которые нашим глазам неподвластны.

    IV. Логическая задача.

    А знаете ли вы, сколько глаз у пчелы? (нет)

    Давайте устно посчитаем.

    У пчелы столько глаз, сколько у тебя, еще раз столько, да еще полстолька. (У пчелы 5 глаз. 2 больших, состоящих в свою очередь из 10 тыс глазков, и расположенных по бокам головы и 3 маленьких на лбу между ними)

    V. Работа над закреплением пройденного материала.

    1. Математический диктант. Работа в тетрадях.

    Пчеловоды ульям на пасеке обычно присваивают свои номера. Такие номера есть и на нашей пасеке. — Но мы их узнаем, когда выполним задание. Записать только ответы.

    1) Произведение чисел 2 и 4

    2)Увеличь 2 в 9 раза

    3) Во сколько раз 14 больше 2

    4)1 множитель 2, второй такой же. Произведение?

    5)Уменьши 20 в 2 раз

    6)Какое число уменьшили в 2 раза, если получили 5

    7)На сколько умножили 8, если получили 16

    Слайд 9

    8 18 7 4 10 10 2

    Оценивание. Взаимопроверка со слайда.

    2. Выступление о пчёлах. (Рубан Ваня.)

    Здравствуйте, ребята! Я рабочая пчела. Мы производим воск, прополис, ценнейшее лекарство — мед и пергу. Перга — это пчелиный хлеб из пыльцы и нектара. Его едим мы, пчелы.

    А что вы знаете про пчелиную семью? (Главная в пчелиной семье — матка — она королева. Остальные пчелы рабочие. Они выполняют работу сторожей, чистильщиков ячеек, вентиляторщиков, сборщиков нектара, строителей ячеек. Живут с ними и трутни, которые ничего не делают, но нужны для продолжения рода.)

    3. Запись выражений и нахождение их значений. Слайд 10

    Пчеле пора на работу. Во сколько начинается рабочий день ученика? (8 час) Как вы определяете время? (по часам)

    Пчела хорошо ориентируется во времени. Для этого ей не нужны ни часы, ни солнце. Ей необходимы цветы. Она вылетает тогда, когда начинают работать цветочные часы.

    Как вы понимаете мои слова?
    Вот и мы поработаем с цветами и найдем значения выражений. Первое число в математическом выражении показывает время, когда цветок “просыпается”, найденный вами ответ — когда “засыпает”.

    Что важно знать, чтобы выполнить это задание? (порядок действий)

    Шиповник 2*7-10:2=

    Мак 5+ 7*2 — 11=

    Оценивание. Взаимопроверка.

    4. Задание на нахождение периметра прямоугольника. Слайд 11

    Что мы видим на слайде? (рамка)

    Для чего она нужна пчеловоду?

    Какую работу мы можем выполнить? (найти стороны и периметр прямоугольника).

    S — 12 дм 2

    Длина — 3 дм

    Какие формулы помогли?

    Формулы нахождения периметра, площади.

    Что ёщё помогло?

    Таблица умножения и деления.

    5. Дифференцированная работа.

    Работа по учебнику № 2 (сильные учащиеся) Взаимопроверка.

    Работа по карточкам (слабые учащиеся) Самопроверка.

    5. Работа над задачей. (Карточки)

    Пчёлы-такие труженицы! И мы решим о них задачу.

    Прочитайте задачу, к ней есть несколько вариантов решения. Нужно выбрать одно правильное решение, пометить его плюсом. Объяснить свой выбор.

    Задача . С одного улья дядя Витя выкачал 7 кг меда, а с другого в 2 раза больше. Сколько всего кг меда дядя Витя выкачал с двух ульев?

    Слайд 12

    VII. Итог урока.

    Наш урок подходит к концу. В начале урока я вас спрашивала, можно ли на уроке повторения и закрепления узнать что-то новое. К какому выводу вы пришли?

    Что нового вы узнали на уроке? (отрасль — пчеловодство, профессия — пчеловод. Чем больше пчел вылетит на работу, тем больший урожай мы соберем, тем краше будет наша Земля от благоухающих цветов.)- Чему учились?

    Наша пчелка вас благодарит за работу.

    Понравилось ли вам сотрудничать, работая в парах, коллективно?

    Вы тоже сегодня трудились, как пчелки, и мне очень понравилось работать вместе с вами.

    3000 примеров по математике (Внетабличное умножение и деление).3-4 классы. Кон — Узорова О.В. | 978-5-17-109382-2

    Стоимость товара может отличаться от указанной на сайте!
    Наличие товара уточняйте в магазине или по телефону указанному ниже.

    г. Воронеж, площадь Ленина, д.4

    8 (473) 277-16-90

    г. Липецк, проспект Победы, 19А

    8 (4742) 22-00-28

    г. Воронеж, ул. Маршака, д.18А

    8 (473) 231-87-02

    г. Липецк, пл.Плеханова, д. 7

    8 (4742) 47-02-53

    г. Богучар, ул. Дзержинского, д.4

    8 (47366) 2-12-90

    г. Воронеж, ул. Г. Лизюкова, д. 66 а

    8 (473) 247-22-55

    г. Поворино, ул.Советская, 87

    8 (47376) 4-28-43

    г. Воронеж, ул. Плехановская, д. 33

    8 (473) 252-57-43

    г. Воронеж, ул. Ленинский проспект д.153

    8 (473) 223-17-02

    г. Воронеж, ул. Хользунова, д. 35

    8 (473) 246-21-08

    г. Россошь, Октябрьская пл., 16б

    8 (47396) 5-29-29

    г. Россошь, пр. Труда, д. 26А

    8 (47396) 5-28-07

    г. Лиски, ул. Коммунистическая, д.7

    8 (47391) 2-22-01

    г. Белгород, Бульвар Народный, 80б

    8 (4722) 42-48-42

    г. Курск, пр. Хрущева, д. 5А

    8 (4712) 51-91-15

    г. Губкин, ул. Дзержинского,д. 115

    8 (47241) 7-35-57

    г.Воронеж, ул. Жилой массив Олимпийский, д.1

    8 (473) 207-10-96

    г. Воронеж, ул. Ростовская, д,58/24 ТЦ «Южный полюс»

    8 (473) 280-22-42

    г. Воронеж, ул. Пушкинская, 2

    8 (473) 300-41-49

    г. Липецк, ул.Стаханова,38 б

    8 (4742) 78-68-01

    г. Курск, ул.Карла Маркса, д.6

    8 (4712) 54-09-50

    г. Старый Оскол, мкр Олимпийский, д. 62

    8 (4725) 39-00-10

    г. Курск, ул. Щепкина, д. 4Б

    8 (4712) 73-31-39

    Открытый урок для 2 класса по теме: «Умножение и деление чисел на 2 и на 3»

    Открытый урок по математике во 2 классе.

    Образовательная программа «Начальная школа 21 века»

    Тема: «Умножение и деление чисел на 2 и на 3»

    Пояснительная записка 

    Учитель

    Технологическая карта урока

    Этап урока

    Гражданцева Светлана Васильевна

    Место работы

    МБОУ «Раздорская СОШ им. Губернатора А.П.Гужвина»

    Должность

    Учитель начальных классов

    Тип урока

    урок закрепления материала (технология деятельностного подхода)

    Цель

    закрепить табличные случаи умножения; отработать вычислительные навыки; умение решать задачи.

    Задачи

    1)      Закрепить знание табличного умножения и деления на 2 и на 3. Развивать вычислительные навыки учащихся.

    2)      Продолжить обучение решению задач с использованием действий умножения и деления.

    3)      Работать над формированием универсальных учебных действий (УУД), повышением познавательной активности обучающихся.

    4)      Воспитывать у учащихся чувство взаимовыручки.

    Формируемые УУД

    Предметные: знать название компонентов и результата действия умножения, понимать связь между умножением и сложением (умножение – есть сумма одинаковых слагаемых), правильно употреблять в речи математические понятия;

    понимать суть арифметических действий – умножения и деления; знать как связаны между собой арифметические действия умножения и деления; табличные случаи умножения и деления на 2 и на 3; различные устные и письменные приемы сложения и вычитания двузначных чисел; отличительные особенности задачи; уметь читать произведения и частные, используя названия компонентов умножения и деления; умножать на 10 и 1; умножать десять на однозначное число; решать задачи и выражения изученных видов, в том числе те, которые решаются умножением и делением.

    Личностные: проявлять положительное отношение к учебному предмету «Математика», осознавать её значение; интерес к учебному материалу, способность к самооценке на основе критерия успешности учебной деятельности.

    Регулятивные: определять тему урока, ставить цель, сохранять её в течение всего урока, выполнять под руководством учителя учебные действия в практической и мыслительной форме, фиксировать в диалоге с учителем в конце урока удовлетворённость/неудовлетворённость своей работой на уроке. Прогнозировать результат решения практической  учебной задачи, оценивать по критериям. Находить и исправлять ошибки, выяснять их причины, намечать путь  исправления.

    Познавательные: ориентироваться в информационном материале учебника, осуществлять поиск необходимой информации (по необходимости совместно с учителем), развитие умений использовать при вычислениях черновик, создавать алгоритм деятельности; логически рассуждать; контролировать и оценивать процесс и результаты деятельности, находить рациональные пути решения, отличать новое от уже известного; добывать новые знания.

    Коммуникативные: планировать учебное сотрудничество с одноклассниками: договариваться о распределении работы между собой и соседом, уметь находить и исправлять ошибки в работе соседа, осуществлять взаимоконтроль  и взаимную помощь, использовать простые речевые средства, включаться в диалог с учителем, уважать другую точку зрения.

    Ресурсы:

    — основные

    — дополнительные

    Учебник «Математика» 2 класс авторы В.Н.Рудницкая и Т.В.Юдачева

    Наглядный (презентация) и раздаточный материал, карточки

    Деятельность учителя

    Деятельность ученика

    Планируемые результаты

    Формируемые УУД

    1. Организационный момент

    Задача:

    Включение детей в деятельность на личностно-значимом уровне.

    — Здравствуйте, дорогие ребята, гости. Какое у вас настроение? (смайлики) Слайд 1.

    -Начинаем урок математики. Давайте настроимся на работу. Предлагаю сказать, под каким девизом мы будем работать? (на экране) Слайды 2 и 3

    Учащиеся настраиваются на работу.

    Девиз: С хорошим настроением принимайся за работу!

    Коммуникативные УУД

    планирование учебного сотрудничества с учителем и сверстниками.

    Познавательные УУД

    построение монологического высказывания.

    Личностные УУД Формирование внутренней позиции школьника на уровне положительного отношения к урокам математики; смыслообразование

    2. Устный счёт

    Задача:

    Актуализация опорных знаний

    Слайд 4. Найдите закономерность и продолжите числовой ряд

    2,4,6,8,10,12,14,16,18,20 (увеличение на 2)

    — На какие группы можно поделить числа? (однозначные, двузначные, круглые)

    -Назовите однозначные числа (2,4,6,8)

    — назовите двузначные и круглые числа (10,12,14,16,18,20)

    — найдите половину числа

    (5,6,7,8,9,10)

    — От каких чисел можно найти треть числа? (6→2, 9→3).

    Слайд 5 — молодцы! Теперь откроем тетрадь и запишем число.

    — Составим примеры на умножение и деление с числами 2 и 3 (2х3=6, 6:2=3, 6:3=2)

    Дают ответы с помощью сигнальных карточек

    Самоконтроль. Работа с листами самооценки.

    Работа в тетради.

    составляют из данных чисел примеры и записывают их, понимают роль каждой цифры в записи примеров; взаимосвязь между действиями умножения и деления.

    Познавательные УУД

    Умение формулировать выводы на основе сравнения, обобщения; проводить классификацию изучаемых объектов.

    Регулятивные УУД:

    контролируют свою деятельность

    Личностные УУД: принимают и осваивают роль обучающегося.

    Понимают важность приобретаемых знаний и умений

    3. Постановка учебной задачи

    Задача:

    Формулирование темы и цели урока.

    — догадались ли вы какова тема нашего урока? (Умножение и деление чисел на 2 и на 3) Слайд 6

    — Давайте вместе подумаем, что нам нужно повторить и чему учиться на уроке. Слайд 7

    — Таковы задачи нашего урока и мы должны их выполнить.

    Высказывают предположения

    Формулируют цель и задачи урока

    Под руководством учителя определяют учебные задачи

    Регулятивные УУД: Целеполагание, планирование.

    Умение проявлять инициативу в учебно-познавательной деятельности.

    Познавательные УУД. Самостоятельно выделять и формулировать познавательные задачи.

    4. Повторение таблицы умножения и деления на 2, на 3.

    Задача:

    Повторение изученного материала.

    1. Игра «Лучший знаток таблицы умножения и деления »

    — Работать будем в группах. Синоним слова помогать – спасать, выручать. Я очень хотела бы, чтобы вы всегда выручали друг друга не только на уроке, но и в жизни.

    Повторение правил работы в группах. Слайд 8

    — Кто из членов вашей команды лучший знаток таблицы? Аплодисменты.

    Вывод: — Какие правила (законы умножения) повторили? -Мы выполнили одну из поставленных задач.

    2. Игра «Не скажу».

    -Встаньте, пожалуйста, поиграем в игру «Не скажу». Ведущий называет пример и подаёт мяч кому-то из детей. Стараемся ответить и передать мяч водящему очень быстро, он горячий.

    3. Игра-тест «Помоги львенку найти друзей» Слайды 9-21

    -Выручите Львенка, помогите ему собрать друзей. А вместе с тем повторим таблицу деления на 3.

    — Молодцы! У Львенка много друзей.

    Вывод:- Что мы повторили и закрепили (таблицу деления на 3)

    Мы справились ещё с одной задачей урока.

    Учащиеся разбиваются на три группы. Повторяют правила работы в группах. Работа по карточкам.

    Проверка 2 -3 человек с группы (сигнальные карточки)

    Ответы учащихся

    Переместительный закон, умножение на 0 и на 1, деление числа само на себя, на 0 делить нельзя.

    Учащиеся передают мяч друг другу, называя табличные случаи умножения или деления. Следующий учащийся не называет ответ, а составляет свой пример начиная с ответа предыдущего.

    Например: 3х2→6х3→18:2→9х3 и т.д.

    Работа на интерактивной доске. Один учащийся, у доски решив пример, выбирает друга. Остальные дают ответы с помощью сигнальных карточек.

    Познавательные УУД. Умение кодировать информацию в знаково — символической форме. Проявлять инициативу в учебно- познавательной деятельности.

    Регулятивные УУД

    Контроль и оценка процесса и результатов деятельности. Уметь вносить необходимые коррективы в действие после его завершения на основе его оценки и учёта характера сделанных ошибок .

    Коммуникативные УУД

    Уметь оформлять свои мысли в устной и письменной форме; Принимать активное участие в работе группами. слушать и понимать речь других.

    Личностные УУД

    Контролировать свои действия в коллективной работе. Принимать учебную задачу и следовать инструкции учителя.

    Ориентация на понимание оценок учителя и одноклассников.

    5. Проверка знания таблицы умножения и деления на 2, на 3.

    Задача:

    Проверить умения в нахождении значений произведений и частного, самоконтроль усвоения знаний.

    — А сейчас мы проверим свои знания.

    Математический диктант

    — 2 умножить на 7

    — по 3 взять 5 раз

    — Запишите произведение чисел 5 и 2

    — 3 разделить на 3

    — первый множитель – 3, второй множитель – 9, запишите произведение

    — 12 разделить на 2

    — чему равно произведение чисел 2 и 0?

    -Поменялись тетрадями по кругу, взаимопроверка, проверяем работу соседа.

    -Правильные ответы на доске: 14,15, 10, 1, 27, 6, 0.

    -Поменялись тетрадями опять по кругу, но в обратном направлении.

    Вывод: -Что мы проверили? (знание таблицы умножения и деления на 2,

    на 3).

    -Результатами я осталась довольна, спасибо, вы старались.

     

    Один ученик работает у доски. Остальные самостоятельно работают в тетрадях. Самоконтроль с интерактивной доской. Слайд 22

    Оценивание и занесение результатов оценивания в лист самоконтроля (высокий, средний, низкий)

    Ответы учащихся.

    Познавательные УУД

    — знание компонентов действия умножения и деления;

    -знание таблицы умножения и деления

    на 2, на 3.

    Регулятивные УУД

    прогнозируют результаты собственной деятельности, контролируют и оценивают себя.

    Коммуникативные УУД

    Уметь оформлять свои мысли в устной и письменной форме; слушать и понимать речь других.

    Личностные УУД

    Контролировать свои действия в коллективной работе. Принимать учебную задачу и следовать инструкции учителя.

    Ориентация на понимание оценок учителя и одноклассников. Понимание чувств одноклассников.

    6. Физминутка

    Слайд 23 – 42. Видеоролик. Я предлагаю отдохнуть.

    Встают с мест. Выполняют танцевальные движения.

    7. Решение задач с использованием действий умножения и деления.

    Задача:

    Проверить умение решать задачи на умножение и деление.

    Работа в группах.

    — Посовещайтесь в группе, каким действием решается задача? Выберите карточку с верным решением.

    1)Задача №1. Слайд 43

    2)Задача №2. Слайд 45

    Что такое неделя? Почему неделю так назвали? (7 дней. Особым почетом в древности была окружена семерка. Отголоски почитания числа 7 дошли до наших дней. Вспомните пословицы «Семеро одного не ждут», «Семь бед – один ответ. Наша неделя состоит из 7 дней. Древние заметили, что 7 нельзя поделить на равные части. Вот и назвали 7 не-деля. Неделя – образовано от словосочетания «не делать», то есть отдыхать. Неделя – означало «день отдыха»

    3)Задача №3. Слайд 47.

    -Что узнаем 1 действием? (сколько съели)

    — Что узнаем 2 действием? (сколько яблок было)

    4)Работа по учебнику. Задание №18 стр.102

    — Из предложенных задач выбери и реши только задачу на деление.

    5) Задание №21 с 111.

    6) Немое кино. Вы ничего не услышите, а все, что увидите, считайте.

    Расскажите, что вы увидели? (Мама купила 14 конфет. Дома она дала трём детям по 2 конфеты.)

    — Можно ли это назвать задачей? Почему? Чего не хватает? (вопроса)

    -Поставьте к задаче вопрос.

    (Сколько конфет осталось у мамы?)

    (Анализ задачи:

    Чтобы узнать сколько конфет осталось у мамы, нужно знать сколько конфет мама купила и сколько она отдала детям. Сколько конфет купили известно, а сколько отдали мы не знаем, но знаем, что трём детям по 2 конфеты, а значит можем узнать.

    -Обсудите план решения в группах.

    -Что узнаем 1 действием?

    ( Сколько конфет мама отдала.)

    -Что узнаем 1 действием?

    ( Сколько конфет осталось у мамы.)

    — Один из вас за доской будет решать задачу, а остальные решат её самостоятельно.

    1) 2*3=6(к.) – отдала.

    2) 14-6=8(к.)- осталось.

    -Самопроверка. Сверьте своё решение с решением ученика на доске.

    -А теперь проверим, сколько конфет лежит в пакете. Вывод?

    (Задача решена верно)

    -Чему мы учились?

    (Решать задачи)

    Вывод: Мы выполнили еще одну задачу, которую поставили в начале урока. Значит все задачи выполнены. Молодцы!

    Выбирают карточку с правильным решением, обосновывают свой выбор.

    Самоконтроль с доской. Слайд 44

    Самоконтроль с доской. Слайд 46

    Самостоятельно в паре решают задачу №3 с последующей самопроверкой с доской. Слайд 48

    Оценивание в листе самоконтроля

    Самостоятельная работа в тетради

    Коллективное составление задачи по слайдам. Анализ задачи. Совместная запись условия вместе с учителем. Самостоятельная запись решения.

    Познавательные УУД Формирование интереса к познанию математических фактов.

    Умение осуществлять поиск нужной информации,

    строить небольшие математические сообщения в устной форме. создают алгоритмы деятельности; устанавливают причинно- следственные связи, строят логическую цепочку рассуждений.

    Интерес к различным видам учебной деятельности, включая элементы предметно- исследовательской деятельности. Применять усвоенный способ действий к решению новой задачи.

    Коммуникативные УУД

    Умение стремиться к координации различных мнений в сотрудничестве; умение договариваться, приходить к общему решению.

    8. Подведение итога урока

    Слайд 54. Организация деятельности учащихся по анализу и оцениванию своей деятельности.

    -Молодцы, ребята. Я очень довольна вашей работой на уроке. Подведём итог нашей работы. Нам поможет карточка – помощница.

     (Дети читают начало в карточке и сами заканчивают мысль)

     На уроке я повторил…      таблицу умножения и деления на 2, на 3.

    Я учился…                     решать задачи.

    На уроке мне было…                 (интересно, скучно, легко, трудно…)

    Я понял, что…     надо знать таблицу, уметь решать задачи

    Я радовался…      (успехам товарищей, своим успехам, полученным  знаниям)

    Я всегда буду… выручать своих товарищей.

    Отвечают на вопросы, делают выводы, обобщения.

    Познавательные УУД логически рассуждают, строят цепочку умозаключений.

    Регулятивные УУД

    осознают важность полученных знаний; понимают причины успеха и неуспеха.

    Личностные УУД Самооценка на основе заданных критериев успешности учебной деятельности

    9. Домашнее задание

    Инструктаж

    А теперь послушайте домашнее задание на завтра.

    -Всем спасибо. Урок закончен

    Записывают домашнее задание.

    10. Рефлексия.

    Задача:

    Осознание обучающимися своей учебной деятельности, самооценка результатов деятельности своей и всего класса.

    -Поднимите тот смайлик, который выражает ваше настроение после урока.

    -Спасибо за урок.

    Регулятивные УУД

    Уметь оценивать правильность выполнения действия на уровне адекватной ретроспективной оценки.

    Личностные УУД

    Формирование самооценки на основе заданных критериев успешности учебной деятельности . Понимание причин успеха в учёбе.

    Примеры по математике для 3 класса

    Примеры на сложение и вычитание:

    Примеры на сложение и вычитание двузначных чисел

    Сумма не превышает 10

    Примеры на сложение и вычитание трёхзначных чисел

    Сумма не превышает 10

    Примеры на сложение и вычитание в пределах 1000

    Сложение двузначных чисел с суммой не превышащей 100

    Примеры на сложение и вычитание в пределах 10000

    Сложение двузначных чисел с суммой не превышащей 1000

    Примеры с пропусками значений

    Примеры на сложение и вычитание с пропусками двузначных чисел

    Сумма не превышает 10

    Примеры на сложение и вычитание с пропусками в пределах 1000

    Сложение двузначных чисел с суммой не превышащей 1000

    Примеры на сложение и вычитание с пропусками в пределах 10000

    Сложение двузначных чисел с суммой не превышащей 10000

    Сравнения

    Сравнения с примерами с двузначными числами

    Неравенства или сравнения примеров, где сумма не превышает 10

    Сравнения с примерами с трёхзначными числами

    Неравенства или сравнения примеров, где сумма не превышает 10

    Таблица умножения

    Примеры на умножение однозначных чисел

    Сумма не превышает 10

    Примеры на умножение однозначных и двузначных чисел

    Сумма не превышает 10

    Примеры на умножение опорных чисел «12», «15», «25», «75», «125»

    Сумма не превышает 10

     

     

    План урока умножения и деления

    Оценка: 03

    CCSS. Math.Content.3.OA.A.2

    Интерпретировать целые частные целых чисел, например, интерпретировать 56 ÷ 8 как количество объектов в каждой доле, когда 56 объектов разделены поровну на 8 долей, или как количество долей, когда 56 объектов разделены на равные доли по 8 объектов. каждый.Например, опишите контекст, в котором количество акций или групп может быть выражено как 56 ÷ 8.

    Оценка: 03

    CCSS.Math.Content.3.OA.A.3

    Используйте умножение и деление в пределах 100 для решения задач со словами в ситуациях, связанных с равными группами, массивами и измеряемыми величинами, например.g., используя рисунки и уравнения с символом неизвестного числа для представления проблемы.1

    Оценка: 03

    CCSS.Math.Content.3.OA.A.4

    Определите неизвестное целое число в уравнении умножения или деления, связывающего три целых числа.Например, определить неизвестное число, которое делает уравнение истинным в каждом из уравнений 8 ×? = 48, 5 = _ ÷ 3, 6 × 6 =?

    Оценка: 03

    CCSS. Math.Content.3.OA.B.5

    Применяйте свойства операций как стратегии умножения и деления.2 Примеры: если известно 6 × 4 = 24, то также известно 4 × 6 = 24. (Коммутативное свойство умножения.) 3 × 5 × 2 можно найти как 3 × 5 = 15, затем 15 × 2 = 30 или 5 × 2 = 10, затем 3 × 10 = 30. (Ассоциативное свойство умножения. ) Зная, что 8 × 5 = 40 и 8 × 2 = 16, можно найти 8 × 7 как 8 × (5 + 2) = (8 × 5) + (8 × 2) = 40 + 16 = 56. собственность.)

    Оценка: 03

    CCSS.Math.Content.3.OA.B.6

    Поймите разделение как проблему с неизвестным фактором. Например, найдите 32 ÷ 8, найдя число, которое дает 32 при умножении на 8.

    Оценка: 03

    CCSS.Math.Content.3.OA.C.7

    Плавно умножайте и делите в пределах 100, используя такие стратегии, как взаимосвязь между умножением и делением (например, зная, что 8 × 5 = 40, каждый знает, что 40 ÷ 5 = 8) или свойства операций. К концу 3 класса выучить по памяти все произведения двух однозначных чисел.

    Оценка: 03

    CCSS.Math. Content.3.OA.D.8

    Решите двухэтапные задачи со словами, используя четыре операции. Представьте эти проблемы, используя уравнения с буквой, обозначающей неизвестную величину. Оцените разумность ответов с помощью мысленных вычислений и стратегий оценки, включая округление.3

    Оценка: 03

    CCSS.Math.Content.3.OA.D.9

    Определите арифметические шаблоны (включая шаблоны в таблице сложения или таблице умножения) и объясните их, используя свойства операций. Например, заметьте, что четырехкратное число всегда четно, и объясните, почему четырехкратное число можно разложить на два равных слагаемых.

    Класс: 04

    CCSS.Math.Content.4.NBT.B.5

    Умножьте целое число до четырех цифр на однозначное целое число и умножьте два двузначных числа, используя стратегии, основанные на разрядах и свойствах операций. Проиллюстрируйте и объясните расчет с помощью уравнений, прямоугольных массивов и / или моделей площадей.

    Класс: 04

    CCSS.Math.Content.4.NBT.B.6

    Найдите целочисленные частные и остатки с четырехзначными дивидендами и однозначными делителями, используя стратегии, основанные на разряде, свойствах операций и / или взаимосвязи между умножением и делением. Проиллюстрируйте и объясните расчет с помощью уравнений, прямоугольных массивов и / или моделей площадей.

    Класс: 04

    CCSS.Math.Content.4.OA.A.1

    Интерпретируйте уравнение умножения как сравнение, например, интерпретируйте 35 = 5 × 7 как утверждение, что 35 в 5 раз больше 7 и 7 раз больше 5. Представьте словесные утверждения мультипликативных сравнений как уравнения умножения.

    Класс: 04

    CCSS.Math.Content.4.OA.A.2

    Умножайте или делите для решения словесных задач, включающих мультипликативное сравнение, например, используя рисунки и уравнения с символом неизвестного числа, чтобы представить проблему, отличая мультипликативное сравнение от аддитивного сравнения.1

    Класс: 04

    CCSS.Math.Content.4.OA.A.3

    Решите многоступенчатые задачи со словами, поставленные с целыми числами и получив ответы с целыми числами, используя четыре операции, включая задачи, в которых необходимо интерпретировать остатки. Представьте эти проблемы, используя уравнения с буквой, обозначающей неизвестную величину. Оцените разумность ответов с помощью мысленных вычислений и стратегий оценки, включая округление.

    Оценка: 05

    CCSS.Math.Content.5.NBT.B.5

    Умножайте многозначные целые числа с помощью стандартного алгоритма.

    Оценка: 05

    CCSS.Math.Content.5.NBT.B.6

    Находите частные целых чисел с дивидендами до четырех и двузначными делителями, используя стратегии, основанные на разряде, свойствах операций и / или взаимосвязи между умножением и делением.Проиллюстрируйте и объясните расчет с помощью уравнений, прямоугольных массивов и / или моделей площадей.

    Оценка: 05

    CCSS.Math.Content.5.NBT.B.7

    Сложить, вычесть, умножить и разделить десятичные дроби до сотых, используя конкретные модели или чертежи и стратегии, основанные на разряде, свойствах операций и / или соотношении между сложением и вычитанием; свяжите стратегию с письменным методом и объясните используемую аргументацию.

    Умножение и деление чисел в научной записи

    Результат обучения
    • Умножать и делить числа, выраженные в экспоненциальном представлении

    Умножение и деление чисел, выраженных в научной записи

    Числа, записанные в экспоненциальном представлении, можно довольно просто умножать и делить, пользуясь свойствами чисел и правилами экспонент, которые вы, возможно, помните. {-2}} [/ латекс]

    Обратите внимание, что при делении экспоненциальных членов вы вычитаете показатель степени в знаменателе из показателя степени в числителе. В следующем видео вы увидите еще один пример деления чисел, записанных в экспоненциальном представлении.

    Задачи умножения и деления слов

    Задачи со словом весело и сложно решать, потому что они представляют реальные ситуации, происходящие в нашем мире. Как студенты, мы всегда задаемся вопросом, зачем нам изучать тот или иной навык, а задачи со словами помогают нам увидеть практическую ценность того, что мы изучаем.

    Прочтите советы и рекомендации, а затем поработайте вместе с детьми над задачами умножения и деления слов из этого урока. Попробуйте использовать три рабочих листа, перечисленных в уроке (вы также найдете их внизу страницы).

    Решение задач мультипликативного сравнения слов

    Умножение в сравнении

    В задачах мультипликативного сравнения сравниваются два разных набора. Первый набор содержит определенное количество предметов.Второй набор содержит несколько копий первого набора.

    Любые два фактора и их произведение можно рассматривать как сравнение. Давайте посмотрим на основное уравнение умножения: 4 x 2 = 8.

    8 то же самое, что 4 подхода по 2 или 2 подхода по 4.
    8 в 4 раза больше, чем 2, и в 2 раза больше, чем 4.
    Какую операцию использовать: Умножение? Делить? Добавлять? Вычесть?

    Самая сложная часть любой проблемы со словами — это решить, какую операцию использовать.В словесную задачу может входить так много деталей, что задаваемый вопрос теряется во всей ситуации. Очень важно уделить время определению того, что важно, а что нет.

    Используйте маркер для письменных задач, чтобы выделить слова, которые говорят вам, что вы решаете, и подсказки о том, какие операции выбрать. Сделайте пометки на полях рядом с этими словами, чтобы помочь вам прояснить ваше понимание проблемы.

    Помните: если вы не знаете, о чем спрашивают, будет очень трудно узнать, есть ли у вас разумный ответ.

    Различные типы проблем

    Существует три типа задач мультипликативного сравнения слов (см. Список ниже). Знание того, какая проблема стоит перед вами, поможет вам узнать, как ее решить.

    • Сравнение продуктов неизвестно
    • Размер набора неизвестен, сравнение
    • Множитель Неизвестных сравнений

    Остальная часть этого урока покажет, как можно решить эти три типа математических задач.

    Задачи умножения: продукт неизвестен

    В некоторых задачах мультипликативного сравнения слов вам дается количество элементов в одном наборе, и вам дается сумма «множителя».Сумма множителя показывает, во сколько раз (или больше) второй набор больше, чем первый. «Больше» также может означать «длиннее», или «шире», или «выше» в задачах, связанных с измерением, или «быстрее» в задачах, связанных со скоростью.

    Эти задачи, в которых вы знаете и число в одном наборе, и множитель, называются сравнениями «Неизвестный продукт», потому что сумма — это часть, которая неизвестна.

    Чтобы ответить на вопрос, который вам задают, вам нужно умножить число в наборе на множитель, чтобы найти произведение.

    Задачи умножения: продукт неизвестен — пример

    Приведенная ниже проблема включает цветовое кодирование, чтобы помочь проанализировать сравнение неизвестных продуктов. Обратите также внимание на важность полного изложения ответа и проверки того, имеет ли ответ смысл.

    Мэри копит деньги на поездку. За месяц за
    она сэкономила в три раза больше денег, чем в прошлом месяце.
    В прошлом месяце она сэкономила 24 доллара.
    Сколько денег Мэри сэкономила в этом месяце?
    Мэри копит деньги на поездку.В этом месяце она сэкономила в три раза больше денег, чем в прошлом месяце. В прошлом месяце она сэкономила 24 доллара. Сколько денег Мэри сэкономила в этом месяце?
    По крайней мере, говорит вам, что у вас есть сравнение.
    Трехкратный множитель: 24,00 доллара — это сумма в первом наборе.
    Сколько денег Мэри сэкономила в этом месяце? это вопрос, который вам задают.
    Чтобы решить, умножьте 24 доллара на 3.
    24 доллара США x 3 = 72 доллара США.
    Важно четко показать, что вы понимаете, что означает ваш ответ.
    Вместо того, чтобы просто писать 72 доллара, напишите:
    Мэри сэкономила 72 доллара в этом месяце.
    Каждый раз, когда вы заканчиваете математическую задачу любого рода, всегда возвращайтесь к исходной задаче. Подумайте: «Какой вопрос мне задают?»
    Убедитесь, что ваш окончательный ответ является разумным ответом на вопрос, который вам задают.
    Меня спросили: «Сколько денег Мэри сэкономила в этом месяце?»
    Мой ответ: Мэри сэкономила 72 доллара в этом месяце. Мой ответ разумен, потому что он показывает, сколько денег Мэри сэкономила в этом месяце. Я умножил целое число на целое, поэтому сумма денег, сэкономленных Мэри в этом месяце, должна быть больше, чем она сэкономила в прошлом месяце. Семьдесят два — это больше 24. В моем ответе есть смысл.

    Попробуйте решить проблемы со словами на листе ниже (рабочий лист также указан внизу этой страницы)

    Задачи умножения: размер набора неизвестен

    В некоторых задачах мультипликативного сравнения слов неизвестной частью является количество элементов в одном наборе.Вам дается сумма второго набора, которая кратна неизвестному первому набору, и величина «множителя», которая говорит вам, во сколько раз (или больше) второй набор больше, чем первый. Помните, что «больше» также может означать «длиннее», «шире» или «выше» в задачах, связанных с измерением, или «быстрее» в задачах, связанных со скоростью.

    Эти задачи, в которых вы знаете и число во втором наборе, и множитель, называются сравнениями «Неизвестный размер набора», потому что число в одном наборе является частью, которая неизвестна.

    Чтобы ответить на вопрос, который вам задают, вам нужно использовать обратную операцию умножения: деление. Этот вид разделения называется разделением на «разделение» или «разделение». Разделив число во втором наборе на множитель, вы получите число в одном наборе, и это вопрос, который вам задают в такого рода задачах.

    Задачи мультипликативного сравнения: размер набора неизвестен — пример
    Джефф прочитал 12 книг в течение августа.Он прочитал в четыре раза больше книг, чем Пол. Сколько книг прочитал Павел?
    Джефф прочитал 12 книг в течение августа. Он прочитал в четыре раза больше книг, чем Пол. Сколько книг прочитал Павел?
    Столько, сколько говорит вам, что у вас есть сравнение. В четыре раза множитель. 12 книг — это сумма во втором наборе. Сколько книг прочитал Павел? это вопрос, который вам задают. Чтобы решить, разделите 12 на 4.
    12 ÷ 4 = 3 Важно четко показать, что вы понимаете, что означает ваш ответ. Вместо того, чтобы просто написать 3, напишите: Павел прочитал три книги.
    Помните, когда вы заканчиваете математическую задачу со словами, всегда возвращайтесь к исходной задаче. Подумайте: «Какой вопрос мне задают?» Убедитесь, что ваш окончательный ответ является разумным ответом на вопрос, который вам задают. Меня спросили: «Сколько книг прочитал Павел?» Мой ответ: Павел прочитал три книги. Мой ответ разумный, потому что в нем указано, сколько книг прочитал Павел. Я разделил целое число на целое, поэтому количество книг Пола должно быть меньше количества книг Джеффа.Три меньше 12. Мой ответ имеет смысл.

    Попробуйте решить проблемы со словами на листе ниже (рабочий лист также указан внизу этой страницы)

    Задачи мультипликативного сравнения: неизвестный множитель

    В некоторых задачах мультипликативного сравнения слов вам дается количество элементов в одном наборе, и вам дается количество элементов во втором наборе, которое кратно первому набору. Сумма «множителя» — это та часть, которая неизвестна.

    Сумма множителя показывает, во сколько раз (или больше) второй набор больше, чем первый. «Больше» также может означать «длиннее», «шире» или «выше» в задачах, связанных с измерением, или «быстрее» в задачах, связанных со скоростью.

    Эти задачи, в которых вы знаете и число в одном наборе, и число во втором наборе, называются сравнениями «Неизвестный множитель», потому что множитель — это часть, которая неизвестна.

    Чтобы ответить на вопрос, который вам задают, вам нужно использовать обратную операцию умножения: деление.Такое деление называется «измерительным».

    Задачи мультипликативного сравнения: неизвестный множитель — пример
    Рост гориллы в зоопарке Лос-Анджелеса — шесть футов. Рост жирафа — 18 футов. Во сколько раз жираф выше гориллы?
    Рост гориллы в зоопарке Лос-Анджелеса — шесть футов. Рост жирафа — 18 футов. Во сколько раз жираф выше гориллы?
    Выше, чем говорит вам, что у вас есть сравнение. Шесть футов — это количество в первом наборе. 18 футов — это количество во втором наборе. Во сколько раз жираф выше гориллы? это вопрос, который вам задают. Чтобы решить, разделите 18 футов на шесть футов.
    18 ÷ 6 = 3Важно четко показать, что вы понимаете, что означает ваш ответ. Вместо того, чтобы просто писать 3, напишите: Жираф в три раза выше гориллы.
    Помните, когда вы заканчиваете математическую задачу со словами, всегда возвращайтесь к исходной задаче.Подумайте: «Какой вопрос мне задают?» Убедитесь, что ваш окончательный ответ является разумным ответом на вопрос, который вам задают. Меня спросили: «Насколько выше гориллы жираф?» Мой ответ: жираф в три раза выше гориллы. Мой ответ разумен, потому что он говорит о том, насколько жираф выше гориллы. Я разделил целое число на целое, так что мое частное должно быть меньше моего дивиденда. Три меньше 18, поэтому мой ответ имеет смысл.

    Попробуйте решить проблемы со словами на листе ниже (рабочий лист также указан внизу этой страницы)

    Листы мультипликативного сравнения

    Щелкните по ссылкам ниже и попросите вашего ребенка попробовать рабочие листы, которые позволят попрактиковаться в решении математических задач, описанных выше. Первые три из них были перечислены выше в уроке, а четвертый — со смешанными типами — нет.

    Символы, выражения и отношения умножения и деления

    Назначение

    Этот модуль развивает понимание умножения и деления, в том числе обратной связи между двумя операциями, а также того, когда и как их использовать в ситуациях решения проблем.Студенты изучают правила представления операций умножения и деления в виде уравнений.

    Конкретные результаты обучения

    • Прочтите, запишите и поймите символы умножения и деления, знак равенства и язык, связанный с этими символами.
    • Напишите контекст истории для заданных уравнений умножения и деления.
    • Помните, что операция умножения коммутативна.
    • Определите связанные факты умножения и деления («семейства фактов»).
    • Признайте обратную связь между операциями умножения и деления.
    • Помните, что деление не коммутативно.
    • Используйте слова «фактор» и «продукт» надлежащим образом и определите факторы заданных сумм.

    Описание математики

    Эта последовательность уроков устанавливает связь между повторным сложением и умножением. Он вводит деление и исследует взаимосвязь между операциями умножения и деления.

    В рамках этих уроков развиваются три основных понимания.

    1. Учащимся необходимо понимать отношения между величинами, которые представлены уравнениями умножения и деления. Например, 4 x 5 = 20 может означать, что «четыре количества из пяти равны 20» или «20 в четыре раза больше 5».
    2. Учащимся необходимо выучить словарный запас, связанный с умножением и делением, а также значение этих слов.Важный словарь включает факторы (умножаемые числа), произведение (ответ на умножение), умножение на (увеличение одного количества в x раз), равенство (одинаковость количества).
    3. Умножение можно также представить в пространстве. Массивы — это мощный способ показать структуру и шаблон нескольких групп и, в этом случае, прочно увязать умножение и деление с измерением.

    При исследовании структуры и паттерна умножения и деления основное внимание также уделяется раннему пониманию свойств числа .В этих уроках формально исследуется коммутативность умножения. Свойство распределения, в котором один или оба фактора разделяются (например, 12 x 55 = 10 x 55 + 2 x 55), является основополагающим для стратегий вычислений, включая письменные алгоритмы.

    При изучении поведения операций умножения и деления важно, чтобы учащиеся сделали обобщение , в котором они могли бы заявить, «что всегда происходит», когда предпринимаются определенные действия. Например, признание того, что правило «перевернуть» (коммутативное) равно , всегда верно для умножения, но это неверно для деления.

    Эта серия уроков посвящена однозначным множителям и делителям. Он признает, что для построения правильного понимания того, как мы используем символы и выражения умножения и деления для математического мышления и для выражения взаимосвязей, учащиеся должны иметь много возможностей для представления операций для решения текстовых задач. Студенты также должны уметь создавать контексты, которые может выразить уравнение. Установление связей между языком и символами важно для развития правильного понимания математических идей и концепций.

    Ссылки на числовую структуру
    Ранняя добавка (стадия 5)
    Расширенная добавка (стадия 6)

    Возможности адаптации и дифференциации

    Возможности обучения в этом модуле можно дифференцировать путем предоставления или прекращения поддержки учащихся и изменения требований к заданиям. Способы дифференциации включают:

    • Обеспечьте физические материалы, чтобы учащиеся могли предвидеть действия и обосновывать свои решения.Используйте такие материалы, как кубики и квадратные плитки, чтобы моделировать ситуации и связывать стратегии, используемые учащимися, с представленными количествами. Прогресс в создании диаграмм массивов на бумаге в квадрат.
    • Соедините символы и математический словарь, особенно символы для умножения и деления (x, ÷) и для равенства (=). Явно смоделируйте правильное использование уравнений и алгоритмов и обсудите значение символов в контексте.
    • Изменить сложность используемых чисел.Умножение на такие множители, как два, четыре, пять, десять и деление на те же делители, как правило, проще, чем на множители, такие как три, шесть, семь, восемь и девять.
    • Поощряйте студентов к сотрудничеству в небольших группах, а также к тому, чтобы делиться своими идеями и оправдывать их.
    • Используйте технологии, особенно калькуляторы, для прогнозирования, основанных на шаблонах, для оценки продуктов и коэффициентов, например Если ответ на 4 x 8 = 32, ответ на 32 ÷ 5 будет больше или меньше 8? Откуда вы знаете? Разрешите использование калькуляторов там, где вы хотите, чтобы учащиеся больше сосредотачивались на процессе получения разумного ответа или на обнаружении закономерностей, чем на отработке навыков вычислений.

    Контекст, используемый для этого устройства, — лоскутные одеяла и ткань тапа. Вы можете изменить контекст на ситуации, более соответствующие повседневной жизни, интересам или культурной самобытности ваших учеников. Массивы широко распространены в разных культурах и могут быть найдены в узорах плитки, текстиле, упаковках, сваях для домов и игровых досках для игр. Поощряйте учеников проявлять творческий подход, принимая различные стратегии от других и прося учеников создавать свои собственные проблемы для решения другими в значимых контекстах.

    Требуемые ресурсные материалы

    • Как минимум два прямоугольных одеяла или ткань тапа
    • Цветные пластиковые квадратные плитки (или маленькие квадраты разноцветных карточек)
    • Квадратная бумага
    • Калькуляторы
    • Кубы Unifix
    • Карты игральные
    • Первая и вторая точки PowerPoints
    • Один, два и три копировщика

    Деятельность

    Сессия 1

    Деятельность 1

    1. Покажите ученикам два разных прямоугольных лоскутных одеяла.Или используйте PowerPoint One, чтобы показать фотографии подходящих одеял или тапа. Например:

      Предположим, класс собирается изготовить лоскутное одеяло или ткань тапа для детской палаты в местной больнице (или хосписе).
      Вовлеките студентов в обсуждение квилтинга, выясняя, как создаются рисунки.

    2. Спросите: «Что за математика есть в этих одеялах?» (например, одеяло 3 x 3)
      Запишите идеи учащихся в таблицу класса. (Они могут включать число, геометрию, формулировки измерений: например, 3 + 3 + 3 = 9, 3 x 3 = 9, 9 квадратов, один большой квадрат, стороны одинаковой длины, 9, разделенные на 3 и т. Д.). Сравните количество квадратов в разных примерах.
    3. Выделите операцию , взаимосвязь и символов (или слов), которые были записаны. Например:
    4. Напишите каждый символ на отдельном листе бумаги формата А4. Попросите пары учащихся взять один лист (один символ), и каждая по очереди запишет за 2 минуты , используя слова и картинки / диаграммы , — мозговой штурм всего, что они знают об этом символе (или слове). Попросите учащихся привести пример того, где можно использовать их символ.
    5. Попросите учащихся вернуться на коврик, сидя в отдельных двух группах: группе с операцией символов (+ — x ÷) и группой с взаимосвязью символа ( =). Попросите выбранные пары учеников объяснить, почему они сидят там, где они находятся, и какие идеи они записали для своих символов.
      В этом обсуждении выделите используемый язык , , получите представление о том, что такое операция с числами , (математический процесс, который изменяет число или сумму), и просмотрите значение числа , равного знаку.
      Сохраните листы мозгового штурма для использования в будущем.

    Деятельность 2

    1. Подготовьте пакеты из 12, 18, 20, 24 и 30 пластиковых плиток, маленьких цветных квадратов карт или тканевых квадратов. Сделайте их, карандаши и бумагу, доступными для студенческих пар.

      Задайте проблему. «Покажите, с помощью диаграмм и уравнений , сколькими различными способами вы можете расположить эти заплатки, чтобы сделать« мини-лоскутное одеяло »?»
      Попросите учащихся поработать в парах, чтобы записать свои идеи.

    2. Попросите учеников поделиться своими идеями с парой, у которой было одинаковое количество плиток, и запишите все аранжировки, о которых они не думали.
    3. В классе делитесь идеями, исследуйте и записывайте ключевые моменты в таблицу класса. Сохраните эту студенческую работу для Занятия 2.
      Например: Из пакета с 18 «заплатками» (плитками).

      В ходе обсуждения основывайтесь на идеях, изложенных в Задании 1 (выше), выделяя и записывая словами следующие идеи:

      • Аранжировки «патчей» могут быть записаны с использованием различных операций .
      • Умножение
      • с использованием символа x может показать ту же идею как повторное сложение (равных величин) с использованием символа + .
      • Символ для деления или разделения на равные группы: ÷ . Он называется символом деления .
      • Эта структура с равными строками и столбцами называется массивом .
    4. Поза и запись: «9 + 9 = 6 x 3. Вы согласны или не согласны». Попросите пары учащихся обсудить это утверждение и подготовиться к обоснованию своей позиции (объясните, почему они согласны или не согласны, и откуда они знают, что они правы).
      Запишите обоснование учащегося, выделив отношение эквивалентности (оба равны 18, всего 18 патчей в обоих массивах). Выделите мультипликативные представления, такие как «9 равно 3 x 3, поэтому 9 + 9 равно 6 x 3».

    Деятельность 3

    Напишите в таблице классов два уравнения: одно умножение и одно деление.
    Например: 6 x 5 = 30 28 ÷ 4 = 7. Прочтите их вместе. Попросите каждого ученика нарисовать схему лоскутного одеяла или ткани тапа, которая представляет уравнение. Попросите их написать словами, как одеяло / ткань представляет уравнение.

    Действие 4

    Завершите сеанс, рассмотрев символы операций и отношений и их значения.

    Сессия 2

    Деятельность 1

    1. Начните с того, что по крайней мере два ученика поделятся своими схемами лоскутного одеяла / ткани с предыдущего занятия. Попросите других студентов записать уравнения, представленные на схеме. Подчеркните тот факт, что математику из реальной жизни можно представить с помощью диаграмм, слов и символов.
    2. Мозговой штурм на диаграмме класса другие ситуации в нашей жизни, где мы видим и используем умножение или деление. По мере того, как учащиеся делятся идеями, попросите их назвать конкретные числа.Запишите эти истории, используя схемы и слова.
      Например: Мы видим умножение, когда:
      • 12 пакетов по 20 изюмов ​​завернуты в большую пачку — четыре пакета в ряд и три ряда.
      • Вы покупаете три пакета жевательной резинки по десять штук в каждой упаковке
      • Мы составляем четыре команды по шесть человек для спортивной игры по физкультуре.
    3. Прочитайте истории снова вместе. Попросите учащихся использовать символы для записи уравнений для каждой из историй в своих книгах / на доске / бумаге.Те ученики, которые заканчивают быстро, могут придумать больше контекстных историй.
      Попросите студентов поделиться своими уравнениями в парах. Если учащиеся записали, используя повторное сложение, попросите их также записать уравнения умножения.

    Деятельность 2

    1. Просмотрите информацию о символах из сеанса 1, выделив символы операций, + — x ÷, и символы взаимосвязи, равно (=), больше (>) и меньше ( Попросите учащихся поработать в парах, используя ситуации из предыдущего задания.Студенты должны обсудить ситуации и посмотреть, сколько уравнений или неравенств они могут написать, например:
      3 x 4 = 4 x 3
      3 x 4 4 x 6> 2 x 10> 4 x 3
      Они должны использовать диаграммы, чтобы показать, откуда они знают, что они верны.
    2. Попросите учащихся разделить свою работу в паре. При этом они должны по очереди прочитать вслух то, что они написали.

    Деятельность 3

    1. Вернитесь к лоскутным одеялам / тапам (изображения).Объясните, что некоторым маленьким детям нравятся лоскутные одеяла с алфавитом, на каждой нашивке которых изображено что-то, начинающееся с другой буквы алфавита. Поговорите о том, что некоторые из них могут быть. Например: A может изображать яблоко, B — бабочку, C — кошку и так далее.
    2. Раздайте ученикам бумагу, карандаши и фломастеры.
      Задайте задачу: Вы собираетесь сделать лоскутное одеяло / тапа с алфавитом для маленького ребенка. У вас есть до конца сегодняшнего занятия, чтобы спланировать свой дизайн и то, как вы расположите свои «квадратные пятна» .Где-то в проблеме может быть проблема. Вы, , решаете, как лучше всего решить эту проблему для своего дизайна лоскутного одеяла.
      Сколько букв в алфавите? (26)
      Почему сделать квилт из 26 квадратов может быть проблемой?
    3. Предложите учащимся поэкспериментировать с 26 квадратами. Они могут нарисовать возможные варианты использования квадратных плиток или кубиков.
      (26 образуют только массивы 1 x 26 или 2 x 13, что нежелательно для стеганого одеяла такого типа. Учащиеся столкнутся с «остатком» (6 x 4 + 2, 5 x 5 + 1) или найдут это какие-то «заплатки» короткие (7 x 4). Принимайте реалистичные решения для контекста. (например, одеяло 5 x 5: поместите 2 буквы на одном патче, одеяло 6 x 4: сделайте его размером 7 x 4 и включите 2 романа или пустые нашивки.)
    4. Предложите: Если мы добавим патчи для каждой из цифр 0-9, сколько патчей у нас будет тогда? (26 + 10 = 36)
      Посмотри, какие лоскутные одеяла ты тогда сможешь сделать.
      Поищите учащихся, чтобы они нашли все возможные варианты:
      1 x 36 2 x 18 3 x 12 4 x 9 6 x 6
      Какой набор является лучшим стеганым одеялом / тканью тапа? Почему?

    Сессия 3

    Деятельность 1

    1. Попросите учащихся поделиться своими эскизами лоскутных одеял с алфавитом для 36 заплат.Обсудите «оставшуюся проблему» и порекомендуйте творческие решения.
      Почему невозможно было изготовить лоскутное одеяло с пятью заплатами подряд?
      Запишите 36 ÷ 5 = 7 r 1 и спросите учащихся, что означает r 1 (остаток от 1).
      Укажите, что часто проблемы с разделением не решаются равномерно. Мы называем то, что осталось, остатком .
    2. Представьте, что у нас есть 26 патчей, и мы пытаемся разместить по шесть патчей в каждом ряду. Один из способов записать эту проблему — 26 ÷ 6 = 4 r2.
    3. На диаграмме классов быстро нарисуйте массивы, разработанные для 26 патчей.
    4. Обсудите «размеры» массива, введя слова факторы и продукт . Модель с примером:

      Попросите каждого ученика записать под своим дизайном лоскутного одеяла, что указано в поле выше, корректируя числа в соответствии с их собственным дизайном.

    Деятельность 2

    1. Напишите на доске 4 36 9.
      Вот еще три числа, которые связаны умножением и делением.
      Запишите набор уравнений умножения и деления, используя эти числа.

    2. Попросите учащихся работать в парах, чтобы разработать уравнения и создать массив, представляющий все четыре уравнения. Студенты должны быть готовы обосновать свою позицию (объяснить, откуда они знают, что они правы).
      4 x 9 = 36 9 x 4 = 36 36 ÷ 4 = 9 36 ÷ 9 = 4
      Свяжите каждое уравнение с массивом 9 x 4, который учащиеся должны распознать по дизайну как задание для квилтинга.Обратите особое внимание на разделение. Например, 36 ÷ 4 = 9 дает количество строк, созданных из 36 фрагментов (области), если каждый ряд состоит из четырех фрагментов.

    3. Обобщите полученные данные в таблице класса. Например:
      • Есть только четыре связанных факта, (семейство фактов) и не более.
        4 x 7 = 28 7 x 4 = 28 28 ÷ 4 = 7 28 ÷ 7 = 4
      • Умножение — это «оборотная» операция. Вы можете изменить порядок факторов, не меняя продукт.(Это похоже на сложение.)
        Мы говорим, что умножение (и сложение) коммутативны .
        4 х 7 = 7 х 4 = 28
      • Division не коммутативна, например 36 ÷ 4 = 9, но 4 ÷ 36 = 0,1111… (1/9). У делений разное частное (ответ).
        Мы говорим, что деление (и вычитание) не коммутативны .

    Деятельность 3

    1. Попросите учащихся сыграть в игру Умножь, нарисуй и напиши в парах .
      Им нужны игральные карты (с цифрами от 2 до 9), карандаш и бумага.
      Побеждает тот, у кого после десяти раундов больше всего пар карточек с одинаковыми товарами, но сделанными с разными факторами.
      Например: 6 x 4 = 8 x 3 = 24 или 4 x 4 = 2 x 8 = 16
      Как играть:
      Карты перемешиваются и кладутся рубашкой вверх между обоими игроками.
      Игроки по очереди переворачивают три карты из стопки. Это факторы.Игрок возвращает одну карту в конец стопки. Игрок должен записать факт умножения для двух карт. Они также могут нарисовать массив и написать семейство фактов.
      Например:
    2. Учащиеся завершают занятие, записывая словесные сценарии для своих наборов уравнений (семейство фактов). Это не обязательно сценарии лоскутного одеяла.
      Например: «Было три мешка по пять яблок в каждом. Пятнадцать, разделенные на три сумки, составляют пять. Если эти пятнадцать яблок положить в пять пакетов, то в каждом будет по три.Это будет пять лотов из трех ».

    Сессия 4

    Деятельность 1

    Покажите альтернативный набор стеганых одеял или тапа (PowerPoint Two). Например:

    Попросите четырех студентов записать по одному из связанных фактов.
    (6 x 5 = 30, 5 x 6 = 30, 30 ÷ 5 = 6, 30 ÷ 6 = 5) и объясните каждый факт со ссылкой на лоскутное одеяло, включая демонстрацию коммутативного (поворотного) свойства умножения. Поверните одеяло, чтобы продемонстрировать это.

    Действие 2

    1. Раздайте учащимся связующие кубики (или цветные фишки). Попросите пары учеников взять по 48 кубиков. Спросите, какие факторы могут дать 48. Запишите возможности, используя умножение; 1 x 48, 2 x 24, 3 x 16 и т. Д.
    2. Пусть по одному учащемуся из каждой пары учеников моделируют 4 x 12, соединяя кубики. Затем попросите их партнера использовать те же кубики для моделирования 12 x 4. Обсудите, что происходит. (Им нужно было их перегруппировать). Повторите с 6 х 8 и 8 х 6.Подчеркните, что коммутативное свойство включает в себя те же факторы и продукт, но требует другого способа просмотра массива (т. е. строки или столбец образуют равные наборы).
    3. Поместите факты умножения 48 на карты (Копимастер Один). Сдержать 5 раз? и 7 х? Сопоставьте пары уравнений, которые показывают коммутативность.
      Как вы думаете, это все факты умножения на произведение 48? (Вы можете расположить карты по первому множителю.)
      Почему нет фактов 5 x и 7 x? (Используйте карточки. Студенты должны понимать, что 48 не входит в набор, кратный 5 и 7. 48 не делится на 5 и 7).
      Воспользуйтесь калькулятором, чтобы показать, что 48 ÷ 5 = 9,6 и 48 ÷ 7 = 6,857142857…
      Как вы думаете, что показывает десятичная часть произведения? (остаток, поэтому 48 не делится на 5 и 7)
    4. Попросите учащихся изучить факты умножения с разным количеством кубиков, используя язык тех же факторов и продукта, уделяя особое внимание перегруппировке.Исследование может показать, что некоторые числа имеют только два делителя, например 17 и 31. Это простые числа.

    Деятельность 3

    1. Запишите одно знакомое уравнение умножения в таблицу классов. Например, 6 x 2 = 12. Попросите одного из учеников в каждой паре смоделировать это, составив 6 групп по 2 и соединив кубики вместе в одну линию из 12.

      Запишите 12 ÷ 6 = 2. Студент в паре разыграет это кубиками.

      Попросите учащихся описать то, что произошло, и записать такие идеи, как: это противоположное, деление без умножения, все наоборот, мы вернулись к тому, с чего начали.
      Спросите, Всегда ли это правда? Как мы можем узнать? Принимайте идеи студентов. Сюда должны входить учащиеся, исследующие больше примеров.

    2. Сделайте вывод, что невозможно проверить все факты умножения и деления. Скажем, идея «отмены» означает, что умножение и деление являются обратными операциями, как включение и выключение света.Отмена друг друга — это просто способ, которым ведут себя умножение и деление.

    3. Запишите обратную связь в таблице классов. Обсудите слова, похожие на обратное, например: перевернуть, отменить, вернуть, вернуть и их значение. Установите связь с обратной зависимостью между сложением и вычитанием. Выделите, что в каждой паре операций одна операция или действие отменяет другое.

    4. Вернитесь к лоскутному одеялу в Задании 1 (выше) и к записанным уравнениям:
      (6 x 6 = 30, 5 x 6 = 30, 30 ÷ 5 = 6, 30 ÷ 6 = 5)
      Попросите студентов объяснить « отмена »(снова обратная зависимость, применительно к лоскутному одеялу.(Это немного труднее увидеть, потому что этот массив физически невозможно «отменить». Однако вы можете создать ряды из шести кубиков и отобразить 5 x 6, расположив пять рядов по вертикали. Сколько у меня патчей? Что произойдет, если я теперь разделите на пять? )

    5. Напишите в таблице класса:
      Знание того, что умножение и деление являются обратными операциями, полезно, потому что …… ..
      Попросите учащихся указать причины и записать их, в том числе:
      Мы можем использовать умножение, чтобы помочь нам решить задачи деления.
      Мы можем проверить операции деления с помощью умножения. (Как?)

    Деятельность 4

    Раздайте Copymaster Two студентам, с которыми они могут работать. Подчеркните обратные операции, и необходимость для учащихся показать или объяснить , как умножение помогает решать задачи деления.

    Сессия 5

    Деятельность 1

    Просмотрите основные выводы занятия 4. Предложите учащимся поработать в парах, чтобы поделиться своими решениями задач лоскутного одеяла из занятия 4, занятие 4.Поощряйте их задавать вопросы друг другу.

    Действие 2

    1. Покажите несколько примеров стеганой ткани или ткани тапа с помощью PowerPoint One:
    2. Напишите в таблице классов:
      Одно лоскутное одеяло из шестнадцати заплат:
      Одно лоскутное одеяло из тридцати заплат:
      Одно лоскутное одеяло из сорока пяти заплат:
      Если бы я расположил лоскутки в один ряд, как бы выглядело лоскутное одеяло? (Больше похоже на длинный шарф)
    3. Попросите студентов записать уравнения умножения для каждого из этих утверждений.
      Одно стеганое одеяло из шестнадцати заплат: 1 x 16 = 16
      Одно одеяло из девяти заплат: 1 x 30 = 30
      Одно одеяло из тридцати заплат: 1 x 45 = 45
      Если ваши уравнения верны, каковы ответы на 16 ÷ 1 = ☐, 30 ÷ 1 = ☐, 45 ÷ 1 = ☐?
    4. Попросите студентов обсудить свои идеи, а затем объяснить и обосновать свое мышление. Связаны ли они с делением с вопросом «Сколько столбцов в одном патче составляет в общей сложности 16, 30 или 45 патчей?»
      Если ваши уравнения верны, каковы ответы на вопросы: 16 ÷ 16 = ☐, 30 ÷ 30 = ☐, 45 ÷ 45 = ☐?
      Связаны ли они с делением с вопросом «Сколько рядов по 16 фрагментов составляют в общей сложности 16 фрагментов и т. Д.?»
    5. Приведите другие примеры деления числа на единицу и само себя.Калькуляторы можно использовать для проверки ответов.

    Деятельность 3

    Попросите учащихся работать группами от 2 до 4 человек по телефону Это факт? (Мастер-3 (Цель: различать правильные и неправильные уравнения и выражения умножения и деления и уметь объяснять, почему, обосновывая свое решение)

    Учащиеся по очереди выбирают утверждение и объясняют остальным в группе, является ли это утверждение фактом и почему оно неверно (истинное или ложное).

    Попросите учащихся создать свои собственные факты или не факты, которые включают умножение и деление, например 8 x 9 = 72, поэтому 72 ÷ 18 = 4. Обменивайтесь фактами и не-фактами между учащимися.

    Действие 4

    Завершите это занятие, проанализировав обучение, полученное за пять занятий.

    Домашняя ссылка

    Уважаемые родители и ванау,

    В алгебре на этой неделе мы изучали числовые операции умножения и деления и взаимосвязь между ними.

    Один из способов закрепить то, что они изучали, — это прочитать истинные и ложные утверждения, а также определить и объяснить, какие из них неверны и почему.

    Ваш ребенок может захотеть поиграть на Это факт? игра с вами. По очереди очень важно, чтобы каждый человек объяснил, почему утверждение является верным или нет.

    Надеемся, вам понравится этот вызов.

    Спасибо.

    Попробуйте эти примеры:

    1 x 25 = 25, поэтому 25 ÷ 25 = 1 Верно или неверно

    28 ÷ 4 = 7, поэтому 4 ÷ 28 = 7 Верно или неверно

    предложений умножения и деления — элементарная математика

    Назначение

    Для распознавания и обозначения предложений, связанных с умножением и делением

    Материалы

    Нет

    Обзор

    Чтобы подготовиться к предстоящей работе с умножением и делением, попросите ваших учеников попрактиковаться в фактах до 10 × 10. Приведите факт умножения, например 5 × 6, и попросите учащегося назвать произведение и его предложение умножения (5 × 6 = 30). Затем попросите другого ученика дать соответствующее разделение (30 ÷ 6 = 5 или 30 ÷ 5 = 6).

    Класс также можно разделить на две команды. Первая группа дает предложение умножения и произведение, а вторая команда дает соответствующее предложение деления и частное. Когда учитель говорит: «Переключитесь!» каждая команда работает с противоположной операцией.

    О последовательности

    Часть 1 просит студентов попрактиковаться в умножении до 5 × 10 и поделиться соответствующими предложениями умножения и деления.Часть 2 включает в себя факты размером до 10 × 10 и факты расширенного теста до 12 × 12, оба с дополнительной практикой по предоставлению связанных предложений умножения и деления.

    Часть 1

    Давайте продолжим практиковать наши факты умножения. Я поделюсь фактом, и один доброволец (или команда) даст продукт вместе с предложением умножения, которое к нему прилагается. Второй доброволец (или команда) разделяет частное и соответствующее предложение деления. Итак, если я скажу 2 × 6, наш первый доброволец (или команда) скажет 2 × 6 = 12, а второй доброволец (команда) скажет 12 ÷ 6 = 2 или 12 ÷ 2 = 6.Давайте начнем!

    Примеры:

    • 2 × 4 = 8 (8 ÷ 4 = 2 или 8 ÷ 2 = 4)
    • 3 × 5 = 15 (15 ÷ 5 = 3 или 15 ÷ 3 = 5)
    • 4 × 4 = 16 (16 ÷ 4 = 4)
    • 5 × 4 = 20 (20 ÷ 4 = 5 или 20 ÷ 5 = 4)
    • 4 × 3 = 12 (12 ÷ 3 = 4 или 12 ÷ 4 = 3)
    • 3 × 3 = 9 (9 ÷ 3 = 3)
    • 2 × 10 = 20 (20 ÷ 10 = 2 или 20 ÷ 2 = 10)
    • 1 × 12 = 12 (12 ÷ 12 = 1 или 12 ÷ 1 = 12)
    • 2 × 7 = 14 (14 ÷ 7 = 2 или 14 ÷ 2 = 7)
    • 3 × 6 = 18 (18 ÷ 6 = 3 или 18 ÷ 3 = 6)

    Пока дети наслаждаются развитием мастерства, не стесняйтесь повторять.Когда дети хотят большего, попробуйте Часть 2.

    Часть 2

    Давайте продолжим и еще несколько фактов!

    Примеры:

    • 10 × 10 = 100 (100 ÷ 10 = 10)
    • 9 × 8 = 72 (72 ÷ 8 = 9 или 72 ÷ 9 = 8)
    • 7 × 6 = 42 (42 ÷ 6 = 7 или 42 ÷ 7 = 6)
    • 8 × 5 = 40 (40 ÷ 5 = 8 или 40 ÷ 8 = 5)
    • 6 × 9 = 54 (54 ÷ 9 = 6 или 54 ÷ 6 = 9)
    • 7 × 7 = 49 (49 ÷ 7 = 7)
    • 9 × 9 = 81 (81 ÷ 9 = 9)
    • 6 × 8 = 48 (48 ÷ 8 = 6 или 48 ÷ 6 = 8)
    • 9 × 1 = 9 (9 ÷ 1 = 9 или 9 ÷ 9 = 1)

    Как всегда, когда детям кажется, что их ждет новая задача, двигайтесь дальше.

    добавочный номер

    Давайте попробуем еще более важные факты.

    • 11 × 12 = 132 (132 ÷ 12 = 11 или 132 ÷ 11 = 12)
    • 12 × 12 = 144 (144 ÷ 12 = 12)
    • 10 × 12 = 120 (120 ÷ 12 = 10 или 120 ÷ 10 = 12)
    • 11 × 9 = 99 (99 ÷ 9 = 11 или 99 ÷ 11 = 9)
    • 12 × 4 = 48 (48 ÷ 4 = 12 или 48 ÷ 12 = 4)
    • 12 × 8 = 96 (96 ÷ 8 = 12 или 96 ÷ 12 = 8)
    • 11 × 11 = 121 (121 ÷ 11 = 11)
    • 9 × 12 = 108 (108 ÷ 12 = 9 или 108 ÷ 9 = 12)
    • 11 × 6 = 66 (66 ÷ 6 = 11 или 66 ÷ 11 = 6)

    Решение задач умножением и делением дробей и смешанных чисел

    Пример 1. Если для изготовления платья требуется 5/6 ярдов ткани, то сколько ярдов потребуется для изготовления 8 платьев?

    Анализ: Чтобы решить эту проблему, мы преобразуем целое число в неправильную дробь.Затем мы умножим две дроби.

    Решение:

    Ответ: Для изготовления 8 платьев потребуется 6 и 2/3 ярда ткани.


    Пример 2: У Рене была коробка кексов, половину которой она отдала своему другу Хуану. Хуан отдал 3/4 своей доли своей подруге Елене. Какая дробная часть оригинальной коробки кексов досталась Елене?

    Анализ: Чтобы решить эту проблему, мы умножим эти две дроби.

    Решение:

    Ответ: Елене досталось 3/8 оригинальной коробки кексов.


    Пример 3: Класс математики Нины имеет длину 6 и 4/5 метра и ширину 1 и 3/8 метра. Какая площадь классной комнаты?

    Анализ: Чтобы решить эту проблему, мы умножим эти смешанные числа. Но сначала мы должны преобразовать каждое смешанное число в неправильную дробь.

    Решение:

    Ответ: Площадь аудитории 9 и 7/20 квадратных метров.


    Пример 4: Плитка шоколада имеет длину 3/4 дюйма. Если его разделить на части длиной 3/8 дюйма, то сколько это будет частей?

    Анализ: Чтобы решить эту задачу, мы разделим первую дробь на вторую.

    Решение:

    Ответ: 2 шт.


    Пример 5: У электрика есть кусок провода длиной 4 и 3/8 сантиметра. Она делит проволоку на кусочки длиной 1 и 2/3 сантиметра. Сколько у нее штук?

    Анализ: Чтобы решить эту проблему, мы разделим первое смешанное число на второе.

    Решение:

    Ответ: Электрик имеет 2 и 5/8 куска провода.


    Пример 6: На складе 1 и 3/10 метров ленты. Если они разделят ленту на куски длиной 5/8 метров, то сколько кусков у них получится?

    Анализ: Чтобы решить эту проблему, мы разделим первое смешанное число на второе. Сначала мы преобразуем каждое смешанное число в неправильную дробь.

    Решение:

    Ответ: На складе будет 2 и 2/25 кусков ленты.


    Резюме: В этом уроке мы узнали, как решать задачи со словами, связанные с умножением и делением дробей и смешанных чисел.


    Упражнения

    Указания: вычтите смешанные числа в каждом упражнении ниже. Обязательно упростите свой результат, если необходимо. Щелкните один раз в ОКНО ОТВЕТА и введите свой ответ; затем нажмите ENTER. После того, как вы нажмете ENTER, в БЛОКЕ РЕЗУЛЬТАТОВ появится сообщение, указывающее, правильный или неправильный ваш ответ. Чтобы начать заново, нажмите ОЧИСТИТЬ.

    Примечание. Чтобы написать смешанное число четыре и две трети, введите в форму 4, пробел и затем 2/3.

    1. Одна партия печенья содержит 1 и 3/4 стакана растопленного шоколада. Сколько чашек растопленного шоколада нужно для изготовления 8 партий печенья?
    2. Тодд выпил 5/8 банки сока объемом 24 унции. Лайла выпила на треть меньше сока, чем Тодд. Сколько унций выпила Лила?
    3. Прямоугольный коврик имеет длину 3 и 2/3 фута и ширину 2 и 3/4 фута. Какова площадь коврика?
    4. У Джанет 5 и 3/4 сантиметра лакричника. Она делит лакрицу на кусочки длиной 1 и 7/8 сантиметра. Сколько кусочков солодки у нее будет?
    5. Кусок дерева длиной 15 футов.Сколько 3/4 фута можно вырезать из него?

    Умножение 2- и 3-значных чисел

    Урок 2: Умножение 2- и 3-значных чисел

    / ru / multiplicationdivision / от введения к умножению / содержание /

    Комплексные задачи умножения

    Умножая число или сумму, вы на умножаете во много раз. Из «Введение в умножение» вы узнали, что умножение может быть способом понять вещи, которые происходят в реальной жизни.Например, представьте, что в магазине продаются коробки с грушами. В маленьких коробках по штук по пять груш по груша. Вы покупаете два . Вы можете написать такую ​​ситуацию и использовать таблицу умножения для ее решения:

    Теперь представьте, что вы решили купить двух больших коробок , содержащих 14 груш в каждой. Эта ситуация будет выглядеть так:

    Эту проблему решить сложнее. Подсчет груш займет некоторое время. К тому же в таблице умножения нет 14.К счастью, есть способ записать проблему, чтобы можно было разбить ее на более мелкие части. Это называется , укладка . Это означает, что мы будем писать числа друг над другом, , а не рядом.

    • Давайте попрактикуемся в наложении этой задачи, 14 x 2.

    • Сначала напишите числа друг над другом. Хорошая привычка всегда писать сверху большее число . Если вы этого не сделаете, решить проблему будет труднее.

    • Затем напишите знак умножить на на слева чисел.

    • Вместо знака равно поставьте черту под числом внизу.

    • Обратите внимание, как числа выровнены до справа ?

    • Когда вы пишете задачу умножения с накоплением, всегда следите за тем, чтобы числа были выстроены таким образом.

    • Например, давайте рассмотрим другую задачу, 5 x 112.Видите, как 2 находится прямо над 5?

    • Также обратите внимание, что мы поместили большее число вверху, хотя это было второе число в нашем исходном выражении.

    • Всегда настраивайте задачи умножения с накоплением одинаково: с большим числом наверху …

    • Всегда настраивайте задачи умножения с накоплением одинаково: с большим числом наверху … и правильными цифрами выстроились в очередь.

    Решение задач сложного умножения

    На первый взгляд задачи сложенного умножения могут показаться довольно сложными.Не волнуйтесь! Если вы можете решить задачи из «Введение в умножение», вы также сможете научиться решать эти проблемы. Чтобы умножать большие числа, вы будете использовать те же базовые навыки, что и для умножения маленьких. Вы даже можете использовать те же инструменты, например, таблицу умножения на .

    Давайте посмотрим, как работает решение задач умножения с накоплением.

    • Помните пример с двумя коробками, в каждой по 14 груш? Чтобы узнать, сколько всего груш, решим эту задачу: 14 x 2.

    • Когда вы умножаете сложенные числа, вы начинаете с правой цифры в нижнем числе задачи. Наше нижнее число состоит только из одной цифры: 2.

    • Мы умножим 2 на верхнее число, 14. Поскольку в таблице умножения нет 14, нам придется умножать по одной цифре за раз.

    • Как обычно, решим проблему с справа налево . Итак, мы умножим 2 на цифру в правом верхнем углу . Здесь это 4.

    • Теперь пришло время решить 2 x 4. Мы можем использовать таблицу умножения.

    • 2 x 4 равно 8. Запишем 8 под 2 и 4.

    • Теперь умножим 2 на следующую цифру слева: 1.

    • Теперь решим 2 x 1.

    • Всякий раз, когда вы умножаете число на 1, это число остается таким же, как . Итак, 2 x 1 равно 2. Чтобы быть уверенным, мы проверим таблицу умножения.

    • Напишите 2 под линией, непосредственно под 1.

    • Готово! Всего у нас 28, или двадцать восемь. 14 x 2 = 28.

    • Попрактикуемся с другой задачей, 31 x 7.

    • Всегда начинайте с цифры справа внизу . Здесь это 7.

    • Сначала умножьте 7 на цифру в правом верхнем углу, 1.

    • 7 x 1 равно 7. Запишите 7 прямо под цифрами, которые мы только что умножили.

    • Затем мы умножим 7 на следующую цифру слева.Это 3.

    • Мы будем использовать таблицу умножения, чтобы найти 7 x 3.

    • 7 x 3 равно 21. Обязательно выровняйте числа так, чтобы правая цифра 21, 1 была непосредственно ниже 3.

    • Наш ответ — 217. Итак, 31 x 7 = 217.

    Попробуйте это!

    Сложите и решите эти задачи умножения. Затем проверьте свой ответ, введя его в поле.

    Использование переноски

    На последней странице вы практиковались в умножении вертикально сложенных чисел.Некоторые проблемы требуют дополнительного шага. Давайте посмотрим на следующую задачу:

    Если вы попытаетесь умножить 9 x 5, вы можете заметить, что нет места для записи произведения, 45. Когда произведение двух чисел больше, чем 9 , вам нужно будет использовать технику под названием , несущую . Если вы знаете, как складывать большие числа, вы, возможно, помните, как добавляли перенос. Посмотрим, как это работает при умножении.

    • Давайте попробуем решить задачу, которую мы только что рассмотрели, 29 x 5.

    • Как обычно, мы начнем с умножения 5 на верхнюю правую цифру, 9.

    • Согласно нашей таблице умножения, 5 x 9 равно 45, но нет места для записи обеих цифр под 5 и 9.

    • Правую цифру 5 запишем под чертой …

    • Правую цифру 5 запишем под чертой … тогда перенесем левая цифра 4 до следующего набора цифр в задаче.

    • Посмотрите, как это работает? Мы умножили 5 на 9 и получили 45.Мы поместили 5 под линией, перенесли 4 и поместили ее над следующим набором цифр.

    • Теперь пора сделать следующий шаг. Это то же самое, что и с любой другой задачей умножения. Умножим 5 x 2.

    • 5 x 2 = 10. Впрочем, 10 под чертой пока писать не будем — еще один шаг.

    • Помните номер, который у нас был, 4?

    • Нам нужно к добавить к нашему продукту, 10.

    • 4 + 10 равно 14.

    • Напишем 14 под строкой.

    • Наше общее количество составляет 145. Теперь мы знаем, что 29 x 5 = 145.

    • Давайте попробуем другую задачу, просто для практики. 208 x 6.

    • Сначала мы умножаем нижнее число 6 на цифру в правом верхнем углу. Это 8.

    • 6 x 8 равно 48.

    • Мы запишем 8 под чертой …

    • Мы напишем 8 под чертой … и перенесем 4.Поместим его над следующей цифрой.

    • Следующая цифра — 0.

    • Все, умноженное на ноль, равно 0, поэтому мы знаем, что 6 x 0 = 0.

    • Помните, мы еще не пишем 0 под линией. Мы должны добавить его к четырем только что перенесенным.

    • 4 + 0 = 4. Напишем 4 под строкой.

    • Наконец, мы умножаем 6 и 2.

    • 6 x 2 = 12, поэтому мы напишем 12 под линией.

    • Готово! Ответ: 1248, или одна тысяча двести сорок восемь.208 x 6 = 1248.

    Попробуй!

    Сложите и решите эти задачи умножения. Затем проверьте свой ответ, введя его в поле.

    Умножение больших чисел

    На последних нескольких страницах вы тренировались умножать большие числа на маленькие. Что произойдет, если вам нужно умножить два больших числа?

    Например, представьте, что ваш счет за мобильный телефон составляет 43 доллара в месяц . В году 12 месяц, поэтому, чтобы узнать, сколько вы платите за свой телефон каждый год, вы можете решить для 43 x 12. Вы могли бы написать такое выражение:

    Сначала эта проблема может показаться сложной, но не беспокойтесь. Если вы можете умножать маленькие числа, вы можете умножать и большие числа. Все, что вам нужно сделать, это разделить эту большую проблему на несколько более мелких. Как всегда, вы можете воспользоваться таблицей умножения .

    • Чтобы решить такую ​​большую задачу, начните с тех же шагов, которые вы используете для решения любой другой задачи умножения.

    • Как всегда, вы начинаете с цифры в правом нижнем углу.Здесь это 2.

    • Мы умножим это на цифру в верхнем правом углу, 3.

    • Благодаря нашей таблице умножения мы знаем, что 3 x 2 равно 6.

    • Мы будем напишите 6 под линией в крайнем правом углу.

    • Затем мы умножим 2 x 4.

    • 2 x 4 равно 8.

    • Напишите 8 под линией, прямо под 4.

    • ОК. Первая половина задачи сделана.

    • Теперь пора снова взглянуть на нижнее число.

    • Мы собираемся умножить следующую цифру. Это 1.

    • Сначала умножьте 1 на верхнее число справа. Здесь это 3.

    • 1 x 3 равно 3 … но мы не собираемся писать 3 в обычном месте.

    • Вместо того, чтобы записывать 3 в правый , как мы обычно делаем …

    • Вместо того, чтобы записывать 3 в правый , как мы обычно делаем … мы собираемся написать он находится на одну позицию слева под вторым набором цифр.

    • Рекомендуется отметить место, которое вы оставили пустым. Таким образом, вы будете знать, что нельзя ничего случайно там написать. Мы добавим 0, так как ноль — это то же самое, что ничего .

    • Теперь давайте умножим последний набор чисел. Это 1 x 4.

    • 1 x 4 = 4. Мы запишем 4 под строкой слева от 3, которые мы только что написали.

    • Последний шаг. Чтобы получить окончательный ответ, нам нужно к прибавить число, которое мы только что получили в результате умножения.

    • Как всегда, начнем добавлять справа.

    • 6 + 0 равно 6. Запишем 6 под линией.

    • Далее, 8 + 3.

    • 8 + 3 равно 11. Поскольку 11 — это двухзначное число, нам придется нести.

    • Запишите правую цифру 1 под 8 и 3 …

    • Запишите левую цифру 1 под 8 и 3 … затем перенесите правую цифру и поместите ее над цифрой, чтобы слева.

    • Наконец, мы прибавим 4 к 1, которую только что перенесли.

    • 4 + 1 равно 5.

    • Готово! Наше общее количество составляет 516. Другими словами, 43 x 12 = 516.

    Попробуйте это!

    Сложите и умножьте эти двузначные числа. Затем проверьте свой ответ, введя его в поле.

    Умножение двух трехзначных чисел

    Умножение больших чисел всегда работает одинаково, независимо от того, сколько цифр они имеют. При умножении будьте осторожны, записывая числа в правильных местах.Давайте рассмотрим задачу с двумя 3-значными числами , чтобы увидеть, как это работает с еще большими числами.

    • Давайте попробуем эту задачу: 601 x 243.

    • Как всегда, начните с умножения правой нижней цифры на верхнюю правую цифру. Итак, 3 x 1.

    • Благодаря нашей таблице умножения мы знаем, что 3 x 1 равно 3. Запишите 3 под линией, в крайнем правом углу.

    • Теперь умножьте 3 на следующее число, 0.

    • Все, что умножено на ноль, равно 0, поэтому напишите 0 под строкой, рядом с 3.

    • Далее, 3 x 6.

    • 3 x 6 = 18. Напишите 18 под линией.

    • Мы закончили с первой цифрой в нижнем числе.

    • Затем умножьте на второе число внизу, 4.

    • 4 x 1 равно 4. Помните, вы не собираетесь писать 4 до упора вправо.

    • Вместо этого напишите 4 на одну позицию слева под вторым набором чисел.

    • Чтобы все было выровнено, мы поместим ноль в качестве заполнителя справа от четырех.

    • Теперь перейдем к следующему числу сверху — 0.

    • 4 x 0 равно 0. Запишите 0 под линией.

    • Затем умножьте 4 на последнюю цифру в верхнем числе — 6.

    • 4 x 6 равно 24. Напишите 24 под линией.

    • Мы готовы умножить на последнюю цифру в нашем нижнем числе — 2.

    • Как всегда, начинайте с верхней правой цифры 1.

    • 2 x 1 равно 2.

    • Мы запишем 2 под линией, два пробела справа.

    • Обратите внимание, куда мы поместили 2.

    • Когда мы умножили на первую цифру в нижнем числе …

    • Когда мы умножили на первую цифру в нижнем числе .. Мы выстроили продукт до конца вправо .

    • Когда мы умножили на секунд цифру …

    • Когда мы умножили на секунд цифра … мы записали произведение на один пробел слева.

    • Теперь, когда мы умножили на третью цифру

    • Теперь, когда мы умножили на третью цифру … мы поместили произведение на два пробелов слева.

    • Вы могли заметить закономерность.Каждый раз, когда мы умножали новую цифру, мы записывали произведение на одну цифру левее. Это верно независимо от того, сколько цифр в числах, на которые вы умножаете.

    • Вернемся к нашей проблеме. Мы просто умножили 2 x 1.

    • Следующая цифра — 0.

    • 2 x 0 равно 0. Напишите 0 под линией.

    • Наконец, умножьте 2 x 6.

    • 2 x 6 равно 12. Напишите 12 под линией.

    • Время добавлять.Как всегда, начните с цифр справа. Здесь это означает, что мы складываем 3 + 0 + 0.

    • 3 + 0 + 0 = 3. Напишите 3 непосредственно под цифрами, которые мы только что добавили.

    • Затем мы добавим 0 + 4 + 0.

    • 0 + 4 + 0 равно 4.

    • Теперь следующий набор цифр, 8 + 0 + 2.

    • 8 + 0 + 2 = 10. 10 — это двузначное число, поэтому нам нужно нести . Напишите 0 под только что добавленными цифрами и поместите 1 над следующим набором цифр.

    • Пришло время добавить 1, которую мы только что принесли. Это означает, что мы решаем для 1 + 1+ 4 + 0.

    • 1 + 1 + 4 + 0 = 6. Напишите 6 под линией.

    • Далее, 2 + 2.

    • 2 + 2 = 4. Напишите 4 под строкой.

    • Слева всего одна цифра — 1.

    Сложение, вычитание, умножение, деление в пределах 100. Путешествие в зимний лес

    Цели: Закрепить навыки сложения, вычитания, умножения деления в пределах 100 через решение примеров и задач. Развивать в логическом мышлении мыслительные операции анализа и синтеза. Активизировать и пополнять словарный запас. Развивать познавательные интересы. Прививать навыки само и взаимоконтроля.

    Оборудование: иллюстрации с изображением зимнего леса, карточки.

    Ход урока

    1. Орг. момент.

    — Угадайте загадку.

    Запорошила дорожки
    Разукрасила окошки.
    Радость детям подарила
    И на санках прокатила.
    — Почему решили, что это зима?

    — Какое сейчас время года?

    — Какой месяц?

    — Декабрь. Какой это месяц по календарю?

    — Какой по счёту месяц зимы?

    а) Оформление записи в тетради.

    б) Сообщение темы урока.

    Рано утром от вокзала
    Электричка побежала.
    На скамейке ровно в ряд
    Двадцать пять ребят сидят.

    Едут лжи, едут санки,
    Шубы,
    Валенки,
    Ушанки-
    Саша, Даша, Коля,
    Таня, Аня, Оля.
    Остановка… Настежь двери!

    — Сегодня с вами мы совершим экскурсию в зимний лес, а по пути повторим сложение, вычитание, умножение, деление в пределах 100.

    И так:

    Нас встречает зимний лес.
    Он стоит, как белый терем,
    Полный сказок и чудес.

    2. Устный счёт.

    Между сосен, между елей,
    Где вчера мели метели
    Мчатся лыжи, мчатся санки
    Шубы, Валенки, Ушанки.

    Мы с вами тоже встали на лыжи и поехали. Чтобы не сбиться с пути поедем от одного флажка к другому.

    Дети решают примеры, написанные на обратной стороне флажков.

    Если правильно решите все примеры, то узнаете как на Руси в старину называли декабрь.

    29-15=14 х

    70-8=62 м

    48+12=60 у

    25+33= р

    6*8= 48 е

    48:6=8 нь

    — Как вы думаете, почему декабрь называли хмурнем?

    3. Физминутка.

    4. Закрепление полученных знаний.

    А) решение примеров из учебника.

    Мы вспугнули стайку птиц
    Двух клестов и трёх синиц.
    — Не желаете ли птицы
    Вместе с нами подкрепиться?

    Стр. 108 № 1и 2.

    1-я группа решает все примеры, 2-я группа 1 или 2 столбика.

    Индивидуально:

    Проверка: Ответы написаны на птичках. На каждый правильный ответ на кормушку прилетает птичка.

    — Назовите птиц, которые прилетели на кормушку. (Голубь, ворона, сорока, свиристели, щеглы, клесты)

    — Что вызнаете о клестах?

    — Что это за птицы?

    — Почему их называют зимующими?

    Б) Устное решение задач.

    Едут дальше по тропинке
    Ровно двадцать пять ребят.
    Тихо белые снежинки
    В зимнем воздухе кружат.
    Задевая за верхушки
    Проплывают облака
    Вот и домик на опушке.
    Это — домик лесника.

    — Кто такой лесник?

    — Зайдём в домик. Погреемся. А пока греемся, по решаем устно задачи.

    На ветвях украшенных
    Снежной бахромой
    Яблоки румяные выросли зимой.
    Прилетели весело их десятка три.
    Тут смотри, ещё летят,
    Их теперь уж пятьдесят.
    Вы подумайте о том,
    Сколько птиц прилетело потом? 50-20=30

    На большой зелёной ели
    Сорок семь сорок галдели.
    Посидели, погалдели
    И семнадцать улетели.
    Отвечай-ка точно в срок:
    Сколько осталось на ёлке сорок? 47-17=

    Письменное решение задачи на карточках.

    — Какой праздник будем отмечать в конце декабря?

    — Чем этот праздник отличается от других?

    Для новогоднего праздника дети сделали 24 фонарика, 36 шариков, а снежинок в 6 раз меньше. Сколько всего игрушек сделали дети?

    После разбора задачу решают самостоятельно.

    2 группе дать карточку с вопросами в разброс. У доски задачу решает:

    На нашей ёлке загорелись з фонарика. На них 3 ответа к данной задаче. И только один из них верный.

    Если вы правильно решите задачу, то узнаете, какие птицы выводят своих птенцов зимой.

    90 — свиристели.

    70 — клесты

    66 — щеглы.

    5. Работа с геометрическим материалом.

    У лесного родника Лепим мы снеговика

    — Назовите геометрические фигуры. Есть ли здесь замкнутые ломаные линии?

    — Покажите их.

    — Что, из себя представляют окружности?

    6. Итог урока.

    Словно занавес на сцене
    Опустился вечер в друг.
    От берёз упали тени,
    Прекратился дятла стук.
    Встали лыжи, встали санки
    Шубы, Валенки,  Ушанки.
    Миша, Гриша, Рома,
    Лена, Гена, Тома…
    От реки и до вокзала
    Электричка побежала,
    Ровно двадцать пять ребят
    В окна синие глядят.
    Машет нам ветвями лес,
    Полный сказок и чудес.

    Завершилась экскурсия в зимний лес. Что мы видели в лесу? Что по пути повторили?

    7. Домашнее задание на карточках.

    С неба падают снежинки,
    Словно белые пушинки,
    Но снежинки важные,
    В них примеры разные.

    1 группа примеры в 2 действия, 1 группа в 1 действие.

    примеры на умножение и деление, сложение и вычитание

    Ваш ребенок еще только учится в начальной школе, а вы уже задумываетесь о его дальнейшей учебе, развитии и будущем? Это очень похвально. А думали ли вы над тем, что успеваемость ребенка можно улучшить, если заниматься с ним ежедневно по математике всего лишь 15 минут в день дополнительно? И это не выдумки. В материалах этой статьи мы приведем примеры и задачи для школьников начальной школы по математике, а именно, для третьеклассников. (Для удобства решения приведенные ниже задания вы можете распечатать).

    Содержание

    1. Как учить ребенка учиться
    2. Примеры и задачи по математике на умножение и деление
    3. Примеры и задачи по математике на сложение и вычитание
    4. Вместо заключения

    Как учить ребенка учиться

    Умеет ли ваш ребенок учиться? Уверена, что многих родителей этот вопрос поставил в тупик. А действительно, что значит «уметь учиться»? Когда ваш юный школьник только пошел в школу, после занятий, возможно, он бежал домой и очень хотел сразу же делать уроки. Так бывает, когда дети очень ждут поступления в 1 класс. Но со временем интересы к своевременному выполнению домашнего задания ослабевают и «домашка» становится скучным времяпровождением.

    А ведь именно нежелание выполнять домашние задания, готовиться к школьным рефератам, семинарам и викторинам, становится основной причиной того, что ребенок вначале не хочет, а после и не умеет учиться. Пробелы в знаниях могут накапливаться словно снежный ком, снижая успеваемость школьника и убивая в нем желание учиться.

    Чтобы школьник учился этой сложной и ответственной науке – учиться – родители должны всячески помогать ему: составить распорядок дня, учить ребенка выполнять домашнее задание наперед, прорешивать или прописывать дополнительные упражнения, чтобы тренировать и руку для письма, и мозг для устного счета. Математике дается детям начального звена сложнее всего, именно поэтому мы и подготовили для школьников 3 класса этот материал.

    Примеры по математике на умножение и деление

    Еще во втором классе дети выучили таблицу умножения. Если вы сейчас находитесь в полном заблуждении, как выучить с ребенком таблицу умножения, то рекомендуем к ознакомлению следующий материал по ссылке. На протяжении второго класса школьники постепенно осваивали простые примеры и задачи, используя таблицу умножения, а в третьем классе они оттачивают навыки умножения и сложения.

    Задание 1

    Заменить сложение вычитанием в тех примерах, в которых от замены знака ответ не изменится:

    5 + 5 + 5 =
    1 + 1 + 1 + 1 =
    0 + 0 + 0 + 0 + 0 =
    8 + 8 + 8 + 8 =
    7 + 7 — 7 + 7 =
    7 + 7 + 7 — 7 =
    14 + 14 =
    61 + 61 =

    Подсказка:

    5 + 5 + 5 = 15, если заменить знак «+» на знак «•», то получится
    5 • 5 • 5 = 125. 15 не равно 125. Значит, в первом равенстве заменить знак «+» на знак «•» нельзя.

    По аналогии решаем стальные равенства и делаем выводы о возможной или невозможной замене знака «+» на знак «•».

    Задание 2

    Какие выражения нельзя заменить суммой, чтобы ответ не изменился:

    0 • 4 =
    1 • 0 =
    1 • 1 =
    1 • 6 =
    0 • 9 =
    7 • 0 =
    5 • 2 =
    2 • 2 =

    Подсказка:

    Вспомните, каким правилом следует пользоваться при умножении на ноль.

    Задание 3

    Решите примеры:

    45 : 5 + 1 =
    45 : 5 • 1 =
    543 — 5 • 1 =
    (543 — 5) • 1 =
    423 + 7 • 0 =
    (423 + 7) • 1 =
    10 — 0 + 4 =
    10 • 0 + 4 =

    Задание 4

    Из каждого выражения на умножение составьте выражения на деление:

    6 • 8 =
    7 • 1 =
    4 • 0 =
    0 • 3 =
    4 • 9 =

    Подсказка

    6 • 8 = 48
    48 : 8 = 6
    48 : 8 = 6

    Задание 5

    Какое значение имеют следующие выражение:

    а : а =
    а : 1 =
    0 : а =
    а : 0 =

    Задание 6

    Решите примеры:

    (596 + 374) • 1 =
    596 + 374 • 1 =
    (596 + 374) • 0 =
    596 + 374 + 0 =
    0 • 320 : 1 =
    0 + 320 : 1 =

    Обязательно повторите с ребенком правила умножения и деления числа на единицу и умножения или деления числа на ноль, а также особенности деления ноля на любое число. Часто именно в этих примерах дети делают ошибки, которые влекут за собой дальнейшее неправильное решение примеров, выражений и задач.

    Задание 7 (задача)

    В оздоровительный лагерь привезли фрукты: 7 ящиков винограда и 5 ящиков персиков. Масса привезенных персиков составляет 40 килограммов. Какая масса винограда, если ящик винограда на 1 килограмм весит больше, чем ящик персиков.

    Решение

    Найдем, сколько весит один ящик персиков. Известно, что общая масса персиков составляет 40 кг, а всего ящиков – 5.

    Первое действие:
    40 : 5 = 8 (кг) весит один ящик персиков.

    Теперь найдем, сколько весит один ящик винограда, если известно, что он тяжелее на 1 кг, чем ящик персиков.

    Второе действие:
    8 + 1 = 9 (кг) весит один ящик винограда.

    Теперь находим общую массу всего винограда, если известно, что один ящик весит 9 кг, а всего винограда – 7 ящиков.

    Третье действие:
    9 • 7 = 63 (кг) – общая масса винограда.

    Ответ: масса привезенного винограда составляет 63 кг.

    Задание 8

    Сосна может расти 600 лет, береза – 350 лет. А ива – в 6 раз меньше от сосны. Что может расти дольше береза или ива? И насколько лет?

    Решение

    Вначале рассчитаем, сколько лет может расти ива, если известно, что она растет в 6 раз меньше, чем сосна.

    Первое действие:
    600 : 6 = 100 (лет) может расти ива.

    Теперь, когда известно, что ива может расти 100 лет, сравним продолжительность «жизни» березы и ивы. Известно, что береза растет 350 лет, а ива – 100. 350 больше чем 100, значит береза может расти дольше ивы. Чтобы рассчитать, на сколько береза может расти дольше ивы, решаем равенство.

    Второе действие:
    350 — 100 = 250 (лет) – на столько береза может расти дольше ивы

    Ответ: береза может расти дольше ивы на 250 лет.

    Важно! Если задачу можно решить несколькими способами, обязательно сообщите об этом ребенку. Пусть потренирует логику и начертит все возможные схем решения задачи, т.е. составить схематическое условие. Ведь правильно составленное условие задачи – это 90% успешного решения.

    Задание 9

    В понедельник гусеница начала ползти вверх по дереву высотой 9 метров. За день она поднялась вверх на 5 метров, а за ночь – опустилась на 2 метра. На какой день гусеница достигнет верхушки дерева?

    Решение

    Для начала рассчитаем, на сколько метров поднимается гусеница вверх за один день, с учетом того, что ночью на опускается.

    Первое действие:
    5 — 2 = 3 (м) гусеница проползает за сутки вверх.

    Теперь найдем количеств дней, необходимых на преодоление расстояния 9 метров вверх по дереву.

    Второе действие:
    9 : 3 = 3 (дня) нужно гусенице, чтобы достичь вершины дерева.

    Ответ: 3 дня нужно гусенице, чтобы достичь вершины дерева.

    Задание 10

    В коробке было 18 килограммов печенья. Сначала из нее взяли 13 килограммов печенья, потом досыпали в 4 раза больше, чем оставалось. Сколько килограммов печенья стало в коробке.

    Решение

    Сначала найдем, сколько килограммов печенья осталось в коробке, после того, как из нее забрали 13 килограммов.

    Первое действие:
    18 — 13 = 5 (кг) печенья осталось в коробке

    Теперь рассчитаем сколько килограммов печенья досыпали в коробку.

    Второе действие:
    5 • 4 = 20 (кг) досыпали

    Сложим тот вес, который оставался в коробке, и тот, который досыпали, чтобы найти, сколько килограммов печения стало в коробке.

    Третье действие:
    5 + 20 = 25 (кг) стало

    Ответ: 25 килограммов печения стало в коробке.

    Задание 11

    За лето хозяйка вырастила 208 домашних птиц. Кур и уток было 129, а уток и гусей – 115. Сколько кур, уток и гусей вырастила хозяйка за лето?

    Решение

    Известно, что кур и уток было 129, а всего птиц – 208. Значит, можно найти количество гусей.

    Первое действие:
    208 (птиц) – 129 (уток + кур) = 79 гусей

    Также известно, что уток и гусей всего 115, значит мы можем найти, сколько было кур.

    Второе действие:
    208 (птиц) – 115 (уток + гусей) = 93 кур

    Теперь, когда мы знаем количество гусей и кур, а также общее количество домашних птиц, мы можем найти количество уток.

    Третье действие:
    208 — (79 + 93) = 36 уток

    Ответ: за лето хозяйка вырастила 79 гусей, 93 кур и 36 уток.

    Второй вариант решения

    Известно, что кур и уток было 129, а всего птиц – 208. Значит, можно найти количество гусей.

    Первое действие:
    208 (птиц) – 129 (уток + кур) = 79 гусей

    Также известно, что уток и гусей всего 115, значит мы можем найти, сколько было уток

    Второе действие:
    115 (уток + гусей) – 79 (гусей) = 36 уток

    Теперь, когда мы знаем количество гусей и уток по отдельности, а также общее количество домашних птиц, мы можем найти количество кур.

    Третье действие:
    208 – (79 + 36) = 208 – 115 = 93 кур

    Ответ: за лето хозяйка вырастила 79 гусей, 93 кур и 36 уток.

    Примеры и задачи по математике на сложение и вычитание

    Основной задачей заданий и примеров по математике на сложение и вычитание в третьем классе является популяризация математических знаний и идей, поддержка и развитие математических знаний школьников, стимулирование и мотивация учеников в изучении естественно-математический предметов.

    Задание 1

    Реши уравнения:

    Х – 40 = 60
    Х + 4 = 61
    Х – 16 = 25
    Х + 25 = 84
    Х – 45 = 251
    Х + 56 = 106
    Х + 78 = 301

    Задание 2

    Расставьте скобки так, чтобы ответом выражения в первом случае было 6, а в втором – 2:

    12 : 2 + 2 • 2 =

    Подсказка

    12 : (2 + 2) • 2 = 6
    12 : (2 + 2 • 2) = 2

    Важно! Некоторые условия составлены таким образом, чтобы ребенок включал логическое мышление. Прорешивая такие задания он мыслит, делает предположения, размышляет, и находит правильное решение задания.

    Задание 3

    Перевести в одну систему измерения и решить выражения:

    1 м – 5 дм =
    1 м – 5 см =
    6 м 5 дм – 8 дм =
    5 см + 5 см =
    15 см + 5 дм =
    3 дм – 6 см =
    3 дм 5 см – 15 см =
    1 дм 2 см – 3 см =
    1 м 6 дм – 8 дм =

    Задание 4

    Из каждого выражения произведения отнять 15 и записать новые выражение и решить их:

    7 • 3 =
    7 • 6 =
    7 • 9 =
    8 • 6 =
    8 • 4 =
    3 • 9 =
    4 • 4 =
    5 • 7 =

    Подсказка

    Если 7 • 3 = 21, то 21 – 15 = 6

    Задание 5

    Решить примеры:

    7 • 6 + 7 • 4 =
    21 : 3 – 6 =
    (35 – 28) • 5 =
    (68 – 26) : 7 =
    7 + (6 : 2) =
    3 – 14 : 2 =
    60 – 63 : 7 =
    81 – 56 : 7 =
    50 + 42 : 7 =

    Задание 6 (задача)

    В шести одинаковых бочонках 24 литра воды. Сколько литров воды в сети таких же бочонках, на сколько литров больше во втором случае, чем в первом?

    Решение

    Вначале найдем, сколько воды вмещается в один бочонок.

    Первое действие:
    24 : 6 = 4 (л) в одном бочонке

    Теперь рассчитаем, сколько воды в семи одинаковых бочонках

    Второе действие:
    4 • 7 = 28 (л) в сети одинаковых бочонках

    Найдем ответ на главный вопрос задачи, на сколько литров больше во втором случае, чем в первом.

    Третье действие:
    28 – 24 = 4 (л) на столько литров больше во втором случае, чем в первом

    Ответ: на 4 литра воды больше во втором случае, чем в первом

    Задание 7

    Отец и сын купили на рынке картошку в 6 одинаковых сетках. Отец принес домой 4 сетки, а сын 2. Всего получилось 18 килограммов картошки. Сколько килограммов принес отец? Сколько килограммов принес сын? На сколько больше килограммов картошки принес отец?

    Решение

    Рассчитаем, сколько картошки было в одной сетке, если известно, то всего принести 18 килограммов в 6 одинаковых сетках.

    Первое действие:
    18 : 6 = 3 (кг) в одной сетке.

    Теперь узнаем сколько килограммов принес отец и сколько килограммов принес сын.

    Второе действие:
    3 • 4 = 12 (кг) принес отец

    Третье действие:
    3 • 2 = 6 (кг) принес сын

    Найдем искомую разницу.

    Четвертое действие:
    12 – 6 = 6 (кг) на столько больше принес отец.

    Ответ: Отец принес на 6 килограммов больше картошки, чем сын.

    Задание 8

    За 5 часов работы двигателя было израсходовано 30 литров бензина. Сколько бензина будет израсходовано за 8 часов работы двигателя. На сколько больше двигатель израсходует бензина за разницу во времени?

    Решение

    Рассчитаем, сколько бензина расходует двигатель за час своей работы.

    Первое действие:
    30 : 5 = 6 (л) за один час работы

    Рассчитаем, сколько составляет разница во времени?

    Второе действие:
    8 – 5 = 3 (ч) разница во времени

    Теперь можно рассчитать, сколько бензина израсходовано за оставшиеся 3 часа.

    Третье действие:
    3 • 6 = 18 (л) потрачено за 3 часа.

    Ответ: за 3 часа двигатель истратил 18 литров бензина

    Второй способ решения

    Рассчитаем, сколько бензина расходует двигатель за час своей работы.

    Первое действие:
    30 : 5 = 6 (л) за один час работы

    Рассчитаем, сколько бензина будет израсходовано за 8 часов работы двигателя.

    Второе действие:
    8 • 6 = 48 (л) израсходовано за 8 часов работы двигателя

    Теперь можно рассчитать разницу потраченного топлива.

    Третье действие:
    48 – 30 = 18 (л) разница потраченного топлива

    Ответ: за 3 часа двигатель истратил 18 литров бензина

    Важно! Задания на сложение и вычитание не исключают в своем условии или решении возможность других математических действий, например, умножения или деления. Ученик третьего класса уже должен уметь различать в условии требования к сложению и умножению, делению и вычитанию. Именно потому задания по математике для этого класса часто носят смешанный характер.

    Задание 9

    В двух прудах плавало 56 уток. Когда из первого пруда во второй перелетело 7 уток, то в нем осталось 25. Сколько уток с самого начала плавало во втором пруду?

    Решение

    Известно, что после того, как из первого пруда улетело 7 уток, в нем осталось 25. Находим количество уток в первом пруду с самого начала.

    Первое действие:
    7 + 25 = 32 (утки) было в первом пруду.

    Теперь можем найти, сколько уток плавало во втором пруду с самого начала.

    Второе действие:
    56 – 32 = 24 (утки) было во втором пруду.

    Ответ: с самого начала во втором пруду было 24 утки.

    Задание 10

    С первого куста собрали 9 килограммов ягод. Со второго куста собрали на 3 килограммов больше, чем с первого, а с третьего – на 2 килограммов больше, чем со второго. Сколько килограммов ягод собрали с третьего куста? Сколько всего ягод собрали?

    Решение

    Вначале найдем, сколько килограммов ягод собрали со второго куста.

    Первое действие:
    9 + 3 = 12 (кг) ягод со второго куста

    Теперь определяем, сколько килограммов ягод собрали с третьего куста

    Второе действие:
    12 + 2 = 14 (кг) год с третьего куста

    Когда все составляющие известны, находим ответ на главный вопрос задачи.

    Третье действие:
    9 + 12 + 14 = 35 (кг) ягод всего

    Ответ: всего собрали 35 килограммов ягод.

    Вместо заключения

    Уделяйте математике достаточно внимания уже с начальной школы. Этот предмет не только тренируем мозг в устном счете, но и умении логически мыслить, развивать смекалку. Постепенно привыкая к выполнению дополнительных и основных заданий, ребенок учится учиться, выполнять требования учителя, грамотно планировать свое время, распределять время для учебы и досуга.

    Математические задания для третьеклассников моно составлять самостоятельно по приведенным нами аналогии, это не составит особого труда. Зато ваш ученик сможет больше тренироваться в математике, выполнять задания на каникулах и выходных, а также заниматься дополнительно после школы.

    Сайт vpr-klass.com — впр-класс.ком : гдз, решебник, гиа, егэ, решение задач, задания, варианты, подготовка к экзамену, тесты, презентации.

    Error in links file

     Сайт vpr-klass.com — впр-класс.ком : гдз, решебник, гиа, егэ, решение задач, задания, варианты, подготовка к экзамену, тесты, презентации. 
    Образовательный сайт vpr-klass.com (впр-класс.ком) — готовые решения задач!

    У нас вы найдете много учебных материалов: решебники, ГДЗ, тестовые задания, видео уроки, генераторы задач, решения упражнений гиа и егэ.

    

    Расскажи друзьям


    Ищи САЙТ в Яндексе и Google по слову:
    vpr-klass или впр-класс


    Сохрани сайт в закладки — нажми Ctrl+D

    Презентации


    Детские презентации


    Презентации по математике


    Презентации по астрономии

    Демо-варианты:


    ЕГЭ

    Математика


    Русский язык


    Физика


    Обществознание


    Английский язык


    Информатика


    История


    Биология


    Химия


    Литература


    География


    ГИА (ОГЭ)

    Математика


    Русский язык


    Разделы сайта vpr-klass. com (впр-класс)

    Последние новости ГИА и ЕГЭ 2017.
    ГИА по математике.
    ЕГЭ по математике.
    КДР по математике.
    Математика 1-4 класс.
    Математика 5-6 класс.
    Алгебра и геометрия 7-9 класс.
    Алгебра и геометрия 10-11 класс.
    ГДЗ, решебники по математике, алгебре, геометрии.
    Онлайн калькуляторы по математике.
    Генераторы случайных примеров и задач по математике.
    Презентации.
    Другие школьные предметы.

    Новое на сайте:

    Сайт Vpr-klass.com — это учебный-образовательно-познавательный сайт для школьников!

    Приветствуем на уникальном сайте помощи всем ученикам 1-11 классов. На образовательном ресурсе полно полезной, учебной информации от способов решения заданий по математике до разных генераторов задач по алгебре и онлайн калькуляторов по геометрии, которые облегчат жизнь школьника. В частности, сделан больший уклон на решебники и ГДЗ, ведь правильная домашняя работа — это хорошие оценки и учеба в школе. Также имеется достаточно материалов, которые пригодятся к экзаменам в 9-ых и 11-ых классах. Есть много готовых решенных задач ЕГЭ (ГИА, ОГЭ) и упражнений для отличной самоподготовки к экзаменам. Имеются демонстрационные варианты разных лет и онлайн тесты на основе КИМов для качественной самопроверки знаний. Также есть уникальные генераторы заданий, которые помогут учителям создать карточки для учеников. Есть разделы посвещенные контрольным и самостоятельным и проверочным работам для 3-4-ых и 5-6 классов. Помимо прочего имеются полезные презентации для учителей по разным школьным предметам — биология, обж, информатика, кубановедение, химия и другие. Кроме того есть обучающие видео уроки по математике (ЕГЭ, ГИА, КДР) и информатике (ОГЭ), которые принесут огромную пользу старшеклассникам в подготовке к экзаменам 2018 учебного года.

    

    Интересно


    ГИА (ОГЭ) по математике

    Много разных решений


    Тесты ГИА онлайн.


    Видео — ГИА 2013: геометрия


    Видео — ГИА 2012


    Видео — Демо-вариант 2012.


    Решение Демо-варианта 2013 года (2014 года).


    Задача №1, Вычислить.


    Задача №2, Числа и прямая.


    Задача №3, Сравнение чисел.


    Задача №4, Уравнения.


    Задача №5, Графики и формулы.


    Задача №6, Прогрессии.


    Задача №7, Упростить выражение.


    Задача №8, Неравенства, системы неравенств.


    Задача №9, Задания по геометрии.


    Генератор вариантов ГИА 2014


    ЕГЭ по математике

    Много разных решений.


    Онлайн тесты.


    Видео уроки ЕГЭ по математике.


    Генератор вариантов ЕГЭ 2014


    Книги, справочники


    Решение демо варианта ЕГЭ по математике 2014


    Задания B1, задача.


    Задания B2, диаграммы.


    Задания B5, уравнения.


    Задания B8, производная.


    Задания B10, вероятность.


    ОГЭ по информатике

    Видео уроки

    Copyright © 2017 vpr-klass.com | Если какой-либо из материалов нарушает ваши авторские права, просим немедленно связаться с Администрацией!!! Наш e-mail: [email protected] | Правообладателям | sitemap. xml

    Примеры со скобками, урок с тренажерами. Порядок выполнения действий в выражениях без скобок и со скобками Действия с 1 по 2

    Мы рассмотрим в этой статье три варианта примеров:

    1. Примеры со скобками (действия сложения и вычитания)

    2. Примеры со скобками (сложение, вычитание, умножение, деление)

    3. Примеры, в которых много действий

    1 Примеры со скобками (действия сложения и вычитания)

    Рассмотрим три примера. В каждом из них порядок действий обозначен цифрами красного цвета:

    Мы видим, что порядок действий в каждом примере будет разный, хотя числа и знаки одинаковые. Это происходит потому, что во втором и третьем примере есть скобки.

    *Это правило для примеров без умножения и деления. Правила для примеров со скобками, включающих действия умножения и деления мы рассмотрим во второй части этой статьи.

    Чтобы не запутаться в примере со скобками, можно превратить его в обычный пример, без скобок. Для этого результат, полученный в скобках, записываем над скобками, далее переписываем весь пример, записывая вместо скобок этот результат, и далее выполняем все действия по порядку, слева направо:

    В несложных примерах можно все эти операции производить в уме. Главное — сначала выполнить действие в скобках и запомнить результат, а затем считать по порядку, слева направо.

    А теперь — тренажеры!

    1) Примеры со скобками в пределах до 20. Онлайн тренажер.

    2) Примеры со скобками в пределах до 100. Онлайн тренажер.

    3) Примеры со скобками. Тренажер №2

    4) Вставь пропущенное число — примеры со скобками. Тренажер

    2 Примеры со скобками (сложение, вычитание, умножение, деление)

    Теперь рассмотрим примеры, в которых кроме сложения и вычитания есть умножение и деление.

    Сначала рассмотрим примеры без скобок:

    Есть одна хитрость, как не запутаться при решении примеров на порядок действий. Если нет скобок, то выполняем действия умножения и деления, далее переписываем пример, записывая вместо этих действий полученные результаты. Затем выполняем сложение и вычитание по порядку:

    Если в примере есть скобки, то сначала нужно избавиться от скобок: переписать пример, записывая вместо скобок полученный в них результат. Затем нужно выделить мысленно части примера, разделенные знаками «+» и «-«, и посчитать каждую часть отдельно. Затем выполнить сложение и вычитание по порядку:

    3 Примеры, в которых много действий

    Если в примере много действий, то удобнее будет не расставлять порядок действий во всем примере, а выделить блоки, и решить каждый блок отдельно. Для этого находим свободные знаки «+» и «–» (свободные — значит не в скобках, на рисунке показаны стрелочками).

    Эти знаки и будут делить наш пример на блоки:

    Выполняя действия в каждом блоке не забываем про порядок действий, приведенный выше в статье. Решив каждый блок, выполняем действия сложения и вычитания по порядку.

    А теперь закрепляем решение примеров на порядок действий на тренажерах!

    Если у вас не открываются игры или тренажёры, читайте .

    Для правильного вычисления выражений, в которых нужно произвести более одного действия, нужно знать порядок выполнения арифметических действий. Арифметические действия в выражении без скобок условились выполнять в следующем порядке:

    1. Если в выражении присутствует возведение в степень, то сначала выполняется это действие в порядке следования, т. е. слева направо.
    2. Затем (при наличии в выражении) выполняются действия умножения и деления в порядке их следования.
    3. Последними (при наличии в выражении) выполняются действия сложения и вычитания в порядке их следования.

    В качестве примера рассмотрим следующее выражение:

    Сначала необходимо выполнить возведение в степень (число 4 возвести в квадрат и число 2 в куб):

    3 · 16 — 8: 2 + 20

    Затем выполняются умножение и деление (3 умножить на 16 и 8 разделить на 2):

    И в самом конце, выполняются вычитание и сложение (из 48 вычесть 4 и к результату прибавить 20):

    48 — 4 + 20 = 44 + 20 = 64

    Действия первой и второй ступени

    Арифметические действия делятся на действия первой и второй ступени. Сложение и вычитание называются действиями первой ступени , умножение и деление — действиями второй ступени .

    Если выражение содержит действия только одной ступени и в нём нет скобок, то действия выполняются в порядке их следования слева направо.

    Пример 1.

    15 + 17 — 20 + 8 — 12

    Решение. Данное выражение содержит действия только одной ступени — первой (сложение и вычитание). Надо определить порядок действий и выполнить их.

    Ответ: 42.

    Если выражение содержит действия обеих ступеней, то первыми выполняются действия второй ступени, в порядке их следования (слева направо), а затем действия первой ступени.

    Пример. Вычислить значение выражения:

    24: 3 + 5 · 2 — 17

    Решение. Данное выражение содержит четыре действия: два первой ступени и два второй. Определим порядок их выполнения: согласно правилу первым действием будет деление, вторым — умножение, третьим — сложение, а четвёртым — вычитание.

    Теперь приступим к вычислению.

    Когда мы работаем с различными выражениями, включающими в себя цифры, буквы и переменные, нам приходится выполнять большое количество арифметических действий. Когда мы делаем преобразование или вычисляем значение, очень важно соблюдать правильную очередность этих действий. Иначе говоря, арифметические действия имеют свой особый порядок выполнения.

    В этой статье мы расскажем, какие действия надо делать в первую очередь, а какие после. Для начала разберем несколько простых выражений, в которых есть только переменные или числовые значения, а также знаки деления, умножения, вычитания и сложения. Потом возьмем примеры со скобками и рассмотрим, в каком порядке следует вычислять их. В третьей части мы приведем нужный порядок преобразований и вычислений в тех примерах, которые включают в себя знаки корней, степеней и других функций.

    Определение 1

    В случае выражений без скобок порядок действий определяется однозначно:

    1. Все действия выполняются слева направо.
    2. В первую очередь мы выполняем деление и умножение, во вторую – вычитание и сложение.

    Смысл этих правил легко уяснить. Традиционный порядок записи слева направо определяет основную последовательность вычислений, а необходимость сначала умножить или разделить объясняется самой сутью этих операций.

    Возьмем для наглядности несколько задач. Мы использовали только самые простые числовые выражения, чтобы все вычисления можно было провести в уме. Так можно быстрее запомнить нужный порядок и быстро проверить результаты.

    Пример 1

    Условие: вычислите, сколько будет 7 − 3 + 6 .

    Решение

    В нашем выражении скобок нет, умножение и деление также отсутствуют, поэтому выполняем все действия в указанном порядке. Сначала вычитаем три из семи, затем прибавляем к остатку шесть и в итоге получаем десять. Вот запись всего решения:

    7 − 3 + 6 = 4 + 6 = 10

    Ответ: 7 − 3 + 6 = 10 .

    Пример 2

    Условие: в каком порядке нужно выполнять вычисления в выражении 6: 2 · 8: 3 ?

    Решение

    Чтобы дать ответ на этот вопрос, перечитаем правило для выражений без скобок, сформулированное нами до этого. У нас здесь есть только умножение и деление, значит, мы сохраняем записанный порядок вычислений и считаем последовательно слева направо.

    Ответ: сначала выполняем деление шести на два, результат умножаем на восемь и получившееся в итоге число делим на три.

    Пример 3

    Условие: подсчитайте, сколько будет 17 − 5 · 6: 3 − 2 + 4: 2 .

    Решение

    Сначала определим верный порядок действий, поскольку у нас здесь есть все основные виды арифметических операций – сложение, вычитание, умножение, деление. Первым делом нам надо разделить и умножить. Эти действия не имеют приоритета друг перед другом, поэтому выполняем их в написанном порядке справа налево. То есть 5 надо умножить на 6 и получить 30 , потом 30 разделить на 3 и получить 10 . После этого делим 4 на 2 , это 2 . Подставим найденные значения в исходное выражение:

    17 − 5 · 6: 3 − 2 + 4: 2 = 17 − 10 − 2 + 2

    Здесь уже нет ни деления, ни умножения, поэтому делаем оставшиеся вычисления по порядку и получаем ответ:

    17 − 10 − 2 + 2 = 7 − 2 + 2 = 5 + 2 = 7

    Ответ: 17 − 5 · 6: 3 − 2 + 4: 2 = 7 .

    Пока порядок выполнения действий не заучен твердо, можно ставить над знаками арифметических действий цифры, означающие порядок вычисления. Например, для задачи выше мы могли бы записать так:

    Если у нас есть буквенные выражения, то с ними мы поступаем точно так же: сначала умножаем и делим, затем складываем и вычитаем.

    Что такое действия первой и второй ступени

    Иногда в справочниках все арифметические действия делят на действия первой и второй ступени. Сформулируем нужное определение.

    К действиям первой ступени относятся вычитание и сложение, второй – умножение и деление.

    Зная эти названия, мы можем записать данное ранее правило относительно порядка действий так:

    Определение 2

    В выражении, в котором нет скобок, сначала надо выполнить действия второй ступени в направлении слева направо, затем действия первой ступени (в том же направлении).

    Порядок вычислений в выражениях со скобками

    Скобки сами по себе являются знаком, который сообщает нам нужный порядок выполнения действий. В таком случае нужное правило можно записать так:

    Определение 3

    Если в выражении есть скобки, то первым делом выполняется действие в них, после чего мы умножаем и делим, а затем складываем и вычитаем по направлению слева направо.

    Что касается самого выражения в скобках, его можно рассматривать в качестве составной части основного выражения. При подсчете значения выражения в скобках мы сохраняем все тот же известный нам порядок действий. Проиллюстрируем нашу мысль примером.

    Пример 4

    Условие: вычислите, сколько будет 5 + (7 − 2 · 3) · (6 − 4) : 2 .

    Решение

    В данном выражении есть скобки, поэтому начнем с них. Первым делом вычислим, сколько будет 7 − 2 · 3 . Здесь нам надо умножить 2 на 3 и вычесть результат из 7:

    7 − 2 · 3 = 7 − 6 = 1

    Считаем результат во вторых скобках. Там у нас всего одно действие: 6 − 4 = 2 .

    Теперь нам нужно подставить получившиеся значения в первоначальное выражение:

    5 + (7 − 2 · 3) · (6 − 4) : 2 = 5 + 1 · 2: 2

    Начнем с умножения и деления, потом выполним вычитание и получим:

    5 + 1 · 2: 2 = 5 + 2: 2 = 5 + 1 = 6

    На этом вычисления можно закончить.

    Ответ: 5 + (7 − 2 · 3) · (6 − 4) : 2 = 6 .

    Не пугайтесь, если в условии у нас содержится выражение, в котором одни скобки заключают в себе другие. Нам надо только применять правило выше последовательно по отношению ко всем выражениям в скобках. Возьмем такую задачу.

    Пример 5

    Условие: вычислите, сколько будет 4 + (3 + 1 + 4 · (2 + 3)) .

    Решение

    У нас есть скобки в скобках. Начинаем с 3 + 1 + 4 · (2 + 3) , а именно с 2 + 3 . Это будет 5 . Значение надо будет подставить в выражение и подсчитать, что 3 + 1 + 4 · 5 . Мы помним, что сначала надо умножить, а потом сложить: 3 + 1 + 4 · 5 = 3 + 1 + 20 = 24 . Подставив найденные значения в исходное выражение, вычислим ответ: 4 + 24 = 28 .

    Ответ: 4 + (3 + 1 + 4 · (2 + 3)) = 28 .

    Иначе говоря, при вычислении значения выражения, включающего скобки в скобках, мы начинаем с внутренних скобок и продвигаемся к внешним.

    Допустим, нам надо найти, сколько будет (4 + (4 + (4 − 6: 2)) − 1) − 1 . Начинаем с выражения во внутренних скобках. Поскольку 4 − 6: 2 = 4 − 3 = 1 , исходное выражение можно записать как (4 + (4 + 1) − 1) − 1 . Снова обращаемся к внутренним скобкам: 4 + 1 = 5 . Мы пришли к выражению (4 + 5 − 1) − 1 . Считаем 4 + 5 − 1 = 8 и в итоге получаем разность 8 — 1 , результатом которой будет 7 .

    Порядок вычисления в выражениях со степенями, корнями, логарифмами и иными функциями

    Если у нас в условии стоит выражение со степенью, корнем, логарифмом или тригонометрической функцией (синусом, косинусом, тангенсом и котангенсом) или иными функциями, то первым делом мы вычисляем значение функции. После этого мы действуем по правилам, указанным в предыдущих пунктах. Иначе говоря, функции по степени важности приравниваются к выражению, заключенному в скобки.

    Разберем пример такого вычисления.

    Пример 6

    Условие: найдите, сколько будет (3 + 1) · 2 + 6 2: 3 − 7 .

    Решение

    У нас есть выражение со степенью, значение которого надо найти в первую очередь. Считаем: 6 2 = 36 . Теперь подставим результат в выражение, после чего оно примет вид (3 + 1) · 2 + 36: 3 − 7 .

    (3 + 1) · 2 + 36: 3 − 7 = 4 · 2 + 36: 3 − 7 = 8 + 12 − 7 = 13

    Ответ: (3 + 1) · 2 + 6 2: 3 − 7 = 13 .

    В отдельной статье, посвященной вычислению значений выражений, мы приводим и другие, более сложные примеры подсчетов в случае выражений с корнями, степенью и др. Рекомендуем вам с ней ознакомиться.

    Если вы заметили ошибку в тексте, пожалуйста, выделите её и нажмите Ctrl+Enter

    На данном уроке подробно рассмотрен порядок выполнения арифметических действий в выражениях без скобок и со скобками. Учащимся предоставляется возможность в ходе выполнения заданий определить, зависит ли значение выражений от порядка выполнения арифметических действий, узнать отличается ли порядок арифметических действий в выражениях без скобок и со скобками, потренироваться в применении изученного правила, найти и исправить ошибки, допущенные при определении порядка действий.

    В жизни мы постоянно выполняем какие-либо действия: гуляем, учимся, читаем, пишем, считаем, улыбаемся, ссоримся и миримся. Эти действия мы выполняем в разном порядке. Иногда их можно поменять местами, а иногда нет. Например, собираясь утром в школу, можно сначала сделать зарядку, затем заправить постель, а можно наоборот. Но нельзя сначала уйти в школу, а потом надеть одежду.

    А в математике обязательно ли выполнять арифметические действия в определенном порядке?

    Давайте проверим

    Сравним выражения:
    8-3+4 и 8-3+4

    Видим, что оба выражения совершенно одинаковы.

    Выполним действия в одном выражения слева направо, а в другом справа налево. Числами можно проставить порядок выполнения действий (рис. 1).

    Рис. 1. Порядок действий

    В первом выражении мы сначала выполним действие вычитания, а затем к результату прибавим число 4.

    Во втором выражении сначала найдем значение суммы, а потом из 8 вычтем полученный результат 7.

    Видим, что значения выражений получаются разные.

    Сделаем вывод: порядок выполнения арифметических действий менять нельзя .

    Узнаем правило выполнения арифметических действий в выражениях без скобок.

    Если в выражение без скобок входят только сложение и вычитание или только умножение и деление, то действия выполняют в том порядке, в каком они написаны.

    Потренируемся.

    Рассмотрим выражение

    В этом выражении имеются только действия сложения и вычитания. Эти действия называют действиями первой ступени .

    Выполняем действия слева направо по порядку (рис. 2).

    Рис. 2. Порядок действий

    Рассмотрим второе выражение

    В этом выражении имеются только действия умножения и деления — это действия второй ступени.

    Выполняем действия слева направо по порядку (рис. 3).

    Рис. 3. Порядок действий

    В каком порядке выполняются арифметические действия, если в выражении имеются не только действия сложения и вычитания, но и умножения и деления?

    Если в выражение без скобок входят не только действия сложения и вычитания, но и умножения и деления, или оба этих действия, то сначала выполняют по порядку (слева направо) умножение и деление, а затем сложение и вычитание.

    Рассмотрим выражение.

    Рассуждаем так. В этом выражении имеются действия сложения и вычитания, умножения и деления. Действуем по правилу. Сначала выполняем по порядку (слева направо) умножение и деление, а затем сложение и вычитание. Расставим порядок действий.

    Вычислим значение выражения.

    18:2-2*3+12:3=9-6+4=3+4=7

    В каком порядке выполняются арифметические действия, если в выражении имеются скобки?

    Если в выражении имеются скобки, то сначала вычисляют значение выражений в скобках.

    Рассмотрим выражение.

    30 + 6 * (13 — 9)

    Мы видим, что в этом выражении имеется действие в скобках, значит, это действие выполним первым, затем по порядку умножение и сложение. Расставим порядок действий.

    30 + 6 * (13 — 9)

    Вычислим значение выражения.

    30+6*(13-9)=30+6*4=30+24=54

    Как нужно рассуждать, чтобы правильно установить порядок арифметических действий в числовом выражении?

    Прежде чем приступить к вычислениям, надо рассмотреть выражение (выяснить, есть ли в нём скобки, какие действия в нём имеются) и только после этого выполнять действия в следующем порядке:

    1. действия, записанные в скобках;

    2. умножение и деление;

    3. сложение и вычитание.

    Схема поможет запомнить это несложное правило (рис. 4).

    Рис. 4. Порядок действий

    Потренируемся.

    Рассмотрим выражения, установим порядок действий и выполним вычисления.

    43 — (20 — 7) +15

    32 + 9 * (19 — 16)

    Будем действовать по правилу. В выражении 43 — (20 — 7) +15 имеются действия в скобках, а также действия сложения и вычитания. Установим порядок действий. Первым действием выполним действие в скобках, а затем по порядку слева направо вычитание и сложение.

    43 — (20 — 7) +15 =43 — 13 +15 = 30 + 15 = 45

    В выражении 32 + 9 * (19 — 16) имеются действия в скобках, а также действия умножения и сложения. По правилу первым выполним действие в скобках, затем умножение (число 9 умножаем на результат, полученный при вычитании) и сложение.

    32 + 9 * (19 — 16) =32 + 9 * 3 = 32 + 27 = 59

    В выражении 2*9-18:3 отсутствуют скобки, зато имеются действия умножения, деления и вычитания. Действуем по правилу. Сначала выполним слева направо умножение и деление, а затем от результата, полученного при умножении, вычтем результат, полученный при делении. То есть первое действие — умножение, второе — деление, третье — вычитание.

    2*9-18:3=18-6=12

    Узнаем, правильно ли определен порядок действий в следующих выражениях.

    37 + 9 — 6: 2 * 3 =

    18: (11 — 5) + 47=

    7 * 3 — (16 + 4)=

    Рассуждаем так.

    37 + 9 — 6: 2 * 3 =

    В этом выражении скобки отсутствуют, значит, сначала выполняем слева направо умножение или деление, затем сложение или вычитание. В данном выражении первое действие — деление, второе — умножение. Третье действие должно быть сложение, четвертое — вычитание. Вывод: порядок действий определен верно.

    Найдем значение данного выражения.

    37+9-6:2*3 =37+9-3*3=37+9-9=46-9=37

    Продолжаем рассуждать.

    Во втором выражении имеются скобки, значит, сначала выполняем действие в скобках, затем слева направо умножение или деление, сложение или вычитание. Проверяем: первое действие — в скобках, второе — деление, третье — сложение. Вывод: порядок действий определен неверно. Исправим ошибки, найдем значение выражения.

    18:(11-5)+47=18:6+47=3+47=50

    В этом выражении также имеются скобки, значит, сначала выполняем действие в скобках, затем слева направо умножение или деление, сложение или вычитание. Проверяем: первое действие — в скобках, второе — умножение, третье — вычитание. Вывод: порядок действий определен неверно. Исправим ошибки, найдем значение выражения.

    7*3-(16+4)=7*3-20=21-20=1

    Выполним задание.

    Расставим порядок действий в выражении, используя изученное правило (рис. 5).

    Рис. 5. Порядок действий

    Мы не видим числовых значений, поэтому не сможем найти значение выражений, однако потренируемся применять изученное правило.

    Действуем по алгоритму.

    В первом выражении имеются скобки, значит, первое действие в скобках. Затем слева направо умножение и деление, потом слева направо вычитание и сложение.

    Во втором выражении также имеются скобки, значит, первое действие выполняем в скобках. После этого слева направо умножение и деление, после этого — вычитание.

    Проверим себя (рис. 6).

    Рис. 6. Порядок действий

    Сегодня на уроке мы познакомились с правилом порядка выполнения действий в выражениях без скобок и со скобками.

    Список литературы

    1. М.И. Моро, М.А. Бантова и др. Математика: Учебник. 3 класс: в 2-х частях, часть 1. — М.: «Просвещение», 2012.
    2. М.И. Моро, М.А. Бантова и др. Математика: Учебник. 3 класс: в 2-х частях, часть 2. — М.: «Просвещение», 2012.
    3. М.И. Моро. Уроки математики: Методические рекомендации для учителя. 3 класс. — М.: Просвещение, 2012.
    4. Нормативно-правовой документ. Контроль и оценка результатов обучения. — М.: «Просвещение», 2011.
    5. «Школа России»: Программы для начальной школы. — М.: «Просвещение», 2011.
    6. С.И. Волкова. Математика: Проверочные работы. 3 класс. — М.: Просвещение, 2012.
    7. В.Н. Рудницкая. Тесты. — М.: «Экзамен», 2012.
    1. Festival.1september.ru ().
    2. Sosnovoborsk-soobchestva.ru ().
    3. Openclass.ru ().

    Домашнее задание

    1. Определи порядок действий в данных выражениях. Найди значение выражений.

    2. Определи, в каком выражении такой порядок выполнения действий:

    1. умножение; 2. деление;. 3. сложение; 4. вычитание; 5. сложение. Найди значение данного выражения.

    3. Составь три выражения, в которых такой порядок выполнения действий:

    1. умножение; 2. сложение; 3. вычитание

    1. сложение; 2. вычитание; 3. сложение

    1. умножение; 2. деление; 3. сложение

    Найди значение этих выражений.

    Сегодня мы поговорим о порядке выполнения математических действий . Какие действия выполнять первыми? Сложение и вычитание, или умножение и деление. Странно, но у наших детей возникают проблемы с решением, казалось бы, элементарных выражений.

    Итак, вспомним о том, что сначала вычисляются выражения в скобках

    38 – (10 + 6) = 22 ;

    1) в скобках: 10 + 6 = 16 ;

    2) вычитание: 38 – 16 = 22 .

    Если в выражение без скобок входит только сложение и вычитание, или только умножение и деление, то действия выполняются по порядку слева направо.

    10 ÷ 2 × 4 = 20 ;

    Порядок выполнения действий :

    1) слева направо, сначала деление: 10 ÷ 2 = 5 ;

    2) умножение: 5 × 4 = 20 ;

    10 + 4 – 3 = 11 , т.е.:

    1) 10 + 4 = 14 ;

    2) 14 – 3 = 11 .

    Если в выражении без скобок есть не только сложение и вычитание, но и умножение или деление, то действия выполняются по порядку слева направо, но преимущество имеет умножение и деление, их выполняют в первую очередь, а за ними и сложение с вычитанием.

    18 ÷ 2 – 2 × 3 + 12 ÷ 3 = 7

    Порядок выполнения действий:

    1) 18 ÷ 2 = 9 ;

    2) 2 × 3 = 6 ;

    3) 12 ÷ 3 = 4 ;

    4) 9 – 6 = 3 ; т.е. слева направо – результат первого действия минус результат второго;

    5) 3 + 4 = 7 ; т.е. результат четвертого действия плюс результат третьего;

    Если в выражении есть скобки, то сначала выполняются выражения в скобках, затем умножение и деление, а уж потом сложение с вычитанием.

    30 + 6 × (13 – 9) = 54 , т.е.:

    1) выражение в скобках: 13 – 9 = 4 ;

    2) умножение: 6 × 4 = 24 ;

    3) сложение: 30 + 24 = 54 ;

    Итак, подведем итоги. Прежде чем приступить к вычислению, надо проанализировать выражение: есть ли в нем скобки и какие действия в нем имеются. После этого приступать к вычислениям в следующем порядке:

    1) действия, заключенные в скобках;

    2) умножение и деление;

    3) сложение и вычитание.

    Если вы хотите получать анонсы наших статей подпишитесь на рассылку “ “.

    Сложение, вычитание, умножение и деление (видео и практика)

    TranscriptPractice

    Привет, ребята! Сегодня мы рассмотрим математические операции: сложение , вычитание , умножение и деление . Эти четыре операции служат фундаментальными строительными блоками для всей математики, поэтому крайне важно иметь четкое представление о том, на чем можно основываться. Давайте углубимся.

    Сложение и вычитание

    Мы используем сложение и вычитание для решения многих реальных ситуаций. Сложение и вычитание — это просто математические термины, используемые для описания «объединения» и «удаления». Когда мы к добавляем , мы объединяем или увеличиваем. Когда мы вычитаем , мы отнимаем или уменьшаем.

    Напоминаем:

    • Для сложения используется символ \(+\)
    • Ответ на задачу на сложение называется суммой
    • Для вычитания используется символ \(–\)
    • Ответ на задачу на вычитание называется разностью

     
    По сути, сложение и вычитание — противоположные операции. Один добавляет стоимость, а другой вычитает стоимость. Одна из стратегий визуализации этих двух операций — использование числовой прямой. Мы будем использовать числовую прямую, чтобы проиллюстрировать следующие примеры.

    Давайте представим ситуацию, связанную с продажей попкорна. Для этого сценария предположим, что вы пытаетесь собрать деньги, продавая пакеты с попкорном, и вы начинаете с 20 пакетов.

    Когда придет ваш первый покупатель, он захочет купить 4 пакета попкорна. Это означает, что ваше оставшееся количество сумок уменьшится. Мы можем представить эту ситуацию с помощью простого уравнения, включающего вычитание. Мы начали с 20 мешков и «уменьшили на 4» или вычли 4. Наше уравнение вычитания записывается как \(20-4=16\).

    На числовой прямой мы можем представить это вычитание, начав с 20 и затем переместившись на четыре единицы назад в отрицательном направлении. Каждый прыжок назад представляет собой вычитание на 1.

    Предположим, вы начали с 20 пакетов попкорна, а к концу дня у вас осталось 6 пакетов. Вам нужно пополнить свой запас, чтобы поддерживать продажи, поэтому вы делаете еще 4 пакета попкорна. Сколько пакетов попкорна у вас сейчас есть в наличии для продажи? Для этого сценария, поскольку мы рассматриваем увеличение мешков на , мы будем использовать сложение.

    Эту ситуацию можно описать уравнением \(6+4=10\). Изначально у вас было 6 мешков, а затем «объединили» это количество еще с 4 мешками. Всего у вас 10 мешков. На числовой прямой сложение представлено скачками вправо в положительном направлении. Каждый прыжок вправо представляет собой добавление одной единицы. Так что в этом примере мы бы начали с 6 и прыгнули бы на 4 единицы вправо. Мы видим, что мы приземлились на 10.

    Важно отметить, что при использовании сложения порядок значений не имеет значения. Например, \(10+30\) равносильно \(30+10\). Размещение или расположение значений не влияет на результат. Обе схемы будут равны 40. Однако то же самое не верно для вычитания . Означает ли \(30-10\) то же самое, что и \(10-30\)? Явно нет. Мы видим, что порядок имеет значение при работе с ситуацией, связанной с вычитанием. Технический термин для этого качества известен как 9.0007 коммутативное свойство . По сути, это свойство верно для операций, в которых значения могут перемещаться, «коммутировать», а результат выражения или уравнения не изменится. Коммутативность применима к сложению, но не к вычитанию.

    Умножение и деление

    Другая операция, которая также обладает свойством коммутативности, — это умножение. Давайте обсудим умножение вместе с делением, как мы делали сложение и вычитание. Умножение и деление похожи на сложение и вычитание тем, что выполняют противоположные функции. Функция умножение предназначено для представления нескольких групп определенного значения, тогда как деление предназначено для отображения разделения или подразделения значения на более мелкие группы.

    Напоминаем:

    • Символ, который мы используем для умножения, это \(\times\)
    • Ответ на задачу на умножение называется произведением
    • Символ, который мы используем для деления, это \(\div \)
    • Ответ на задачу деления называется частное

     

    Умножение — это, по сути, удобный и быстрый способ показать то, что называется «повторяющимся сложением». Например, если вам нужно заполнить 30 мешков попкорна, а в каждом мешке требуется 60 зерен, может потребоваться несколько часов, чтобы подсчитать, сколько всего зерен вам нужно, просто используя сложение. Более быстрый и эффективный способ сделать это вычисление — использовать повторное сложение. Вместо того, чтобы считать каждое семя независимо, мы сгруппировали их и сложили группы вместе. Тогда расчет будет состоять из 30 групп по 60. Эта группировка с целью повторного сложения является по своей сути процессом умножения. 30 групп по 60 записывается как \(30\х60=1800\). Таким образом, для заполнения 30 мешков попкорна требуется 1800 ядер.

    И сложение, и умножение коммутативны, потому что порядок не влияет на ответ. 30 групп по 60 дают тот же результат, что и 60 групп по 30. противоположный. Когда мы используем деление, мы, по сути, делим большую группу на более мелкие подгруппы. В нашем примере с попкорном мы можем использовать деление, чтобы ответить на следующий вопрос:

    Сколько пакетов попкорна я могу приготовить из 1800 ядер, если для каждого пакета требуется 60 семян?

     

    Эта ситуация требует, чтобы мы разделили большое значение 1800 на группы по 60. Каждая меньшая подгруппа теперь будет представлять пакет попкорна. 1800, разделенных на группы по 60, представлены как \(1800\div 60\). В данном случае ответ равен 30, значит, из наших 1800 ядер можно приготовить 30 пакетов попкорна. Как видите, деление не является коммутативным, потому что порядок значений играет решающую роль в определении ответа. \(1800\div 60\) — это не то же самое, что \(60\div 1800\).

    Хорошо, это все для этого обзора математических операций! Спасибо за просмотр и удачной учебы!

    Диаграмма умножения и печати

    Практические вопросы

    Вопрос № 1:


    Ответ на проблему вычитания называется …

    Sum

    66. Ответ:

    Ответ:

    A — правильный ответ. Поскольку вычитание — это разница между меньшим числом и большим числом, ответ на задачу на вычитание называется разницей.

    Скрыть ответ

    Вопрос № 2:

     
    Какое утверждение верно?

    Вычитание и умножение — противоположные операции.

    Вычитание и деление — противоположные операции.

    Умножение и сложение — противоположные операции.

    Умножение и деление — противоположные операции.

    Показать Ответ

    Ответ:

    D — правильный ответ. Деление разбивает большую группу на более мелкие подгруппы, а умножение представляет собой многократное сложение меньших подгрупп, чтобы найти общее количество в большой группе. Следовательно, эти операции выполняют противоположные функции.

    Скрыть ответ

    Вопрос №3:

     
    Джейми арендует велосипед за 8 долларов в час. Всего у него велосипед на 4 часа. Какое уравнение можно использовать, чтобы узнать, сколько денег Джейми тратит на аренду велосипеда?

    \(8+4=12\)

    \(8-4=4\)

    \(8\times4=32\)

    \(8÷4=2\)

    Показать ответ

    Ответ:

    C — правильный ответ. Поскольку Джейми тратит 8 долларов за каждый час пользования велосипедом, он тратит 8 долларов + 8 долларов + 8 долларов + 8 долларов, что равно 8 долларов ✕ 4, или 32 доллара.

    Скрыть ответ

    Вопрос № 4:

     
    Какое утверждение лучше всего иллюстрирует свойство коммутативности?

    \(12÷3=3÷12\)

    \(12\times3=3\times12\)

    \(12-3=3-12\)

    \(12+3=15\)

    Показать ответ

    Ответ:

    B — правильный ответ. Свойство коммутативности гласит, что числа в математической задаче можно перемещать или менять местами, и результат уравнения не изменится. Коммутативное свойство применяется к сложению и умножению, но не работает для деления или вычитания.

    Скрыть ответ

    Вопрос № 5:

     
    В Mike’s Deli индейка стоит 4 доллара за фунт. Кейт покупает индейку, чтобы приготовить бутерброды на обед. Если Кейт тратит 80 долларов, какое уравнение показывает, сколько фунтов индейки она купила?

    \(80+4=84\)

    \(4÷80=20\)

    \(80÷4=20\)

    \(80-4=76\)

    Показать ответ

    Ответ:

    C — правильный ответ. Кейт знает, что общая потраченная сумма составила 80 долларов. Она также знает, что каждый фунт стоил ей 4 доллара. Кате нужно знать, на сколько равных групп разбито 80 долларов, если в каждой группе по 4 доллара. Чтобы решить эту проблему, Кате нужно разделить. Поскольку 80 — это число, которое делится, оно стоит первым в задаче на деление. B неверно, потому что \(4÷80\) не совпадает с \(80÷4\).

    Hide Ответ

    Возвращение к базовым арифметическим видео

    208095521157643326359741797942

    Основной добавление, выявление, умножение и диагностика

    All SSAT Верхний уровень. 311 практических тестов Вопрос дня Карточки Learn by Concept

    ← Предыдущая 1 2 3 4 Следующая →

    Справка по математике верхнего уровня SSAT » Понятия и операции с числами » Основное сложение, вычитание, умножение и деление

    Какова сумма двух процентов в десятичной форме?

    7 процентов и 135 процентов

    Возможные ответы:

    Правильный ответ:

    Объяснение:

    Вы складываете проценты и получаете 142 процента. Чтобы преобразовать проценты в десятичные числа, вы делите на 100, поэтому ответ равен 1,42.

    Сообщить об ошибке

    Возможные ответы:

    Правильный ответ:

    Объяснение:

    Выполните выражение в скобках, и вы получите . Затем вы делаете  и получаете . Затем вы делаете  и получаете . Потом добавляешь.

    Сообщить об ошибке

    Если Сьюзи купила пару туфель за , рубашку за и пару брюк за . Сколько всего она потратила?

    Возможные ответы:

    Правильный ответ:

    Объяснение:

    Сложите все три значения и получите .

    Сообщить об ошибке

    Возможные ответы:

    Правильный ответ:

    Объяснение:

    Сначала преобразуйте в десятичное число. У нас осталось

    Теперь сложите три числа, стараясь точно выровнять десятичные точки. Складываем справа налево:

    1) 4 и 5 = 9

    2) 5, 2 и 2 = 9

    3) 2 стоит одна = 2

    Остается 2,99 для окончательного ответа.

    Сообщить об ошибке

     и  являются простыми целыми числами:  и . Сколько возможных значений  есть?

    Возможные ответы:

    Восемь

    Пять

    Семь

    Шесть

    Четыре

    Пять

    4

    5 Пояснение:

     может быть равно любому из простых чисел от 45 до 55, из которых два — 47 или 53.

     может быть равно любому из простых чисел от 15 до 25, которых три — 17, 19, и 23.

    Их сумма может быть любой из следующих:

    Это дает пять возможных сумм.

    Сообщить об ошибке

     и  являются простыми целыми числами. а также . Каково максимально возможное значение  ?

    Возможные ответы:

    Правильный ответ:

    Объяснение:

    Чтобы найти максимально возможное значение , прибавьте максимально возможное значение к максимально возможному значению . Это происходит, когда  равно наибольшему простому числу между 35 и 45, то есть 43, и когда  равно наибольшему простому числу между 85 и 9.5, что равно 89. Сложите два числа:

    Сообщить об ошибке

    В соревновании по математике команда школы Линкольн Хай состояла из пяти учащихся: Мелани, Неда, Оливии, Филлис и Куинн. Мелани опередила Неда на 12 очков; Нед опередил Куинна на 19 очков; Филлис опередила Неда на 45 очков и Оливию на 20 очков. Кто занял третье место среди пятерки?

    Возможные ответы:

    Мелани

    Филлис

    Куинн

    Оливия

    Нед ​​

    Правильный ответ:

    Мелани

    Объяснение:

    Поскольку это рассуждение справедливо независимо от фактических результатов, предположим, что Куинн набрала 100 баллов. Тогда Нед набрал на 19 баллов больше, чем Куинн, или 119, а Мелани набрала на 12 баллов больше, чем Нед, или 131. Филлис набрала на 45 баллов больше, чем Нед, или 164, а Оливия набрала на 20 очков меньше, чем Филлис, или 144. Получается порядок от большего к меньшему:

    Филлис, Оливия, Мелани, Нед, Куинн

    Мелани заняла третье место среди пяти.

    Сообщить об ошибке

    Какой из следующих углов является дополнительным к углу, равному 45 градусам?

    Возможные ответы:

    Правильный ответ:

    Пояснение:

    Дополнительные углы в сумме дают 180 градусов. Учитывая это , правильный ответ – 135. 

    Сообщить об ошибке

      

    Возможные ответы:

    Правильный ответ:

    Объяснение:

    Используйте порядок операций, чтобы сначала выполнить операцию в скобках, что даст вам .

    Затем вы делаете экспоненты, так что .

    Затем вы делаете деление, которое .

    Затем вы выполняете сложение слева направо, так что .

    Наконец, вы добавляете  к , то есть .

    Сообщить об ошибке

    Упрощение:  

    Возможные ответы:

    Правильный ответ:

    Объяснение:

    Чтобы упростить это выражение, перегруппируйте и объедините похожие термины.

    Сообщить об ошибке

    ← Предыдущая 1 2 3 4 Следующая →

    Уведомление об авторских правах 311 практических тестов Вопрос дня Карточки Учитесь по концепции

    Как складывать, вычитать, умножать и делить дроби

    Введение

    Прежде чем вы сможете освоить более сложные понятия алгебры и геометрии, вам необходимо сначала освоить все математические функции, связанные с дробями. В этой статье мы рассмотрим, как складывать, вычитать, умножать и делить две дроби, а также дробь и целое число. Мы также введем сложные дроби вместе с методами их упрощения. Прежде чем продолжить, убедитесь, что вы полностью понимаете четыре основных математических операции: сложение, вычитание, умножение и деление.

    Ключевые условия

    o Общий знаменатель

    o Взаимная

    o Комплексная фракция

    Цели

    O Изучение

    o Понимание, как интерпретировать дробь, которые включают отрицательные числа

    o Признание и упрощенные комплексы

    Теперь, когда мы разработали прочную основу в отношении того, что такое дроби, а также некоторых различных типов дробей, теперь мы можем перейти к применению основных арифметических операций (сложение, вычитание, умножение и деление) к дробям.

    Сложение и вычитание

    В случаях, когда речь идет о простых числах, сложение и вычитание дробей достаточно просты. Например, добавление одной трети и одной трети, очевидно, дает нам две трети. Точно так же три пятых минус две пятых — это одна пятая. Первый случай проиллюстрирован ниже.

    А как быть с такими случаями, как половина плюс одна треть?

    Обратите внимание, что складывать (вычитать) дроби с одинаковым знаменателем очень просто — мы просто складываем (вычитаем) числители и делим на тот же знаменатель. Мы уже должны знать, что можем написать эквивалентные дроби, которые имеют разные числители и знаменатели. Таким образом, если мы просто преобразуем одну или обе дроби, которые мы складываем или вычитаем, в эквивалентные дроби с тем же знаменателем, то мы можем складывать дроби простым способом, описанным выше. Затем, при необходимости, мы можем уменьшить результат до минимальных значений.

    Сложность сложения и вычитания дробей состоит в том, чтобы найти общий знаменатель . Самый простой способ найти общий знаменатель — просто умножить два существующих знаменателя, а затем соответствующим образом преобразовать числители, чтобы получить эквивалентные дроби. Хотя этот подход концептуально прост, он может быть математически сложным, когда знаменатели велики. Тем не менее, давайте попробуем этот подход для иллюстрации. Обратите внимание на упомянутое выше дополнение.

    Общий знаменатель равен 6 (или 23), потому что мы можем умножить числитель и знаменатель на 3, чтобы получить , и мы можем умножить числитель и знаменатель на 2, чтобы получить . Добавление тогда просто.

    Практическая задача: Подсчитайте результат в каждом случае.

    а. б. в.

    Решение: В каждом случае найдите общий знаменатель и преобразуйте члены в эквивалентные дроби с этим знаменателем. Для каждого случая дается один возможный общий знаменатель. Сумма (разность) дробей есть сумма (разность) числителей над общим знаменателем. Если применимо, уменьшите результат до самых низких значений.

    а. Общий знаменатель: 21

    б. Общий знаменатель: 8

    Хотите узнать больше? Почему бы не пройти онлайн-курс Pre-Algebra?

    в. Общий знаменатель: 45

    Умножение и деление

    Умножение и деление дробей в некоторых отношениях проще, чем их сложение и вычитание. Допустим, мы хотим умножить на . Интуитивно ответ довольно очевиден: половина половины — это четверть (или одна четвертая). Например, если у вас есть 50 центов (полдоллара) и вы хотите умножить их на половину, то вы получите 25 центов (четверть доллара).

    Чтобы умножить две дроби, просто умножьте числители и умножьте знаменатели, чтобы получить произведение. В некоторых случаях товар уже будет в наименьших условиях; в других вам может потребоваться уменьшить его до самых низких значений. Например, произведение и будет следующим:

    При умножении дроби на целое число обратите внимание, что любое целое число — это просто дробь с целым числом в числителе и 1 в знаменателе. Например,

    Практическая задача : Рассчитайте следующие произведения.

    а. б. в.

    Решение : В каждом случае произведение равно произведению числителей на произведение знаменателей. Если один из множителей является целым числом, рассматривайте его как дробь, имеющую целое число в числителе и 1 в знаменателе. Сократите продукт до самых низких условий, если это применимо.

    а.

    б.

    в.

    Теперь рассмотрим случай деления. Допустим, мы хотим разделить на . Интуитивно ответ равен 2. Например, 25 центов (четверть доллара) могут дважды превратиться в 50 центов (полдоллара).

     

    Обратите внимание, что если бы мы перевернули второй множитель так, чтобы числитель стал знаменателем, а знаменатель стал числителем, а также изменили операцию деления на умножение, мы получили бы тот же результат.

    На самом деле это удобный способ деления дробей. Деление на дробь равносильно умножению на 9.0007 обратное этой дроби. Обратное — это просто «перевернутая» дробь. Так, например, обратная величина равна (или ).

    Как и при умножении дробей, помните, что целое число также можно записать в виде дроби. Так, например, обратное число 6 равно . Поэтому мы можем делить дроби на целые числа так же, как и на другие дроби. Кроме того, обратите внимание, что произведение дроби и ее обратной всегда равно 1. Рассмотрим пример ниже.

    В свете того, как мы определили деление и умножение, мы можем дать более строгое обоснование нашего метода вычисления эквивалентных дробей. Обратите внимание, что число 1 можно записать как любое другое число, разделенное само на себя. Например,


    Таким образом, процесс нахождения эквивалентных дробей есть не что иное, как умножение данной дроби на 1! Рассмотрим пример ниже.

    Практическая задача : Вычислите следующие частные.

    а. б. в.

    Решение : В каждом случае умножьте делимое на обратную величину делителя. Сократите продукт до самых низких условий, если это применимо.

    а.

    б.

    в.

      

    Дроби и отрицательные числа

    Поскольку дроби — это не что иное, как представление деления, у нас уже есть инструменты, необходимые для понимания роли отрицательных чисел в дробях. Напомним, что произведение (или частное) двух отрицательных или двух положительных чисел положительно, а произведение (или частное) одного отрицательного числа и одного положительного числа отрицательно. Итак, рассмотрим на примере дроби ; мы рассмотрим каждый возможный случай.

    В первом случае (числитель и знаменатель имеют одинаковый знак) результатом является положительное число. Во втором случае (числитель и знаменатель имеют противоположные знаки) результатом является отрицательное число. Таким образом, во втором случае мы можем иногда просто ставить знак минус рядом со всей дробью, а не рядом с числителем или знаменателем. Тем не менее, обратите внимание, что все три представления равны, и в некоторых ситуациях одно может быть более полезным, чем другое.

    Сложные дроби


    Напомним, что дробь — это просто способ выражения деления двух чисел (где числитель — это делимое, а знаменатель — делитель). Поскольку мы можем делить дроби, мы также можем выразить это деление как «долю дробей» или сложных дробей. Ниже приведен пример сложной дроби. Обратите внимание, что для ясности дроби в числителе и знаменателе сложной дроби показаны «наклонными» — однако это изменение не означает никакой математической разницы.

    Такие дроби можно и часто нужно упрощать. Для этого мы можем воспользоваться одним из нескольких подходов. Напомним, что мы можем найти эквивалентную дробь, умножив числитель и знаменатель на одно и то же число. Таким образом, один из подходов состоит в том, чтобы умножить как числитель, так и знаменатель сложной дроби на произведение знаменателей простых дробей, как показано ниже.

    В качестве альтернативы мы можем умножить и числитель, и знаменатель сложной дроби на обратную величину ее знаменателя. Поскольку знаменатель становится равным 1, результатом является просто значение числителя.

    Другой способ взглянуть на этот последний подход состоит в том, что мы просто выполняем деление:

    В зависимости от конкретной ситуации один подход может быть проще другого; однако все они одинаково приемлемы.

    Практическая задача : Упростите следующие сложные дроби.

    а. б. в.

    Решение : Одним из возможных способов упрощения сложных дробей является умножение дроби в числителе на обратную дробь в знаменателе. Если применимо, уменьшите результат до самых низких значений. В случае части c обратите внимание, что обратная величина 5 равна и что частное (или произведение) положительного числа, деленного (умноженного) на отрицательное число, является отрицательным числом.

    а.

    б.

    в.

    Если у вас по-прежнему возникают проблемы с дробями, вы также можете прочитать эту статью здесь: Как сделать дроби простым способом.

    Формула для сложения, вычитания, умножения и деления в Excel

    Сложение, вычитание, умножение и деление часто используются при математических вычислениях в Excel. Эта статья покажет, как легко и быстро выполнять эти операции с помощью функции или формулы, с подходящими примерами и соответствующими иллюстрациями.


    Формула сложения, вычитания, умножения и деления в Excel

    1. Формула Excel для сложения

    1.1 Использование базовой формулы сложения

    1.2 Использование функции СУММ

    2. Формула вычитания в Excel

    3. Как умножать в Excel

    3. 1 Использование базовой формулы умножения

    3.2 Использование функции ПРОДУКТ

    4. Основная математическая формула деления.

    Дополнительные возможности для деления в Excel

    Вывод

    Статьи по Теме

    Формула сложения, вычитания, умножения и деления в Excel

    Существуют некоторые различия между математическим оператором и операторами, используемыми в Excel. Итак, я показываю операторы сложения, вычитания, умножения и деления в Excel в таблице ниже.

    Эксплуатация Математический оператор Оператор Excel Пример  Результат
    Сумма + +   
    (клавиша плюс)
    = 10 + 5 15
    Вычитание – 
    (минус ключ)
    = 15-10 5
    Умножение х *
    (звездочка или звездочка)
    = 5 * 3  15
    Подразделение ÷ 
    (рывок вперед)
    = 15/5 5

    Теперь мы рассмотрим каждую операцию с некоторыми примерами в следующих разделах.


    1. Формула Excel для сложения

    1.1 Использование базовой формулы сложения
    • Чтобы сложить значения C5 и D5, вы можете просто написать формулу со значениями:

    =800+125

    • В противном случае для записи формулы можно использовать ссылку на ячейку. Вы можете вызвать ячейку, либо набрав столбец ячейки и номер строки, либо просто щелкнув ячейку.

    =C5+D5

    • Если вы используете ссылку на ячейку, то вы можете скопировать формулу с помощью CTRL + C и вставить ее в другие ячейки с помощью CTRL + V .


    1.2 Использование функции СУММ
    • Еще один метод сложения в Excel — использование функции СУММ. Чтобы вызвать функцию СУММ , вы должны написать в ячейке « =сумма» . И там откроются некоторые предложения. Затем выберите функцию СУММ .

    • Затем внутри функции суммирования выберите ячеек , которые будут добавлены. Здесь я сделал горизонтальное сложение, используя эту формулу:

    =СУММ(C5:D5)

    • Точно так же вы можете выполнить сложение по вертикали, используя эту формулу:

    =СУММ(Е5:Е9)

    • Вы также можете вызвать функцию SUM из меню ленты. Для этого сначала нажмите на ячейку, в которую вы хотите добавить значения. Перейдите на верхнюю ленту, нажмите на опцию Formulas . Затем выберите AutoSum , и откроется раскрывающееся меню. Выберите параметр СУММА .

    • После нажатия на опцию SUM вы увидите автоматический выбор ячеек. Если предложение верное, нажмите Enter.  Если нет, исправьте ссылки на ячейки и нажмите Enter.

    Подробнее: Как сложить и разделить в Excel (5 подходящих примеров)


    2. Формула вычитания в Excel

    • Чтобы вычесть значения, вы можете просто написать эту формулу, используя значения:

    =800-125

    • Кроме того, вы можете использовать ссылку на ячейку для вычитания значений по этой формуле:

    =C5-D5

    Использование ссылки на ячейку является хорошей практикой, так как это позволит вам копировать и вставлять в другие ячейки, что сделает вашу работу быстрее и эффективнее. Есть нет функции вычитания в Excel.


    3. Как умножать в Excel

    3.
    1 Использование базовой формулы умножения
    • Чтобы умножить в Excel, вы должны использовать знак звездочки ( * ). Вы можете просто умножить значения, написав эту формулу:

    =800*80

    • Вы также можете использовать ссылку на ячейку для умножения значений. Для этого вы можете написать такую ​​формулу:

    =C5*D5

    Использование ссылки на ячейку позволит вам копировать и вставлять формулу в другие ячейки.


    3.2 Использование функции ПРОДУКТ
    • Вы также можете выполнять умножение в Excel с помощью функции ПРОИЗВЕД . Для этого нажмите на ячейку, где вы хотите получить результат. Затем напишите ‘=product’. Будут предложения. Из них выберите ПРОДУКТ 9Вариант 0008.

    • После выбора опции ПРОДУКТ выберите ячейки, которые вы хотите умножить, и нажмите Enter.

    Теперь вы можете скопировать формулу и вставить ее в другие ячейки, чтобы произвести умножение аналогичным образом.


    4. Основная математическая формула деления

    Чтобы разделить значений в Excel на , используется ‘/’ или знак тире вперед. Чтобы разделить значения, вы можете просто написать формулу, подобную этой, с указанием значений:

    =800/15

    • Кроме того, вы можете вызывать ссылки на ячейки для разделения значений. Для этого вы пишете такую ​​формулу:

    =C5/D5

    Затем вы можете скопировать и вставить формулу в другие ячейки, чтобы выполнить аналогичное деление .

    Подробнее: Как разделить в Excel на весь столбец (7 быстрых приемов)


    Дополнительные возможности для деления в Excel

    Использование функции QUOTIENT:

    В предыдущем вычислении деления вы можете видеть, что результаты представлены в дробном формате. И есть несколько цифр после точки. Часто вы хотите получить результат в ближайшем целочисленном значении. Затем вы можете использовать функцию ЧАСТНОЕ .

    • Чтобы вызвать функцию ЧАСТНОЕ , просто напишите «= частное» в ячейках. И появятся какие-то предложения. И выберите ЧАСТНОЕ  опция.

    • В параметре ЧАСТНОЕ выберите ячейку, в которой находится значение делимого , а затем выберите ячейку со значением делителя . Здесь числитель обозначает значение дивиденда, а знаменатель — значение делителя.

    • Вы также можете скопировать и вставить формулу в другие ячейки, чтобы аналогичным образом найти значения частных.


    Используйте функцию MOD, чтобы найти остаток:

    Опять же, иногда вам может понадобиться найти остаток после деления на . Чтобы найти значение остатка, вы можете использовать функцию MOD .

    • Для этого просто напишите ‘=Mod’ и появятся предложения и выберите функцию MOD .

    • Затем в функции MOD сначала выберите ячейку значения дивиденда, затем поставьте запятая и затем выберите ячейку значения делителя. Затем нажмите Enter.

    • Вы можете скопировать и вставить формулу в другие ячейки, чтобы аналогичным образом найти остальные значения.


    Заключение

    Мы должны использовать формулы для сложения, вычитания, умножения и деления в Excel, регулярно делая любые математические вычисления. Новичкам может быть сложно найти способы сложения, вычитания, умножения и деления в Excel. Итак, в этой статье я попытался показать формулы и функции для сложения, вычитания, умножения и 9.0007 деление в excel. Я надеюсь, что вы нашли эту статью полезной. Вы можете посетить наш веб-сайт ExcelDemy , чтобы узнать больше, связанное с Excel. Пожалуйста, оставьте комментарии, предложения или вопросы, если они у вас есть, в разделе комментариев ниже.


    Связанные статьи

    • Как разделить значение, чтобы получить процент в Excel (5 подходящих примеров)
    • [Исправлено] Формула деления не работает в Excel (6 возможных решений)
    • Как разделить число на процент в Excel (3 подходящих примера)

    Математика для детей (Основы сложения, вычитания, умножения, деления) — Тождества триггеров

    Что такое базовая математика?

    Базовая математика определяется как «наука о количестве», она дает вам основные концепции математики. Каждый учащийся должен знать эти понятия и практиковать их в повседневной жизни, решая текстовые задачи.

    Базовая математика — это простые понятия, которые подготавливают учащихся к изучению дробей, НОК и НОД. Обычно основная математика — это счет, сложение, вычитание, умножение и деление. Все математические понятия основаны на этих четырех операциях (сложение, вычитание, умножение и деление). С этими операциями учащийся также должен понимать различные свойства и связь с этими операциями.

    Вот основные понятия математики, а также примеры для учащихся, которые хотят изучать базовую математику. Сегодня мы собираемся обсудить математику для детей , что очень важно для начала изучения математики.

    Их также называют натуральными числами, потому что они, естественно, являются первыми числами, которые мы изучаем (1, 2, 3, 4 и т. д.). Иногда их также называют положительными числами.

    Эти арифметические операции используются для подсчета физических объектов в мире.

    0, 1, 2, 3, 4, 5, 6, 7, 8, 9, 10…

    Ежедневные примеры

    Ответ : , если сегодня понедельник, оставшееся 6 дней. .

    Ответ: есть 20 чисел.

    • Сосчитайте носки, чтобы убедиться, что у вас есть совпадающие пары (2).
       Ответ:   Если у вас есть 6 носков, то у вас есть 12 одинаковых пар. 
    • Подсчитайте, сколько людей в вашем доме.
    • Подсчитайте учеников в вашем классе.
    • Подсчитайте, сколько дней осталось до ваших бумаг.
    • Сосчитайте все алфавиты. (А, В, С… и так далее).
    • Расположите все эти числа в правильном порядке (98, 23, 67, 12, 45, 56 и 10).

    Ваш ребенок умеет считать, если правильно ответит на все вопросы. Хотя ему/ей нужно немного практики ежедневно.

    Сложение

    Сложение — одна из основных математических операций. Простыми словами сложение означает соединение двух вещей вместе или добавление двух вещей. Его также можно определить как набор объектов или их сложение вместе. Он представлен

    Знак «+».

    Примеры

    Допустим, у вас есть 2 синих мяча, а у вашего друга Джона 3 красных мяча. Вы и Джон думаете, что можете подсчитать количество мячей, чтобы у вас обоих было больше мячей для игры, но сначала вы находите общее количество мячей у каждого из них.

    Вместо того, чтобы считать каждый мяч отдельно, вы можете использовать базовую математическую операцию (Сложение), чтобы быстрее найти общее количество мячей. Сложение — это когда мы складываем два числа вместе, чтобы найти сумму чисел.

    У вас 2 мяча, а у Джона 3 мяча, поэтому сумма мячей равна 5.

    2 синих шара + 3 красных шара = 5 мячей

    Пример 2 4 печенья. Они хотят испечь кекс, но для этого им нужно подсчитать, сколько у них печенья.

    Вместо того, чтобы подсчитывать отдельные файлы cookie, они просто используют базовую арифметическую операцию (всего) для сложения чисел.

    4 куки + 4 куки = 8 куки

    Пример

    Решите вопросы по добавлению

    5 12 34 56 47

    +19 +34 +14 +34 +23 9

    9565. :                        24              46              48              90                  70

     

    Вычитание

    Это основная арифметическая операция, используемая для нахождения разницы между двумя числами. Когда у вас много денег и вы вычитаете из них половину, то денег становится меньше. В вычитании есть три части, начальное число — уменьшаемое. Вычитаемое число является вычитаемым, а число, оставшееся после вычитания, известно как разность.

    Методы вычитания

    Существуют различные методы вычитания. Один из них заключается в простом вычитании двух чисел, как указано ниже. ?

    5 яиц – 2 яйца = 3 яйца.

    Вычесть

    А

    12 34 23 13 7 3 67
        _

    Б

    2 32 12 10 3 1 34
    А-Б 10 2 11 3 4 2 33

     

    Умножение

    Вы знаете это или нет, но иногда вы делаете умножение, просто складывая их.

    Есть два метода умножения.

     

    Метод 1:

    Пример:

    На приведенном выше рисунке есть три группы, и каждая из этих групп имеет 6 файлов cookie.

    Таким образом, общее количество файлов cookie в трех группах равно 5 + 5 + 5 = 15 файлов cookie.

    Пример 2:

    Допустим, у вас есть три группы, и в каждой группе по 6 маркеров, поэтому вы подсчитываете общее количество маркеров.

    Итак,             6 маркеров + 6 маркеров + 6 маркеров = 18 маркеров.

    Метод 2:

    Примеры

    Умножение

    А

    12 6 5 9 4 3 4
        *

    Б

    2 5 3 10 3 1 8
    А * В 24 12 15 90 12 3 32

     

    Пример:

    11 13 2 4 4 9 7 6

    * 2* 4* 5* 3* 2* 7* 2

    ______________________________________________________________

    ОТВЕТ 22 52 10 12 18 49 12

    Дивизион

    9

    Дивизион

    69

    Дивизион 9000

    69

    Дивизион 9000

    69

    Дивизион 1666

    9

    Дивизион 1666

    9

    . это просто процесс деления числа на две равные части. Когда вы делите число, вы начинаете с большого числа, а после деления оно делится на свою половину. Делимым является первое число; делитель — это число, которое вы хотите разделить, а частное — это ответ числа.

    Дивиденд 10/ делитель 2 = частное 5

    Для представления деления у нас есть два оператора

    1. / косая черта деления.
    2. ÷ Знак деления.

    Чтобы разделить любые числа, вам всегда нужны следующие шаги:

    • Разделить
    • Умножить
    • Вычесть
    • Опустить номер вниз

    Повторяйте эти шаги, пока проблема не будет решена.

    Случаи:

    При делении двух чисел учитываются некоторые случаи.

    • Когда мы делим любое число на единицу, всегда получается исходное число.

    20 / 1 = 20                           5 / 1 = 5

    • Делится на ноль: при делении числа на ноль ответ всегда не определен. Другими словами, вы не можете разделить число на ноль.

    20/0= не определено                40/0=не определено.

    Если делимое равно делителю, то ответ всегда равен единице.

    20/20 = 1                     2)   5/5 = 1

    Пример:

    Отдел

    А

    12 6 12 10 20 3 8
        /

    Б

    2 3 3 2 5 1 4
    А/Б 6 2 4 5 4 3 2

     

     

    Вы можете узнать больше о математике на trigidentities. info

     

     

    Числовой ряд и сложение, вычитание, умножение и деление

    Авторы: Марк Зегарелли и

    Обновлено: 26 марта 2016

    Базовая математика и предварительная алгебра для чайников
    Книга 9004 Подробнее

    Числовая строка — это просто строка с числами, отмеченными через равные промежутки. Вы, вероятно, увидели свой первый числовой ряд, когда учились считать до десяти. Вы можете использовать этот надежный инструмент для выполнения операций Большой четверки (сложение, вычитание, умножение и деление) над относительно небольшими числами.

    Числовая строка может быть полезным инструментом для сложения и вычитания небольших чисел:

    • При сложении переместите вверх по числовой строке вправо.

    • При вычитании переместите вниз по числовой строке влево.

    Чтобы умножить на числовой прямой, начните с 0 и посчитайте первое число в задаче столько раз, сколько указано вторым числом .

    Чтобы разделить на числовой прямой, сначала закройте отрезок числовой строки от 0 до первый номер в задаче. Затем разделите этот отрезок поровну на количество частей, указанное вторым числом . Длина каждой части — это ответ на деление.

    Примеры вопросов

    1. Добавьте 6 + 7 в числовой строке.

      13. Выражение 6 + 7 означает , начиная с 6, до 7, , что приводит к 13.

    2. Вычтите 12 – 4 в числовой строке.

      8. Выражение 12 – 4 означает, что начинается с 12, вниз на 4, , что приводит к 8.

    3. Умножьте 2 x 5 на числовой прямой.

      10. Начиная с 0, сосчитайте двойками всего пять раз, чтобы получить 10.

    4. Разделите 12 / 3 на числовой прямой.

      4. Закрасьте отрезок числовой прямой от 0 до 12. Теперь разделите этот отрезок поровну на три меньшие части. Каждая из этих частей имеет длину 4, так что это ответ на задачу.

    Практические вопросы

    1. Добавьте в числовую строку следующие числа:

      а. 4 + 7 = ?
      б. 9 + 8 = ?
      г. 12 + 0 = ?
      д. 4 + 6 + 1 + 5 = ?
    2. Вычтите из числовой строки следующие числа:

      а. 10 – 6 = ?
      б. 14 – 9 = ?
      г. 18 – 18 = ?
      д. 9 – 3 + 7 – 2 + 1 = ?
    3. Умножьте следующие числа на числовой прямой:

      а. 2 х 7 = ?
      б. 7 х 2 = ?
      г. 4 х 3 = ?
      д. 6 х 1 = ?
      эл. 6 х 0 = ?
      ф. 0 х 10 = ?
    4. Разделите следующие числа на числовой прямой:

      а. 8/2 = ?
      б. 15/5 = ?
      г. 18/3 = ?
      д. 10/10 = ?
      эл. 7/1 = ?
      ф. 0 / 2 = ?

    Ответы на практические вопросы следующие:

    1. Добавить в числовую строку.

      а. 4 + 7 = 11 . Выражение 4 + 7 означает, что начинается с 4, до 7, , что приводит к 11.
      б. 9 + 8 = 17 . Выражение 9 + 8 означает, что начинается с 9, до 8, , что приводит к 17.
      г. 12 + 0 = 12 . Выражение 12 + 0 означает, что начинается с 12, до 0, , что приводит к 12.
      д. 4 + 6 + 1 + 5 = 16 . Выражение 4 + 6 + 1 + 5 означает, что начинается с 4, до 6, до 1, до 5, , что приводит к 16.
    2. Вычесть из числовой строки.

      а. 10 – 6 = 4 . Выражение 10 – 6 означает, что начинается с 10, уменьшается на 6, , что приводит к 4.
      б. 14 – 9 = 5 . Выражение 14 – 9 означает, что начинается с 14, вниз на 9, , что приводит к 5.
      г. 18 –18 = 0 . Выражение 18 – 18 означает, что начинается с 18, уменьшается на 18, , что приводит к 0,
      д. 9 – 3 + 7 – 2 + 1 = 12 . Выражение 9 – 3 + 7 – 2 + 1 означает, что начинается с 9, вниз 3, вверх 7, вниз 2, вверх 1, , что приводит к 12.
    3. Умножить на числовой прямой.

      а. 2 х 7 = 14 . Начиная с 0, посчитайте двойками семь раз, чтобы получить 14.
      б. 7 х 2 = 14 . Начиная с 0, сосчитайте по семеркам всего два раза, чтобы получить 14.
      г. 4 х 3 = 12 . Начиная с 0, посчитайте по четыре в общей сложности три раза, что приведет вас к 12.
      д. 6 х 1 = 6 . Начиная с 0, сосчитайте до шести один раз, и вы получите 6.
      эл. 6 х 0 = 0 . Начиная с 0, сосчитайте по шесть ноль раз, и вы получите 0.
      ф. 0 х 10 = 0 . Начиная с 0, сосчитайте нулями десять раз, чтобы получить 0.
    4. Разделить по числовой прямой.

      а. 8/2 = 4 . Заблокируйте отрезок числовой прямой от 0 до 8. Теперь разделите этот отрезок поровну на две меньшие части. Каждая из этих частей имеет длину 4, так что это ответ на задачу.
      б. 15/5 = 3 . Заблокируйте отрезок числовой прямой от 0 до 15. Разделите этот отрезок поровну на пять меньших частей. Каждая из этих частей имеет длину 3, так что это ответ на задачу.
      г. 18/3 = 6 . Заблокируйте отрезок числовой прямой от 0 до 18 и разделите этот отрезок поровну на три меньшие части. Каждая часть имеет длину 6, ответ на задачу.
      д. 10/10 = 1 . Заблокируйте отрезок числовой прямой от 0 до 10 и разделите этот отрезок поровну на десять меньших частей. Каждая из этих частей имеет длину 1,
      эл. 7 / 1 = 7 . Отметьте отрезок числовой прямой от 0 до 7 и разделите этот отрезок поровну на 1 часть (то есть вообще не делите). Этот кусок по-прежнему имеет длину 7.
      ф. 0 / 2 = 0 .

    admin

    Добавить комментарий

    Ваш адрес email не будет опубликован. Обязательные поля помечены *